Vous êtes sur la page 1sur 204

Apresentao

Caros professores orientadores e alunos do Programa de Iniciao Cientfica PIC da OBMEP Este um nmero especial da Revista do Professor de Matemtica RPM, elaborado para utilizao no PIC da OBMEP a ser realizado a partir do primeiro semestre de 2009. A RPM, como seu nome diz, uma revista dedicada aos professores de Matemtica da educao bsica, a alunos e professores de cursos de licenciatura em Matemtica e a todos aqueles que se interessam pela Matemtica do nvel mdio. O tratamento dado aos temas abordados procura ser acessvel e agradvel, sem sacrificar o rigor. A revista uma publicao da Sociedade Brasileira de Matemtica SBM e tem sido editada e distribuda sem interrupes desde 1982. A revista publica crnicas, artigos e sees, como Problemas, O leitor pergunta, Livros, Olhando mais de cima, etc. Nos artigos, temas interessantes de nvel elementar ou avanado so apresentados de modo acessvel ao professor e ao aluno do ensino bsico ou de cursos de Licenciatura em Matemtica. Uma experincia interessante em sala de aula, um problema que suscita uma questo pouco conhecida, uma histria que merea ser contada ou at uma nova abordagem de um assunto conhecido. Nas sees, a revista conversa com o leitor, publicando problemas e/ou solues propostas por eles, cartas, resenhas de livros, erros encontrados em textos didticos, etc.

RPM OBMEP

Para este exemplar especial, o Comit Editorial da RPM escolheu artigos que pretendem ampliar o conhecimento dos alunos em diferentes tpicos, bem como temas que motivem discusses ou satisfaam curiosidade terica e histrica de alunos interessados em Matemtica. Alm disso, publicamos vrias propostas de atividades que podem ser aplicadas nas salas de alunos de ensino fundamental e mdio. As atividades tentam despertar a curiosidade dos alunos para tpicos importantes da Matemtica que so explicitados nas justificativas dos procedimentos propostos. Apresentamos tambm uma seo Problemas com: Problemas I: problemas interessantes com nmeros primos. Problemas II: uma seleo de problemas extrados do PISA (Programme for International Student Assessment Programa Internacional de Avaliao de Alunos). Problemas III: vinte problemas selecionados entre os publicados na seo Problemas da RPM, que abrangem a maioria dos tpicos da educao bsica. As solues dos problemas propostos esto no final deste fascculo. Os artigos aqui publicados no apresentam as referncias bibliogrficas, citaes ou agradecimentos que constam nos artigos originais da RPM. Comit Editorial da RPM

RPM OBMEP

Contedo
Atividades em sala de aula . . . . . . . . . . . . . . . . . . . . . . . . . . . . . . . 05 Um jogo aritmtico . . . . . . . . . . . . . . . . . . . . . . . . . . . . . . . . . . . . . . . 12 Seis problemas no triviais equivalentes . . . . . . . . . . . . . . . . . . . . . . . 16 O menino . . . . . . . . . . . . . . . . . . . . . . . . . . . . . . . . . . . . . . . . . . . . . 22 O problema dos cinco discos: sorte ou sabedoria? . . . . . . . . . . . . . . . 26 Calculadora padro: um problema interessante . . . . . . . . . . . . . . . . . . 31 Uma equao interessante . . . . . . . . . . . . . . . . . . . . . . . . . . . . . . . . . 37 Painis
Painel I O nmero 12 . . . . . . . . . . . . . . . . . . . . . . . . . . . . . . . . . . . . . . . . . . . . . . Painel II Sexta-feira 13 . . . . . . . . . . . . . . . . . . . . . . . . . . . . . . . . . . . . . . . . . . . . . . Painel III O jogo de bilhar . . . . . . . . . . . . . . . . . . . . . . . . . . . . . . . . . . . . . . . . . . . . . Painel IV Codificando e decifrando mensagens . . . . . . . . . . . . . . . . . . . . . . . . . . . . Painel V Qual a relao entre os nmeros 102.564 e 410.256? . . . . . . . . . . . . . . . Painel VI Uma demonstrao visual para a frmula do sen(A + B) . . . . . . . . . . . . . Painel VII Valores irracionais de funes trigonomtricas . . . . . . . . . . . . . . . . . . . . Painel VIII Mgica com nmeros . . . . . . . . . . . . . . . . . . . . . . . . . . . . . . . . . . . . . . . . . Painel IX Destreza ou esperteza? . . . . . . . . . . . . . . . . . . . . . . . . . . . . . . . . . . . . . . . Painel X Determinante para fatorar . . . . . . . . . . . . . . . . . . . . . . . . . . . . . . . . . . . . .

41 43 45 47 49 51 52 54 56
RPM OBMEP

58

Funes interessantes . . . . . . . . . . . . . . . . . . . . . . . . . . . . . . . . . . . 60 A formiga inteligente . . . . . . . . . . . . . . . . . . . . . . . . . . . . . . . . . . . 66 A demonstrao feita por Heron . . . . . . . . . . . . . . . . . . . . . . . . . . . . . 72

A Matemtica da folha de papel A4 . . . . . . . . . . . . . . . . . . . . . . . . . .

75

Retngulo ureo, diviso urea e sequncia de Fibonacci . . . . . . . . . 82 Usando Geometria para somar . . . . . . . . . . . . . . . . . . . . . . . . . . . . . 95 Mdias . . . . . . . . . . . . . . . . . . . . . . . . . . . . . . . . . . . . . . . . . . . . . . . 100 Problemas diversos resolvidos com Geometria Analtica . . . . . . . . 112 A sombra do meu abajur . . . . . . . . . . . . . . . . . . . . . . . . . . . . . . . . 119 Ilha do tesouro. Dois problemas e duas solues . . . . . . . . . . . . . . . 125 Qual mesmo a definio de polgono convexo? . . . . . . . . . . . . . . . 129 A soluo de Tartaglia para a equao do 3o grau e a emergncia dos nmeros complexos . . . . . . . . . . . . . . . . . . . . . . . 135 Grandezas incomensurveis e nmeros irracionais . . . . . . . . . . . . . 153 A outra face da moeda honesta . . . . . . . . . . . . . . . . . . . . . . . . . . . . . . 162 Nmero de regies: um problema de contagem . . . . . . . . . . . . . . . . . 165 Intuio e Probabilidade . . . . . . . . . . . . . . . . . . . . . . . . . . . . . . . . . . . 170 Problemas I . . . . . . . . . . . . . . . . . . . . . . . . . . . . . . . . . . . . . . . . . . . . 172 Problemas II . . . . . . . . . . . . . . . . . . . . . . . . . . . . . . . . . . . . . . . . . . . . 175 Problemas III . . . . . . . . . . . . . . . . . . . . . . . . . . . . . . . . . . . . . . . . . . . . 181 Soluo dos Problemas . . . . . . . . . . . . . . . . . . . . . . . . . . . . . . . . . . . . 185

RPM OBMEP

Atividades em sala de aula

Aprende-se Matemtica fazendo Matemtica. Apresentamos aqui dois grupos de atividades que permitem a alunos do ensino fundamental fazer Matemtica. As folhas contendo as atividades foram copiadas de nmeros da revista The Mathematics Teacher* (O professor de Matemtica). No incio da primeira atividade, alguns exemplos podem ser feitos coletivamente. Ao fim de cada atividade interessante comparar os resultados obtidos pelos alunos reforando o fato de que um mesmo problema pode ter vrias solues. O primeiro grupo de atividades trabalha as operaes aritmticas com nmeros inteiros. O segundo grupo de atividades trabalha com visualizao de figuras no espao, permitindo aos alunos descobrir padres e fazer conjecturas. As primeiras partes podem ser aplicadas em salas do ensino fundamental e a parte final no ensino mdio, explorando generalizaes e suas representaes algbricas.
RPM OBMEP

Na pgina 15 deste exemplar est uma soluo de cada problema proposto no Primeiro grupo.
* Publicao do National Council of Teachers of Mathematics NCTM, Reston, Virginia, USA.

Primeiro grupo: atividades I, II e III. I. Instrues


Em cada linha h 5 nmeros e um sexto nmero, chamado total. Coloque os sinais +, , , e parntesis, colchetes, chaves, de

modo que o resultado das contas indicadas seja o total.


Os 5 nmeros devem ser usados, cada um deles uma s vez, em

qualquer ordem. Exemplo: 7, 8, 1, 9, 9 total: 16. Uma soluo: (9 9) (7 + 8 + 1) = 16. 1. 1, 5, 3, 6, 10 2. 8, 11, 9, 1, 8 total: 5 total: 2

3. 11, 10, 15, 20, 3 total: 6 4. 12, 18, 3, 11, 12 total: 8 5. 4, 16, 10, 24, 25 total: 1 6. 17, 14, 7, 17, 13 total: 7 7. 2, 9, 5, 9, 4 8. 3, 6, 10, 5, 7 9. 8, 6, 11, 5, 21 10. 6, 1, 2, 2, 17 total: 22 total: 2 total: 7 total: 8

RPM OBMEP

II. Instrues
Trabalhe com os nmeros 11, 14, 3, 19 e 9. Coloque os sinais +, , , e parntesis, colchetes, chaves, para obter todos os nmeros de 1 at 11. Os nmeros 11, 14, 3, 19 e 9 devem ser usados, cada um deles uma

s vez, em qualquer ordem. 1. (11 + 14 19 + 3) 9 = 1 2. 3. 4. 5. 6. 11 [(19 + 9) 14 + 3] = 6 7. 8. 9. 10. 11. [9 (19 14) 3] 11 = 11


RPM OBMEP

III. Instrues
Trabalhe com os nmeros 2, 3, 5, 7 e 11. (Observe que so os cinco

primeiros nmeros primos.)


Coloque os sinais +, , , e parntesis, colchetes, chaves, para

obter os nmeros pedidos nos itens de 1 a 10 abaixo.


Os nmeros 2, 3, 5, 7 e 11 devem ser usados, cada um deles uma s

vez, em qualquer ordem. 1. Escreva (seguindo as instrues) o menor primo mpar. 3 = [(2 5) + (7 3)] 11 2. Escreva o menor nmero natural mpar. 3. Escreva o menor nmero natural primo. 4. Escreva o menor nmero natural composto. 5. Qual o maior nmero natural composto que voc consegue escrever? 6. Qual o maior nmero natural mpar que voc consegue escrever? 7. Escreva o menor nmero natural que voc consegue achar, usando uma s vez cada uma das operaes. 8. Determine e escreva o maior nmero natural par possvel, usando uma s vez cada uma das operaes. 9. Escreva um nmero natural usando apenas subtraes.
RPM OBMEP

10. Determine e escreva o maior nmero primo possvel obedecendo s instrues.

Segundo grupo Descobertas com cubos: atividades I, II, III e IV Descrio Um cubo grande, decomposto em cubos pequenos, mergulhado numa lata com tinta. Pergunta-se quantas faces dos cubos pequenos ficaro pintadas. Objetivos Estudantes visualizaro figuras no espao, construiro uma tabela, descobriro padres na tabela e, usando esses padres, faro conjeturas. Diretrizes Distribuir para cada aluno folhas com as atividades ou coloc-las no quadro-negro. Sugere-se dividir a classe em grupos de dois alunos, deixando-os trabalhar juntos. Aps completar a atividade I, os estudantes devem registrar seus resultados na tabela (atividade III). Certifique-se de que todos os estudantes tm os valores corretos, pois conjeturas sero feitas a partir dos dados da tabela. Poucos estudantes conseguiro completar a tabela para um cubo 10 10 10, a menos que algum padro tenha sido identificado (atividade IV). Pergunte: Existem constantes em uma coluna? Existem mltiplos?. Sugerir aos alunos que procurem fatores comuns vai ajud-los a reconhecer padres. Por exemplo, 0, 6, 24, 54 e 96 so as 5 primeiras entradas em uma das colunas. Um padro torna-se mais visvel se esses nmeros forem escritos como 0, 6 1, 6 4, 6 9 e 6 16. I. Responda s perguntas a seguir para cada um dos cubos das figuras 1, 2, 3 e 4. a) Quantos cubos pequenos h no cubo grande? Se esse cubo maior for jogado numa lata de tinta e totalmente submerso: b) Quantos cubos pequenos tero 3 faces pintadas? c) Quantos cubos pequenos tero 2 faces pintadas? d) Quantos cubos pequenos tero 1 face pintada? e) Quantos cubos pequenos tero 0 face pintada?
RPM OBMEP

f) Qual a soma de suas respostas em b), c), d) e e)?

II. Complete a figura 5, desenhando um cubo 6 6 6. Responda novamente s perguntas a), b), c), d), e), e f). III. Agora registre as informaes na tabela abaixo. Considerando, em cada caso, o lado dos cubos pequenos como unidade.
Comprimento do lado do cubo maior nmero de cubos pequenos com faces pintadas nmero total de cubos pequenos

2 3 4 5 6
RPM OBMEP

10

IV. Voc observa padres na tabela? Em caso afirmativo, complete a tabela para um cubo 7 7 7. Em caso negativo, desenhe o cubo e ento complete a tabela. Voc realmente pegou o jeito? Se voc acha que sim, complete a tabela para um cubo 10 10 10. Eis uma questo que pode ser usada para culminar essa atividade: Seja n o comprimento de um lado do cubo. Quando voc completar na tabela a linha correspondente a n, a soma dos valores dessa linha ser n3? Nota do tradutor Para completar a tabela para um cubo de lado n, pode-se considerar o cubo grande como sendo formado por n camadas horizontais. Cada camada um quadrado n x n . Nos esboos abaixo, o nmero em cada cubo pequeno indica quantas de suas faces ficam pintadas aps a imerso do cubo na lata de tinta:

faces pintadas camadas sup. e inf. camadas intermedirias total

1 2(n 2)2

2 8(n 2) 4(n 2) 12(n 2)

3 8

(n 2)3 (n 2)3

4(n 2)2 6(n 2)2

8
RPM OBMEP

Adaptado do artigo Atividades em sala de aula


Renate Watanabe, RPM 61

11

Um jogo aritmtico

Introduo Como seria bom se pudssemos fazer da Matemtica uma fonte de prazer ainda maior do que ela j . Isso possvel se tivermos como aliado um poderoso recurso ldico: o jogo. Proponho aqui um jogo aritmtico, que muito fcil de aprender, e pode ser jogado por duas ou mais pessoas. A idia sortear um nmero que, em seguida, deve ser obtido de outros, atravs das quatro operaes. Para representar os inteiros usamos as cartas de um baralho comum, com exceo dos coringas. O s (A), o valete (J), a dama (Q) e o rei (K) representam os nmeros 1, 11, 12 e 13 respectivamente. Formando nmeros Estamos acostumados ao clculo de expresses aritmticas, isto , dada uma expresso envolvendo nmeros e operaes matemticas, encontrar o nmero que lhe corresponde. Aqui se pede a soluo do problema recproco: dado um nmero, encontrar uma expresso aritmtica que corresponde a esse nmero. No jogo s permitido o uso das 4 operaes aritmticas bsicas (adio, subtrao, multiplicao e diviso) e de parnteses. Por exemplo, com os nmeros 2, 5, 7, 8 e 11, alguns dos nmeros que podemos formar so:

RPM OBMEP

12

19 = 11 + 8 33 = (5 2) 11 64 = (8 2) (5 + 11) 81 = 2 5 7 + 11 80 = ((5 2) 7 11) 8 100 = (7 + 2 + 11) 5 Note que: 1. No necessrio usar todos os inteiros disponveis; 2. O uso de parnteses no tem restries. Podemos tambm usar parnteses encaixados como na expresso do nmero 80; 3. S podemos usar cada inteiro disponvel uma nica vez; 4. No se pode formar nmeros por justaposio, isto , com o 5 e o 2 no podemos formar nem o 25 nem o 52. Na prtica, no precisamos escrever a expresso usando parnteses. Para formar o 80, declaramos: 5 menos 2 3; 3 vezes 7 21; 21 menos 11 10; 10 vezes 8 80. Para formar o 64, declaramos: 8 dividido por 2 4; 5 mais 11 16; 4 vezes 16 64. O que necessrio 1. Um baralho (descartam-se os coringas); 2. Cada jogador pode, se julgar necessrio, ter caneta ou lpis e uma ou mais folhas de papel. Incio do jogo Colocamos o baralho na mesa, com as cartas voltadas para baixo, num monte, de modo que no se possa ver que nmeros representam. Escolhe-se de comum acordo um participante para iniciar a rodada. Ento os itens 1, 2 e 3 a seguir devem ser repetidos at que haja um vencedor. O jogo 1. Escolhemos a carta de cima do monte e multiplicamos seu valor por 13: em seguida, somamos o resultado do produto ao valor de uma segunda carta retirada de cima do monte. Obteremos um nmero entre 14 a 182 (13 1 + 1 = 14 e 13 13 + 13 = 182). Esse o nmero que deve ser formado na rodada. As duas cartas tiradas vo para baixo do monte;

RPM OBMEP

13

2. O jogador da vez retira uma carta de cima do monte e a pe com o nmero para cima, no centro da mesa, ou ao lado da ltima carta retirada; 3. Ele ento faz suas anotaes e clculos, e ter duas opes: a) Formar o nmero sorteado ganhando a rodada (1 ponto). Nesse caso, o jogador da vez passa a ser aquele que est sua esquerda e colocam-se as cartas retiradas debaixo do monte. A partir daqui, precisa-se sortear um novo nmero, portanto retorna-se ao item 1 para o incio de outra rodada; b) Passar a vez ao jogador da sua esquerda. Em seguida d-se prosseguimento rodada retornando-se ao item 2. Quem vencer um total de 3 rodadas primeiro vence o jogo. Enquanto isso no ocorrer, repetem-se os itens 1, 2 e 3 sucessivamente. Um exemplo O primeiro jogador, A, tira a carta de cima do monte, digamos, 5 e a carta seguinte, uma dama. Ento, o nmero a ser formado na rodada ser 13 5 + 12 = 77. As duas cartas tiradas vo para baixo do monte. O segundo jogador, B, tira a carta de cima do monte, digamos 8 e a coloca aberta na mesa. No d para formar 77 com o nmero 8. Ele passa a vez para A (se o jogo s tiver dois jogadores), que tira, digamos, 6. Com 8 e 6 e as quatro operaes ainda no d para obter 77. O 6 fica aberto na mesa e A passa a vez para B que tira, digamos, um valete. Com 6, 8 e 11 no d para obter 77. a vez de A que tira, digamos, 3. A dois casos podem ocorrer: (1) A percebe que 6 11 + 8 + 3 = 77. Ento a rodada termina, A ganha 1 ponto, as cartas vo para baixo do monte e tudo comea de novo com B tirando as duas cartas de cima do monte para obter um novo nmero. (2) A no percebeu que podia obter 77 com as cartas da mesa e passa a vez para B. Se B obtiver o 77, ele que ganha um ponto e uma nova rodada se inicia. Se B no obtiver o 77, ele tira mais uma carta do monte e assim, sucessivamente, at que um dos jogadores conseguir formar o 77 com as cartas que esto abertas na mesa.

RPM OBMEP

14

Concluso Tenho jogado com amigos j h algum tempo. Estou convencido de que esse um jogo intelectualmente estimulante e muito agradvel. claro que existem muitos jogos com essas qualidades, mas esse tem a vantagem de ser matematicamente educativo. Alm disso, uma forma de viver a Matemtica, interagir com ela, senti-la, toc-la. Tambm estou certo de que podemos criar jogos matemticos que trabalhem a compreenso de teoremas e suas demonstraes, bem como suas aplicaes na resoluo de problemas..., mas esse j um outro assunto...
Adaptado do artigo Um jogo aritmtico Eric Campos Bastos Guedes, RPM 55.

____________________________________
Respostas das Atividades Primeiro grupo I
1. 10 (6 + 3 + 1) + 5 2. 11 + 1 9 (8 8) 3. 11 [(20 15) 3 10] 4. (11 + 3) 12 + 18 12 5. (4 + 16) 10 (25 24) 6. 17 17 + (14 13) 7 7. (9 9) + ( 5 4) + 2 8. (7 6) + [3 (10 5)] 9. 5 [(11 + 21) 8] + 6 10. (17 1) [(6 2) 2]

II
2. 9 {14 [(19 + 3) 11]} 3. {11 [(19 + 9) 14]} 3 4. 19 9 + 11 14 3 5. 9 {14 [(11 + 19) 3]} 7. 14 {[(3 + 19) 9] 11} 8. 9 [(11 3) (14 + 19)] 9. 11 3 (14 + 19) + 9 10. 19 9 + 11 14 + 3

III
2. 1 = 5 [( 11 + 3) 7 + 2] 3. 2 = [(5 + 3) (11 7)] 2 4. 4 = [(5 + 3) (11 7)] 2 5. 2 3 5 7 11 = 2310 6. 11 7 5 (3 + 2) = 1925 7. 0 = [(11 + 3) 2 7] 5 8. [(11 3) (5 + 7)] 2 = 48 9. 5 {3 [(11 2) 7]} = 4 10. 11 7 5 3 2 = 1153
RPM OBMEP

15

Seis problemas no triviais equivalentes

O poder da Matemtica de relacionar o que aparentemente no tem relao.

Neste artigo dois problemas sero chamados equivalentes se sua resoluo fizer uso do mesmo tipo de Matemtica. Problemas equivalentes evidenciam talvez a qualidade mais importante da Matemtica: a possibilidade de um conceito terico ser usado como modelo para muitas idias diferentes. fcil produzir exemplos. Se o conceito terico for combinaes, como em Probabilidade, essa ideia tambm pode ser usada para determinar as leis de Mendel em Biologia, para calcular coeficientes binomiais, para calcular certas probabilidades em jogos de baralho, para achar o nmero de polgonos de vrios tipos que tenham pontos arbitrrios, como vrtices, e assim por diante, quase que indefinidamente. Mas difcil produzir bons exemplos quando se desejam problemas equivalentes em uma escala muito menor, onde mesmo tipo de Matemtica no significa Matemtica de um mesmo campo, ou de um mesmo tpico dentro de um campo ou assunto que usem as mesmas ideias. Especificamente tentei encontrar problemas satisfazendo as seguintes condies: 1) Os problemas deveriam ser matematicamente idnticos at nos nmeros usados na sua resoluo.

RPM OBMEP

16

2) At que uma resoluo fosse examinada, nada no problema deveria indicar que o mesmo tipo de Matemtica pudesse ser usado. Assim os problemas deveriam, na medida do possvel, vir de tpicos totalmente desvinculados dentro da Matemtica ou dentro de aplicaes da Matemtica. 3) Os problemas deveriam estar no mbito da Matemtica do ensino fundamental ou ensino mdio, quanto mais simples, melhor. Problemas 1. Expresse 1 como soma de duas fraes de numerador 1 (fraes do 2 1 , n um inteiro positivo). tipo n 2. Ache todos os retngulos cujos lados tenham por medida nmeros inteiros e que tenham rea e permetro numericamente iguais. 3. Quais pares de inteiros positivos tm mdia harmnica igual a 4? 4. Ache os possveis pares de inteiros cujo produto seja positivo e igual ao dobro de sua soma. 5. Dado um ponto P, ache todos os n tais que o espao em torno de P possa ser coberto, sem superposio, por polgonos regulares, congruentes, de n lados. 6. Para quais inteiros positivos n > 2, o nmero 2n divisvel por n 2? Para mostrar a equivalncia, verificaremos que os seis problemas se reduzem resoluo de uma equao que a caracterizao do primeiro. Reduo dos problemas a uma equao Problema 1

adiante.)

RPM OBMEP

1 1 1 Se 1 for a soma de duas fraes de numerador 1 ento = + , 2 p q 2 onde p e q so inteiros positivos. (A equao ser resolvida mais

17

Problema 2 Sejam a e b o comprimento e a largura do retngulo procurado. Como a rea e o permetro so numericamente iguais, temos: 2a+ 2b = ab 2(a + b) = ab
a+b 1 = ab 2 1 1 1 + = . a b 2

Como a e b devem ser inteiros e positivos essa ltima equao tem a mesma forma que a equao do Problema 1. Problema 3 A mdia harmnica de dois nmeros x e y

2xy . x+ y

Sejam x e y inteiros positivos. Das condies dadas:


2 xy =4 x+ y xy =2 x+ y x+ y 1 = . xy 2

A ltima equao tem a mesma forma que a equao na 3a linha do Problema 2 e assim se reduz equao do Problema 1. Problema 4 Sejam x e y dois inteiros, z o seu produto, z > 0. Os nmeros x e y devem ser positivos pois a sua soma e produto so positivos. Das
RPM OBMEP

condies dadas obtm-se xy = z e x + y = implicam: x + y =

z . As condies juntas 2

xy 2

x+ y 1 = . xy 2

18

Essa ltima equao idntica equao da ltima linha do Problema 3 e assim reduz-se equao do Problema 1. Problema 5 Este o problema mais difcil de caracterizar. Seja k o nmero de polgonos com vrtice em P. Se os polgonos no se sobrepuserem, forem regulares e congruentes, utilizando a notao da figura abaixo, obter-se-: 1 = 2 = ... = k =
360 em graus. k

Mas os ai so medidas de ngulos de polgonos regulares de n lados, portanto

i =
Temos ento:

(n 2)180 n

1 < i < k.

360 (n 2)180 = k n 2 (n 2) = k n
2n = (n 2)k 2n + 2k = nk. Das condies do problema segue-se que n e k devem ser inteiros positivos e portanto essa equao tem a mesma forma que a da primeira linha do Problema 2.
RPM OBMEP

Problema 6 Se 2n divisvel por n 2 ento 2n = (n 2)k, onde k um numero inteiro. Essa equao idntica a uma das equaes do Problema 5 e portanto se reduz do Problema 1.

19

Equao diofantina Assim, os seis problemas podem ser resolvidos considerando-se a equao do Problema 1. Devido s condies, essa equao uma equao diofantina e sua soluo interessante. 1. Seja

1 1 1 onde p e q so inteiros positivos. = + 2 p q 1 1 1 1 e (pois a soma no chegaria a > > 4 p 4 q

2. impossvel termos

1 1 ser 1 ) e assim pelo menos uma das fraes ou deve ser maior p q 2

do que ou igual a

1 1 1 . Suponham os . p 4 4

3. Ento p = 1, 2, 3 ou 4. 4. p = 1

1 1 = 1 + q = 2 o que no possvel pois q posi2 q 1 1 1 1 = + = 0 , que no tem soluo; 2 2 q q


p = 3 q = 6; p = 4 q = 4.

tivo; p = 2

5. Por causa da simetria de p e q na equao original, obtemos resultados correspondentes se

1 1 . q 4

6. Portanto temos 3 solues: (p, q) = (3, 6); (p, q) = (4, 4); (p, q) = (6, 3). Solues
RPM OBMEP

Todos os problemas esto agora resolvidos. Problema 1 A resposta

1 1 1 1 1 1 1 = + = + = + . 2 3 6 4 4 6 3

20

Problema 2 Existem dois retngulos satisfazendo as condies dadas: um 4 4 e o outro, 3 6. Problema 3 Duas respostas: 4 e 4 ou 3 e 6 so pares de inteiros cuja mdia harmnica 4. Problema 4 Os pares so idnticos ao do problema 3. Problema 5 Os nicos polgonos regulares congruentes que, sem superposio, cobrem o espao em torno de P (e assim cobrem o plano) so os polgonos de 3 lados (seis tringulos eqilteros em torno de P), os de 4 lados (quatro quadrados em torno de P) e os de 6 lados (trs hexgonos regulares em torno de P), como se v na figura.

Seis tringulos, quatro quadrados, trs hexgonos.

Problema 6 A resposta : n 2 um divisor de 2n quando n = 3, n = 4 ou n = 6. (A condio n > 2 no problema original garante ser n 2 positivo. Sem essa condio existiriam as solues n = 1, n = 0 ou n = 2). Resumo Os seis problemas formam um grupo de problemas, no triviais equivalentes que podem ser usados em classes de ensino fundamental e mdio. fcil desenvolver outros grupos de problemas mais apropriados para o uso em lgebra Elementar ou Geometria. Tais grupos de problemas podem ser usados para demonstrar o poder de um pouco de Matemtica abstrata na resoluo de exerccios que, primeira vista, pareciam no relacionados.
RPM OBMEP

Adaptado do artigo Seis problemas no triviais equivalentes Zalman Usiskin, RPM 04.

21

O menino

No havia sada. Teria que esperar por trs horas o prximo vo para Salvador. Arquiteto por formao e profisso, tinha que apresentar um projeto na manh seguinte, numa cidade prxima capital da Bahia. Assentei-me como pude. Teria que olhar para aquele relgio pendurado no teto por trs horas. Como se no bastasse, o relgio registrava os segundos. Relgios que registram segundos demoram mais que os que no o fazem. Alguns apelam para palavras cruzadas, outros giram os polegares e eu, como o vcio do cachimbo entorta a boca, trao em folhas de papel as formas que se me apresentam no ambiente que alcanado pelas retinas. Lpis e papel na mo, registrava dois lances de escada e uma escada rolante que surgiram a minha frente. Mal traara as primeiras linhas, deparei-me com uma questo que me intrigou: quantos degraus deveria desenhar na escada rolante? Em vo, tentei contar os degraus visveis. Se a escada parasse, poderia cont-los. Tive mpetos de apertar o boto vermelho prximo ao corrimo, onde se lia PARAR. Meu censurador no permitiu que o fizesse. Fiquei ali, inerte, com o cachimbo na mo e sem poder fumar.

RPM OBMEP

22

Um menino sentou-se ao meu lado, brincando com uma bola. Sem tirar os olhos da bola, ela disse em voz clara e pausada: Pepino no parece inreal? Olhei-o, ligeiramente, com o canto dos olhos e, sem nada dizer, retornei ao meu cachimbo apagado. Alguns instantes depois, senti minha camisa ser puxada e escutei novamente: Pepino no parece inreal? Dessa vez, com uma mo segurando a bola e com a outra puxando a minha camisa, ele me olhava firmemente. No inreal, irreal. Pois , no parece? Aquela insistncia irritou-me. Eu, diante do mais intrincado problema da existncia humana quantos degraus ficam visveis quando a escada rolante pra e aquele menino me questionando sobre a realidade de um pepino! Tentando dissuadi-lo, resolvi apresentar-lhe a complexidade do problema que me afligia. Olha, menino, estou tentando desenhar aquelas escadas e no sei como acabar o desenho da escada rolante. Quantos degraus devo desenhar? Meu desenho est parado e a escada est subindo. Se a escada parasse de repente, quantos degraus ficariam visveis? Sem nada dizer, colocou a bola sobre a cadeira, subiu e desceu a escada (que sobe). Apontando para o relgio, disse: Eu deso a escada duas vezes mais rpido do que subo. E repetiu sua viagem ao vo da escada, mostrando-me que, no mesmo tempo em que dava um passo para subir, dava dois para descer. Novamente sem nada dizer, comeou a subir a escada rolante, contando os passos: um, dois, trs, ..., num total de vinte passos. Do alto da escada, olhou-me como quem estivesse fazendo a mais bvia das coisas, e comeou a descer a mesma escada rolante, contando os passos: um, dois, trs, ..., num total de trinta e cinco passos. Em seguida tomou o lpis e o papel de minhas mos e completou, com traos infantis, o meu desenho.

RPM OBMEP

23

Nenhum censurador poderia me conter. Levantei-me bruscamente e apertei o boto vermelho. Ansioso, comecei a contar os degraus. Para meu espanto, correspondia ao desenho do menino. Com a maior seriedade que j tive em minha vida voltei-me para o menino e perguntei-lhe: Por que o pepino parece inreal? Quantos degraus o menino desenhou? Vamos resposta: Vamos tomar como unidade de tempo o tempo no qual o menino d um passo subindo a escada. Seja n o nmero de degraus da escada rolante que desaparecem (ou surgem) na unidade de tempo. Como o menino deu 20 passos para chegar ao topo da escada, ele demorou 20 unidades de tempo. Isso significa que desapareceram 20n degraus. Chamando de N o nmero de degraus visveis, temos: N = 20 + 20n ou n =
N 20 . 20

(1)

O menino deu 35 passos para descer a escada rolante (que sobe). Lembremos que a frequncia de seus passos duas vezes maior na descida que na subida. Ou seja, o tempo de dar dois passos descendo igual ao de um passo subindo. Cada passo na descida demora unidade de tempo. Ele demorou
1 da 2

35 unidades de tempo para descer a escada. Isso 2

significa que surgiram 35n degraus novos. Assim, 2

N = 35
RPM OBMEP

35n 2

ou n =

70 2 N . 35

(2)

Igualando (1) e (2):

N 20 70 2 N = 20 35

24

35N 700 = 1400 40N ou 75N = 2100, de onde

2100 = 28 75 O menino desenhou 28 degraus. N=


Adaptado do artigo O menino Ledo Vaccaro Machado, RPM 42.

Desafio
Distribuir os nmeros de 1 a 9 dentro dos pontos brancos (de interseco), sem repetir, de forma que a soma dos nmeros pertencentes circunferncia externa seja exatamente igual soma dos nmeros pertencentes a cada uma das circunferncias internas.

Resposta na pgina 171.

RPM OBMEP

25

O problema dos cinco discos: sorte ou sabedoria?

Neste artigo queremos mostrar uma curiosidade sobre o antigo problema dos cinco discos. A mais bela apresentao desse problema encontra-se em O homem que calculava (Tahan, Malba 32a edio. Record, Rio de Janeiro, 1986). Nele contada uma lenda onde trs prncipes muito sbios e conhecedores da Matemtica que pretendiam casar-se com a princesa Dahiz, filha do rei Cassim. A prova dos cinco discos foi proposta por um grande sbio da corte para decidir qual dos trs pretendentes era o mais inteligente. Foram mostrados aos prncipes cinco discos, sendo dois pretos e trs brancos, todos de mesmo peso e tamanho. Em seguida vendaram-lhe os olhos e, ao acaso, foi pendurado um desses discos s costas de cada um dos trs. Disse o rei: Cada um de vs ser interrogado particularmente e aquele que descobrir a cor do disco que lhe coube por sorte, ser declarado o vencedor. O primeiro a ser interrogado poder ver os discos dos outros dois, ao segundo ser permitido ver o disco do terceiro, e o terceiro ter que formular a resposta sem ver nada. Aquele que der a resposta certa ter que justific-la.

RPM OBMEP

26

Aconteceu ento que o prncipe Camoz quis ser o primeiro. Viu os dois discos dos seus adversrios e errou. Em seguida, sabendo que Camoz havia errado, o prncipe Benefir se prontificou em ser o segundo, mas tambm errou. Aradim, o terceiro prncipe, acertou com absoluta segurana. Qual foi a resposta do prncipe Aradim e como ele descobriu? Esse o problema dos cinco discos. Malba Tahan d uma inteligente soluo desse problema, onde conclui tambm que Aradim foi considerado o mais inteligente entre os trs prncipes. Eis a soluo de Malba Tahan: o prncipe Aradim afirmou que o seu disco era branco e justificou da seguinte maneira: Se Camoz (o primeiro a falar) tivesse visto dois discos pretos, ele obviamente teria acertado. Como ele errou, conclui-se que viu dois discos brancos, ou um preto e um branco. Na hiptese de Benefir ter visto em minhas costas um disco preto, ele (usando o mesmo raciocnio que fiz com relao a Camoz) teria acertado. Logo, ele s pode ter visto um disco branco e, portanto, o meu disco branco. A curiosidade que pretendemos apresentar que, sob o ponto de vista matemtico e levando em conta somente o acerto da cor do disco, a chance de erro dos dois anteriores era bem pequena, o que torna discutvel a concluso de que Aradim fosse mais inteligente que Camoz ou Benefir. Com efeito, vamos calcular as probabilidades de acerto da cor do disco de cada um dos trs prncipes, levando em conta que todos eles so sbios. As possveis distribuies dos discos Sejam b = (disco branco) e p = (disco preto). Por simplicidade escrevemos A = Aradim, B =Benefir e C = Camoz. Ento a ordem em que os prncipes se apresentaram para serem interrogados pode ser representada por uma terna ordenada (C, B, A). A ttulo de exemplo, perguntamos quantas maneiras diferentes podem C possuir disco branco, B possuir disco preto e A possuir disco preto? Isto , de ocorrer (b, p, p).

RPM OBMEP

27

Sabemos que existem trs discos brancos b1, b2 e b3 e dois discos pretos p1 e p2. Por uma simples contagem, obtemos seis maneiras diferentes de ocorrer (b, p, p), a saber: (b1, p1, p2), (b1, p2, p1), (b2, p1, p2), (b2, p2, p1), (b3, p1, p2) e (b3, p2, p1). claro que o nmero total de maneiras em que podem ser distribudos os discos aos prncipes A5, 3 = 60. Descrevendo esses casos, obtemos: Eventos E1 = (b, b, b) E2 = (p, b, b) E3 = (b, p, b) E4 = (b, b, p) E5 = (p, p, b) E6 = (p, b, p) E7 = (b, p, p) Lembretes a) Se os conjuntos unitrios de um espao amostral finito U tm todos a mesma probabilidade, ento a probabilidade de um evento A qualquer de U ser dada por: Frequncia 6 12 12 12 6 6 6

P ( A) =

n( A) n(U )

onde n(A) o nmero de elementos do evento A e n(U) o nmero total de elementos do espao amostral U. b) Nas mesmas condies de a), se A1, A2, ..., An so eventos disjuntos entre si,

P ( A1 A2 ... An ) =
RPM OBMEP

n( A1 ) + n( A2 ) + ... + n( An ) . n(U )

Como o problema afirma que a escolha dos discos feita ao acaso, segue-se que o espao amostral associado ao problema satisfaz as condies necessrias para a validade de a) e b). No difcil verificar tambm que o acerto da cor do disco admite uma estratgia que maximiza

28

a probabilidade de vitria de cada concorrente e garante, com probabilidade 1, a existncia de um vencedor, que certamente ser nico uma vez que o processo terminaria no momento em que um dos concorrentes acertasse a cor do seu disco. Como os concorrentes supostamente so sbios, razovel admitir que eles seguiro a melhor estratgia em cada situao e portanto teremos P(C) + P(B) + P(A) = 1 onde P(C), P(B) e P(A) so, respectivamente, as probabilidades de vitria de C, B e A. A estratgia tima e a correspondente probabilidade de vitria de C. Se C vir dois discos pretos nos seus adversrios, saber que restam trs discos brancos. Responder ento com absoluta segurana que possui um disco branco. Assim o evento E7 lhe favorvel. Caso C veja dois discos brancos, saber que restam dois discos pretos e um disco branco. Logo responder possuir disco preto, contando com a probabilidade 2/3 de acertar. Consequentemente, o evento E2 lhe favorvel e o evento E1 lhe desfavorvel. Suponhamos agora que C tenha visto um disco branco e um disco preto em seus concorrentes. Concluir que restam dois discos brancos e um disco preto. Logo, dever responder que possui um disco branco, contando com a probabilidade 2/3 de acertar. Segue que os eventos E3 e E4 lhe so favorveis e o evento E6 lhe desfavorvel. Em resumo, usando essa estratgia, C ir acertar na hiptese de ter ocorrido qualquer um dos eventos disjuntos E2, E3, E4 ou E7 e ir errar se houver E1, E5 ou E6. Segue-se, ento, que:

P (C ) =

n( E2 ) + n( E3 ) + n( E4 ) + n( E7 ) 6 + 12 + 12 + 12 7 . = = n(U ) 60 10

RPM OBMEP

Isso mostra que a probabilidade vitria do prncipe Camoz, o primeiro candidato, de 70%, contando com a sua sabedoria, restando assim apenas 30% de probabilidade para que os outros dois prncipes tivessem chance de serem apenas interrogados.

29

Considerando ainda que Aradim s seria interrogado caso Benefir (o segundo interrogado) tambm errasse, pode-se mostrar que ele o que teria a menor chance de ser escolhido como noivo de Dahiz. No entanto, Aradim possuidor de muita sorte, pois os dois primeiros concorrentes erraram. Para completar, a probabilidade de Benefir acertar de 20% e a probabilidade do prncipe Aradim acertar de apenas 10%. A reabilitao de Aradim Esse clculo, entretanto, diz respeito s ao fato de acertar, ou no, a cor do seu disco. Acontece que o rei dissera que os prncipes deveriam, tambm, justificar a resposta correta. Fica a pergunta do que o rei entendia por justificar. Seria aceitvel, em caso de dvida, uma adivinhao educada, isto , uma opo pela alternativa mais provvel? Ou seria necessria uma explicao lgica de como se chegou nica alternativa correta possvel? Neste caso, quais seriam as probabilidades de vitria de cada um dos trs concorrentes?
Adaptado do artigo O problema dos cinco discos: Sorte ou Sabedoria? Ma-To Fu e Roberto Elias, RPM 11.

RPM OBMEP

30

Calculadora padro: um problema interessante

Suponhamos que voc tem uma calculdadora de bolso, padro, de 8 dgitos, que efetua as quatro operaes, +, , , e extrai razes quadradas. Ser possvel, usando essa calculadora, extrair a raiz n-sima de um nmero qualquer? Na verdade, dado um nmero real x, no negativo, usando , +, , , possvel achar xp/q, onde p e q so nmeros naturais. Vamos explicar como isso possvel mostrando alguns exemplos. somente as teclas Exemplo 1: Calcule Seja x =
3 3

5.

5 . Ento x3 = 5. Multiplicando por x os dois

lados da igualdade, obtemos x4 = 5x ou x = 4 5 x . Inicialmente criamos uma sequncia de nmeros reais sendo x1 uma aproximao de
3

5 (por exemplo, x1 = 1)

e xn +1 = 4 5 xn , n = 1, 2, 3, . Vamos mostrar os valores de alguns termos da sequncia obtidos na calculadora e, em seguida vamos dar uma justificativa do por que a sequncia converge para
3

5.

Tomemos x1 = 1. Ento, x2 = 4 5 , x3 = 4 5 4 5 , e assim por diante.

RPM OBMEP

31

Vejamos os valores calculados sendo que a notao [*] significa que apertamos a tecla *: [5] [ ][ ] 1.4953487 Com o valor anterior mantido na tela, fazemos [] [5] [=] [ ][ ] 1.653591 ][ ] obtemos Repetindo sempre os comandos [] [5] [=] [ 1.6957019 1.7063962 1.7090802 1.7097519 1.7099199 1.7099619 1.7099724 1.7099750 1.7099757 1.7099758 1.7099759 1.7099759, o que indica que uma aproximao para 1,7099759.
3

5 com erro menor que 107

Uma idia do por que funciona: consideremos a expresso


4 4 4 4 5 5 5 5 4 4 4 4 5 5 5 5

com infinitos radicais. Podemos escrever = 5 4 516 5 64 5 256


1 1 1 1

= 5 4 16 64 256
1

1+ 1 + 1 + 1

1 1 1 1 Sabemos que 4 + 16 + 64 + 256 +

1 = 4 = , pela frmula da 1 3 1 4

soma dos termos de uma PG infinita, logo, temos


RPM OBMEP

4 4 4 4 5 5 5 5

= 51/3 =

5 = x. Ou seja, admitindo que existe o limite


3

de xn quando n tende ao infinito, ento, esse limite ser

5.

32

Um outro modo de verificar o valor x (admitindo que esse limite exista) :


x = 4 54 54 54 5 , logo,

do limite da sequncia

x 4 = 54 54 54 5
3

= 5 x. Como x

diferente de zero, a igualdade implica x = Exemplo 2: Calcule 5 5 .

5.

Seja x = 5 5 . Ento, x5 = 5 e vemos que x8 = x5x3 = 5x3 que implica


8 x = 5 x3 . Vamos construir uma sequncia x1, x2, x3, ..., xn, ..., com

x1=1 e os outros termos como na tabela, usando a calculadora para obter os nmeros aps as flechas. x2 = 8 5 x13 = 8 5 = 58 =
1+ 3 1

5 1.2228445 5 125 1.3186680


1+ 3 + 9

8 8 x3 = 8 5 x23 = 5 53 = 58 64 =

x4 = 8 5 x33 == x5 = 5 x6 = 5
1 + 3 + 9 + 27 8 82 83 84

125

59

= 58 64 864 1.3565069

1 + 3 + 9 + 27 + 81 8 82 83 84 85

Aqui, a cada passo, utilizamos apenas as teclas [], [5], [=] e [

].

Novamente aceitando que a sequncia escolhida converge para um limite x diferente de zero, podemos fazer:
RPM OBMEP

1 3 9 27 + + + + 2 8 8 83 84
1 3 9 27 + + + + 8 82 83 84

1 = 8 = , logo, 3 5 1 8
1 = 55

x=5

= 5 5.

33

Ou, como no exemplo anterior, elevando x a oitava potncia, obtemos x = 5x3, e como x diferente de zero vem que x = 5 5 .
8

A argumentao dos exemplos apresentados foi baseada no fato de converge. Isso realmente que a sucesso x1 , x2 , x3 , , x n , x n +1 , acontece? Vamos responder a essa pergunta no caso do ltimo exemplo. Os grficos das funes y = x8 e y = 5x3 esto mostrados na figura abaixo e como o nmero procurado, x = 5 5 , satisfaz x8 = 5x3, vemos que x ser a bscissa no nula do ponto de interseco dos dois grficos.
y=x 8
20 15 10 5 -2 -1 1 -5 2

y=5x 3

Partindo, por exemplo, de x1=1, examinemos a sucesso de pontos, representados no grfico:

P = (1, 0); P2 = (1, 5); P3 = ( 8 5 , 5); P4 = ( 8 5 , 5 53 ); 1 P5 = ( 5 53 , 5 53 ); P6 = ( 5 53 , 5 53 59 );


Observe que os pontos P2, P4, P6, ... pertencem ao grfico de y = 5x3 e que os pontos P3, P5, P7, ... pertencem ao grfico de y = x8. A sucesso de todos os pontos converge para o ponto de interseco dos dois grficos que o ponto P = ( 5 5 , 5 58 ) , o que mostra que a
RPM OBMEP

8 8

8 8

sucesso das abscissas x1, x2, x3, ..., xn, ..., converge para x = 5 5 . Isso mostrado rigorosamente usando-se tcnicas de Anlise Real. Quanto a escolha x1=1, ela foi feita simplesmente para facilitar os

34

clculos e tambm por sabermos que 5 5 est prximo de 1. Se tivssemos escolhido qualquer outro valor, por exemplo, x1=100, o limite da sucesso x1, x2, x3, ..., xn, ... continuaria satisfazendo x8 = 5x3.
y = x 8 e y = 5x 3 esto mostrados na figura a seguir e como o

nmero procurado, x = 5 5 , satisfaz x8 = 5 x 3 , vemos que x ser exatamente a abscissa, com x 0 , do ponto de interseco dos grficos de y = x 8 e y = 5x 3 . Ora, o fato algbrico de que a raiz x 8 = 5x 3 existe, equivalente ao fato de que os grficos se cortam. Para 0 x 5 5 1,379, o grfico de
y = x 8 estar acima do grfico de y = 5x 3 .

Partindo de um valor arbitrrio, por exemplo, x1 = 1 , examinemos a seguinte sucesso de pontos do plano cartesiano, representados no grfico
8 3 acima: P = (1, 0); P2 = (1, 5); P3 = (8 5 , 5); P4 = (8 5 , 5 5 ); 1

P5 = ( 8 5 8 5 3 , 5 8 5 3 ); P6 = ( 8 5 8 5 3 , 5 8 5 3 8 5 9 );
Observe que os pontos P2 , P4 , P6 , e que os pontos P3 , P5 , P7 , esto sobre o grfico de y = 5x 3

8 esto sobre o grfico de y = x .

Os pontos P1 , P2 , P3 , , Pn , convergem para o ponto P = (51 5 , 58 5 ) , que exatamente o ponto de interseco dos dois grficos. ento bvio que a sucesso x1 , x2 , x3 , , x n , x n +1 , converge exatamente para 5 5 . Pode-se mostrar, rigorosamente, usando as tcnicas de Anlise Real, que isso realmente acontece. Quanto a escolha, x1 = 1, ela foi feita simplesmente para facilitar os clculos e tambm por sabermos que 5 5 est prximo de 1. Se tivssemos sucesso, que estamos supondo existir, continuaria satisfazendo x8 = 5x3 .
RPM OBMEP

escolhido qualquer outro valor para x1 , por exemplo, x1 = 100, o limite da

35

Voc certamente j percebeu o procedimento geral: se quisermos calcular x = n a ou seja, achar x tal que xn = a, devemos transformar essa igualdade de modo a obter, do lado esquerdo, um expoente que seja uma potncia de 2. Isso feito multiplicando-se os dois membros por uma potncia conveniente de x. Assim, por exemplo, se x = 11 a ento x11 = a, que implica x16 = x11x5 = a x5 ou x = 16 ax 5 e utilizaremos a
5 16 sucesso x1, x2, x3, ..., xn, ..., na qual xn +1 = axn .

Adaptado dos artigos Vamos usar a calculadora? Hideo Kamayama e Eduardo Wagner, RPM 26. Vamos continuar usando a calculadora Joo Bosco Pitombeira de Carvalho, RPM 51.

RPM OBMEP

36

Uma equao interessante

H algum tempo, um amigo professor mostrou-me a equao


3

2x 1 + 3 x 1 = 1

e fez a seguinte observao: apesar de, no decorrer da resoluo, elevarmos as equaes somente a potncias mpares (duas elevaes ao cubo), ainda assim, surpreendentemente, aparece uma raiz falsa. Por qu? Antes de mostrar como o professor resolveu a equao, vejamos o porqu da sua surpresa. Sabemos que x = y xn = yn, x, y R, n N, mas a recproca desta afirmao s verdadeira se n for mpar. Isto , xn = yn x = y, x, y R, se n for mpar. fcil ver que a propriedade xn = yn x = y no vale se n for par basta observar que 52=(5)2 e 5 5. O que vale : xn = yn | x | = | y |, x, y R, n N.
RPM OBMEP

As falsas solues aparecem nitidamente quando, resolvemos equaes irracionais. Vejamos um exemplo: Quais so as solues da equao

2x 3 = x 3 ?

37

2 x 3 = x 3 ( 2 x 3 ) 2 = ( x 3) 2 2 x 3 = = x 2 6 x + 9 x 2 8 x + 12 = 0 x = 6 ou x = 2.
3 4

As passagens 2, 3 e 4 so equivalncias, mas a recproca da implicao 1 no verdadeira. por isso que, aps resolvermos a equao, testamos as razes encontradas, para ver se elas, de fato, satisfazem a equao inicial. No exemplo, 6 raiz de (2), mas 2 no . Portanto, estamos acostumados com o aparecimento de falsas razes na resoluo de equaes irracionais. Mas, no exemplo que o professor apresentou, o fato de aparecer uma raiz falsa era estranho, pois a resoluo da equao exigia apenas que seus membros fossem elevados ao cubo e sabemos que, em R, x3 = y3 x = y. Vejamos como o professor resolveu a equao:
3

2x 1 + 3 x 1 = 1 .

(1)

Elevando ao cubo, obtemos

2 x 1 + 3( 3 2 x 1) 2 .3 x 1 + 3 3 2 x 1.( 3 x 1) 2 + x 1 = 1 (2)
3 x 2 + 3 3 2 x 1.3 x 1( 3 2 x 1 + 3 x 1) = 1 o termo entre parnteses vale 1 ( a prpria equao (1)!) (3)

3 x 2 + 3 3 (2 x 1)( x 1) = 1 3 x + 3 3 (2 x 1)( x 1) = 3
3

(4) (5) (6) (7)


3

2 x 2 3x + 1 = 1 x
2

2x2 3x + 1 = (1 x)3 2x 3x + 1 = 1 3x + 3x x
2

(8) (9)

RPM OBMEP

x x = 0.
3 2

E, portanto, x = 0 ou x = 1. Verifica-se, por substituio em (1), que 1 soluo, mas 0 no .

38

Onde e por que apareceu essa falsa raiz? Sugiro que o leitor tente responder a essa pergunta antes de prosseguir. Observe que x = 0 no soluo das equaes (1), (2) e (3), mas soluo das equaes a partir de (4). Na verdade, (1), (2) e (3) so equivalentes entre si (possuem o mesmo conjunto soluo), e as equaes de (4) a (9) tambm so equivalentes entre si, mas (3) e (4) no so equivalentes. Foi nessa passagem que fizemos algo ilcito. O que fizemos para passar de (3) a (4)? Ora, usamos novamente a equao (1) substituindo 3 2 x 1 + 3 x 1 por 1, e esse procedimento no gera uma equao equivalente anterior. Tendo duas equaes equivalentes, (1) e (3), se substituirmos uma na outra, obtemos uma nova equao que consequncia das anteriores, mas no , necessariamente, equivalente a elas. Assim (3) (4), mas no vale a recproca. Vejamos um exemplo onde esse fato mais evidente: x = 2 (o conjunto soluo {2}), 2 = x ( equivalente a de cima). Substituindo uma na outra, obtemos x = x, cujo conjunto soluo R! Assim, o aparecimento de uma raiz falsa no est ligado ao fato de a equao ser irracional nem s potncias que tomamos, e sim, ao procedimento da resoluo. Mais uma palavra sobre esse fato: o truque utilizado na passagem de (3) para (4) til, pois limpou a equao, mas no uma equivalncia no podemos perder de vista a equao original. Situaes como essa so comuns, por exemplo, na trigonometria quando usamos, numa equao, a identidade sen2 x + cos2 x = 1.
RPM OBMEP

Vamos ilustrar o aparecimento de falsas razes atravs de mais dois exemplos: x = 1 x (e, portanto, x = 1/2). Se elevarmos ambos os membros ao cubo, teremos:

39

x = l x x3 = (l x)3 x3 = 1 3x + 3x2 x3 (substituindo x por 1 x) x3 = 1 3(1 x) + 3x2 x3 2x3 3x2 3x + 2 = 0 x = 1/2; x = 1; x = 2. Outro exemplo: x = 1. x = 1 (x l)2 = 0 x2 2x + 1 = 0 (substituindo x por 1) x2 2.1 + 1 = 0 x2 = 1 x = l ou x = l.
Adaptado do artigo Uma equao interessante Cludio Possani, RPM 19.

RPM OBMEP

40

PAINIS

Painel I O nmero 12
Ao nmero 12 so atribudos muitos significados, sobretudo de ndole religiosa ou espiritual, cuja influncia provocou alguns efeitos na organizao de nosso cotidiano. Na historiografia judaico-crist, temos os 12 filhos de Jacob, filho de Isaac e neto de Abrao, dos quais derivaram as 12 tribos de Israel. Refere-se ainda que Jacob usava um peitoral sobre o qual haviam sido incrustadas 12 pedras preciosas que so a revelao de 12 poderes csmicos. Tambm a coroa usada na sagrao da monarquia inglesa tem 12 pedras preciosas. So 12 os deuses principais da mitologia grega, que vivem no Monte Olimpo. O ano tem 12 meses. O zodaco divide a esfera celeste em 12 casas. O relgio est dividido em 12 horas. A bandeira da Unio Europia tem 12 estrelas douradas, que, segundo a Comisso Europia, representam a solidariedade e harmonia entre os povos da Europa, porque o nmero 12 tradicionalmente um smbolo de perfeio, de plenitude e de unio. No consigo acreditar que os nmeros tenham algum significado que os transcenda, porm creio que a natureza abstrata dos nmeros propicia a sua utilizao como representantes de significados que os transcendem. Atribuies e interpretaes do significado do nmero 12, sobretudo relacionadas com questes religiosas e sociais, decorrem do fato, do domnio aritmtico, de 12 ser o produto de 3 por 4 e, alm disso, parece-me que o fato de 12 ter muitos divisores, 1, 2, 3, 4, 6 e 12, pode ter ajudado na sua

RPM OBMEP

41

projeo. que, entre as quatro operaes aritmticas elementares efetuadas no conjunto dos nmeros inteiros, a diviso a nica que nem sempre d resultados inteiros, sendo uma minoria os casos em que isso acontece. Conjugando esse fato com a circunstncia de que a maioria dos seres humanos se mostra mais disponvel para o clculo com inteiros do que com outros tipos de nmeros, compreende-se a importncia dada aos nmeros inteiros que se evidenciam por terem muitos divisores como 12, 24 e 60, por exemplo. Mas o objetivo deste artigo mostar uma interveno do nmero 12 numa relao entre os domnios algbrico e geomtrico. Consideremos uma funo quadrtica f(x) = ax2 + bx + c com a > 0 e = b2 4ac > 0, condio essa, como bem sabemos, que implica a existncia de duas razes reais distintas da equao ax2 + bx + c = 0. Essas razes so as abcissas dos pontos em que a parbola, grfico de f, intersecta o eixo x. Consideramos o tringulo formado pelos pontos A, B e pelo ponto V, vrtice da parbola, como na figura. Esse tringulo issceles, j que AV = VB. Vamos verificar que o muito falado nmero 12 relaciona a funo quadrtica com a possibilidade de o tringulo AVB ser equiltero. Como conhecido, as coordenadas dos pontos A, V e B so dadas em funo dos coeficientes da funo quadrtica:
A=( b ; 0), 2a B=( b + ; 0) 2a
2

V =(

b ; ). 2a 4a

Podemos usar o teorema de Pitgoras para calcular AV:


2 b (b AV = + 2a 4a 2

ou

AV =

( + 4) . 4a

RPM OBMEP

Por outro lado, AB =

b + b = . 2a 2a a

42

Para que AVB seja equiltero, devemos ter AB = AV, o que equivalente a 12 = 2 ou, como > 0, equivalente a = 12. Adaptado do artigo
O nmero 12 Carlos Grosso, RPM 67.

Painel II Sexta-feira 13
As pessoas criaram um mito sobre as sextas-feiras 13, dizendo que essas datas so propcias para ocorrer coisas macabras, horrveis... Parece que nas ltimas dcadas esse mito foi bastante reforado e divulgado pela srie de filmes Sexta-feira 13 que o cinema exibiu. Mas no so apenas coisas ruins que esto ligadas sexta-feira 13, muito pelo contrrio; temos um belo problema de Matemtica: todo ano h pelo menos uma sexta-feira 13. Para verificarmos o prometido, vamos inicialmente enumerar os dias 13 de um determinado ano. Para isso imaginemos um ano de 365 dias (se o ano tiver 366 dias, o mesmo mtodo funciona!). Lembre que, num ano de 365 dias, os meses de janeiro, maro, maio, julho, agosto, outubro e dezembro tm 31 dias, enquanto abril, junho, setembro e novembro tm 30 dias e fevereiro tem 28 dias. Assim, temos que:
1o de janeiro dia 1 2 de janeiro dia 2 3 de janeiro dia 3 ... 13 de janeiro dia 13 ... 13 de fevereiro dia 44 13 de maro dia 72 ... 13 de abril dia 103 ... 13 de maio dia 133 ... 13 de junho dia 164

RPM OBMEP

43

... 13 de julho dia 194 ... 13 de agosto dia 225 ... 13 de setembro dia 256 ... 13 de outubro dia 286 ... 13 de novembro dia 317 ... 13 de dezembro dia 347

Assim, temos que os dias 13 de um determinado ano de 365 dias so 13, 44, 72, 103, 133, 164, 194, 225, 256, 286, 317 e 347, que, quando divididos por 7 (uma semana tem sete dias), deixam restos 6, 2, 2, 5, 0, 3, 5, 1, 4, 6, 2 e 4, respectivamente. Perceba que todos os restos possveis de uma diviso por 7 apareceram, isto , 0, 1, 2, 3, 4, 5 e 6. Assim, perceba que: se a primeira sexta-feira do ano for dia x (x no mximo 7), as sextas-feiras seguintes sero os dias x + 7, x + 14, x + 21, ... Se, por exemplo, x for 7, ento todas as sextasfeiras do ano cairo nos dias 7, 14, 21, 28, 35, 42, ... do referido ano. Como entre os dias 13 h um que mltiplo de 7 (o dia 133), segue que esse dia ser uma sexta-feira 13 (isso ocorreu em 2005; veja que 13 de maio de 2005 foi uma sexta-feira 13). Seguindo o mesmo raciocnio, se a primeira sexta-feira do ano fosse dia 6 de janeiro, ento as sextas-feiras seriam os dias 6, 13, 20, 27, 34, 41, ... Como entre os dias 13 h um cujo resto da diviso por 7 6 (o dia 13), segue que, nesse ano, 13 de janeiro seria uma sextafeira 13. Esse raciocnio mostra que em qualquer ano existe pelo menos uma sexta-feira 13. Perceba que pode haver mais de uma sexta-feira 13. Se, por exemplo, o dia 2 de janeiro for uma sexta-feira, ento as demais sextas-feiras desse ano sero os dias 2, 9, 16, 23, 30, 37,44, ..., ou seja, os dias que deixam resto 2 quando divididos por 7. Assim, em um ano de 365 dias em que 2 de janeiro uma sexta-feira,

RPM OBMEP

44

os dias 44, 72 e 317 (que divididos por 7 deixam resto 2) seriam sextasfeiras 13. Noutras palavras, 13 de fevereiro, 13 de maro e 13 de novembro seriam sextas-feiras 13 (que ano azarado, hein??? Prepare-se, 2009 ser assim!). Adaptado do artigo
Sexta-feira 13

Carlos A. Gomes, RPM 59.

Painel III O jogo de bilhar


Estava numa pousada, no salo de jogos, observando uma partida de bilhar. Em dado momento, apresentou-se a situao ilustrada na figura, sendo que o jogador precisava acertar a bola cinza, mas no podia bater na bola preta. Para ajudar, um amigo do jogador adotou a estratgia: mediu, com um outro taco, colocado apoiado na direo perpendicular borda da mesa, como na figura, a distncia d da bola cinza at o ponto B, na borda da mesa.

marcou nesse taco o ponto A tal que a distncia BA vale d. disse ao jogador para mirar no ponto A e bater na bola branca. Ao bater no ponto C, na borda da mesa, a bola branca, no movimento refletido, acertou a bola cinza. A pergunta que me ocorreu foi: Por que deu certo? A resposta fundamentase na lei fsica que afirma que, na

RPM OBMEP

45

situao descrita, a medida do ngulo de incidncia da bola, ao bater na mesa, igual ao ngulo de reflexo. O ngulo BCA e o ngulo de incidncia so opostos pelo vrtice, logo tm a mesma medida , mostrando que a reta r a bissetriz do ngulo DCA, sendo D um ponto da trajetria de reflexo. Em consequncia, o simtrico de A, em relao a r, que o ponto no qual est a bola cinza, pertence reta CD. Logo, a trajetria de reflexo da bola branca passa pela bola cinza. Alm disso, a estratgia adotada fornece a trajetria de menor percurso para a bola branca atingir a bola cinza nas condies do problema. Isso garantido pelo teorema a seguir, atribudo a Heron, matemtico de Alexandria que viveu no primeiro sculo depois de Cristo. Teorema de Heron Dada uma reta r e dois pontos P e Q, no mesmo lado da reta r, o ponto R sobre a reta r tal que a distncia PR + RQ a menor possvel aquele em que os ngulos que os segmentos PR e RQ fazem com a reta r so iguais. Demonstrao do teorema Seja Q o simtrico de Q em relao reta r. Por hiptese, a reta r bissecciona o ngulo QRQ. Segue, por congruncia de tringulos, a igualdade QR = QR. Seja R qualquer ponto sobre a reta r, diferente de R. Ento, QR + RP = QR + RP = QP. Mas QP < QR + RP= QR + RP. Logo, QR + RP < QR + RP.
RPM OBMEP

Adaptado do artigo
O jogo de bilhar Jos Carlos Magossi, RPM 69.

46

Painel IV Codificando e decifrando mensagens


Operaes de servios disponveis na Internet, movimentaes bancrias e outras transaes eletrnicas necessitam da criptografia para comunicao confidencial de dados. A palavra criptografia tem origem grega (kripto = escondido, oculto; grapho = grafia) e define a arte ou cincia de escrever mensagens em cdigos, de forma que somente pessoas autorizadas possam decifr-las. A criptografia to antiga quanto a prpria escrita; j estava presente no sistema de escrita hieroglfica dos egpcios e os romanos utilizavam cdigos secretos para comunicar planos de batalha. Contudo, desde aquele tempo, seu princpio bsico continua o mesmo: encontrar uma transformao (funo) injetiva f entre um conjunto de mensagens escritas em um determinado alfabeto (de letras, nmeros ou outros smbolos) para um conjunto de mensagens codificadas. O fato de f ser inversvel a garantia de o processo ser reversvel e as mensagens poderem ser reveladas pelos receptores. O grande desafio de um processo criptogrfico, portanto, est em ocultar eficientemente os mecanismos (chaves) para a inverso de f, de modo que estranhos no possam fazlo.

Descreveremos aqui dois exemplos elementares de processos criptogrficos, sendo o primeiro acessvel inclusive para alunos do ensino fundamental. Inicialmente, relacionamos nmeros ao alfabeto (o smbolo # representa um espao em branco) que vamos utilizar nos modelos. Assim:
RPM OBMEP

Portanto, cifrar uma mensagem recai no problema de permutar nmeros por meio de uma regra f. Pode-se fazer isso, de forma muito

47

prtica, por exemplo, atravs das funes afins f(x) = ax + b com a, b inteiros, a 0, definidas no conjunto {0, 1, ..., 26}. Suponhamos que Ana e Ivo desejem trocar mensagens sigilosas utilizando o alfabeto escolhido. O primeiro passo a tomarem definirem a funo cifradora, digamos f(x) = 2x 3. Assim, por exemplo,
mensagem R E V I S T A R P M Ana associa a sequncia numrica 18 5 22 9 19 20 1 0 18 16 13

mas transmite a Ivo a seqncia numrica obtida pelas imagens de f, isto , 33 7 41 15 35 37 1 3 33 29 23.

x+3 nessa 2 sequncia e utilizando a correspondncia alfabeto-numrica, obtm a mensagem original.


1 Ao receb-la, Ivo, calculando a imagem de f ( x) =

Depois de os alunos dominarem o processo, seria oportuno que o professor propusesse situaes em que um intruso tente decifrar mensagens apoderando-se das sequncias numricas codificadas. Como estamos utilizando funes afins, para tanto suficiente apenas duas associaes corretas entre nmeros das sequncias original e codificada. Admitindo conhecidas essas associaes, um exerccio interessante para os alunos determinarem f. O segundo mtodo criptogrfico que apresentaremos utiliza matrizes invertveis como chaves, o que dificulta um pouco mais sua violao. Suponhamos que Ana e Ivo combinem previamente utilizar a matriz
2 3 2 1 1 A= e sua inversa A = como chaves. Para trans1 1 1 3 mitir a mesma mensagem acima, Ana inicialmente monta uma matriz mensagem M dispondo a sequncia numrica associada em coluna e completa a posio restante com 0, ou seja, obtm 18 22 19 1 18 13 M = . 5 9 20 0 16 0 Em seguida, codifica-a calculando,

RPM OBMEP

48

3 2 18 22 19 1 18 13 64 84 97 3 86 39 . AM = = 4 1 1 5 9 20 0 16 0 23 31 39 1 34 13

e transmite a seqncia 64 23 84 31 97 39 3 1 86 34 39 13. Para ler a mensagem recebida, Ivo, da mesma forma, restaura a forma matricial AM, e em seguida, com sua chave A1, pode recuperar M atravs da identidade matricial, M = A1(AM). Os mtodos tratados neste trabalho tem apenas carter instrutivo. Na prtica atual so pouco utilizados pela inconvenincia de exigirem trocas prvias de chaves entre os usurios. So, portanto, inviveis na descrio de transaes eletrnicas nas quais um nico receptor recebe dados de milhares de emissores, como ocorre em vendas pela Internet, transaes bancrias e outras. Mesmo nesses casos mais complexos, a Matemtica resolveu a trama, e desta vez, quem diria, o ramo da Teoria dos Nmeros. O leitor interessado neste envolvente tema poder consultar a apostila Criptografia, IC-OBMEP 2007. Adaptado do artigo Codificando e decifrando mensagens Antonio Carlos Tamarozzi, RPM 45.

Painel V Qual a relao entre os nmeros 102.564 e 410.256?


Facilmente observamos que o segundo nmero do ttulo, 410 256, o qudruplo do primeiro, 102 564. O que nos chama a ateno que os algarismos desses nmeros so os mesmos, tendo sido o bastante trasladarmos o algarismo 4 das unidades do primeiro nmero para a esquerda, a fim de obtermos o segundo nmero. Admitamos agora a questo sendo apresentada sob o seguinte aspecto: determinar um nmero inteiro positivo N, formado de n algarismos e terminando pelo algarismo 4, tal que ao trasladarmos esse 4 (algarismo das unidades) para a primeira posio, obtemos outro nmero que o qudruplo desse nmero N.
RPM OBMEP

49

Resoluo Seja N = a1a2a3...an1 4 um nmero de n algarismos, n natural no nulo. Retirando o algarismo 4 desse nmero, obtemos:

N' =

N 4 = a1a2 a3 ...an 1 . [No exemplo: 10256 = (102 564 4)/10.] 10 Colocando o algarismo 4 esquerda do primeiro algarismo de N, N 4 . 10

obtemos

N " = 4a1a2 a3 ...an 1 = 4 10n 1 + a1a2 a3 ...an 1 = 4 10n 1 +


Para que N = 4N precisamos ter
4 N = 4.10n 1 +

N 4 4(10n 1) 40 N = 4.10n + N 4 N = . 10 39

Para N ser inteiro, devemos ter 39 como divisor de 10n1. O menor valor de n que satisfaz essa condio n = 6:
N= 4(106 1) 4(999999) = = 4 25641 = 102564. N = 410256 = 4N. 39 39

Podemos mostrar que, fazendo n = 6k, k = 1, 2, 3, ..., k N*, obtemos todos os nmeros N terminados em 4 e que satisfazem a condio procurada; logo, N = Exemplos Para k = 2, obtemos N = 102564102564. Para k = 3, obtemos N =
RPM OBMEP

4(106 k 1) , k = 1,2.3,.........., k N*. 39

4(1012 1) , donde N = 4 25641025641 = 39

4(1018 1) , donde 39 N = 4 2564102564125641= 102564102564102564.

Podemos propor problemas semelhantes ao anterior, como, por exemplo, obter um nmero inteiro positivo N, formado por n algarismos e terminando com o algarismo a, tal que ao trasladarmos esse a (algarismo das unidades) para a primeira posio, temos como resultado

50

outro nmero que igual a aN. Procure obter N fazendo a = 1, 2, 3, ..., 9, verificando os resultados curiosos que sero obtidos, podendo, em alguns casos, no haver soluo.
Adaptado do artigo Qual a relao que existe entre os nmeros 102.564 e 410.256? Augusto Manoel de Albuquerque Barros, RPM 63.

Painel VI Uma demonstrao visual para a frmula do sen(A + B)


Cada um tem a sua demonstrao favorita das importantes frmulas de sen(A B) e cos (A B). De qualquer forma, sabido que, deduzida uma delas, as outras podem ser obtidas por complemento, suplemento, etc. Uma das mais simples e rpidas uma demonstrao visual, que se baseia, na igualdade a = b cosC + c cosB onde a, b, c, A, B, C so os lados e ngulos respectivos de um tringulo. A igualdade pode ser obtida facilmente e diz apenas que o lado a a soma (ou a diferena, se B ou C for obtuso) das projees ortogonais dos lados b e c sobre o prprio a, como se v na figura ao lado. Por outro lado, tambm bastante conhecida a lei dos senos em um tringulo, segundo a qual:
a b c = = = 2R senA senB senC
RPM OBMEP

onde R o raio do crculo circunscrito. Isso decorre imediatamente da figura da pgina seguinte. Ento num crculo de dimetro 1, tem-se: a = senA, b = senB e c = senC.

51

Para um tringulo inscrito nesse crculo, a igualdade inicial fica: senA = senB cosC + senC cosB . E como, finalmente, o ngulo A o suplemento de B + C, ou seja, tm o mesmo seno, obtm-se a clebre frmula: sen(B + C) = senB cosC + senC cosB . Essa deduo vlida para B + C < 180o, o que suficiente para deduzir o caso geral.
senA = a 2 a = 2R R senA

A demonstrao anterior baseia-se numa ideia de S.H. Kung, encontrada na revista Mathematics Magazine, vol. 64, no 2, abril de 1991.
Adaptado do artigo Demonstraes visuais Jos Paulo Q. Carneiro, RPM 27.

Painel VII Valores irracionais de funes trigonomtricas


RPM: O que segue uma transcrio adaptada de alguns resultados encontrados no livro Nmeros: racionais e irracionais, de I. Niven, SBM,

RJ, 1984, que decidimos publicar por julgar do interesse de nossos leitores. So conhecidas as identidades trigonomtricas cos2 = cos2 sen2, sen2 = 2sencos, sen(a + b) = sena cosb + senb cosa e cos(a + b) = cosa cosb + sena senb,
RPM OBMEP

as quais, juntamente com a relao fundamental, cos2 + sen2 = 1, implicam cos3 = 4cos3 3cos. Fazendo = 20 na ltima igualdade, obtemos:

52

1 = cos 60 = 4 cos3 20 3 cos 20 . 2 Se escrevemos x no lugar de cos20, obtemos a equao


8x3 6x 1 = 0, que por construo tem cos20 como raiz. Aplicando a essa equao o conhecido resultado sobre razes racionais de equaes polinomiais: Se p/q, frao irredutvel, raiz de uma equao com coeficientes inteiros anxn + an1xn1 + ... + a1x + a0 = 0, ento p divisor de a0 e q divisor de an, temos que as nicas possveis razes racionais da equao so

1 1 1 1, , e . Mas, substituindo-se na equao, um clculo 2 4 8 simples mostra que nenhum desses nmeros raiz; logo, a equao no tem razes racionais e, portanto, cos20 um nmero irracional.
Tambm temos cos20 = cos210o sen210o = 1 2sen210o. Logo, se sen10 fosse racional, ento 1 2sen210o seria racional, o que implicaria cos20 racional, o que uma contradio. Portanto, sen10 irracional. Usando cos20 = cos210o sen210o = 2cos210o 1, conclui-se, de modo anlogo, que cos10 tambm irracional. Generalizando, temos o resultado: Se for um ngulo tal que cos2 irracional, ento cos, sen e tg so tambm irracionais. A verificao de que cos e sen so irracionais se faz de modo anlogo ao utilizado para = 10, usando as igualdades cos2 = cos2 sen2 = 1 2sen2 = 2cos2 1. 1 cos 2
RPM OBMEP

Finalmente, se tg fosse racional, ento tg2 seria racional e de 1 + tg 2 = sec 2 = ,

53

teramos cos racional e, novamente, concluiramos que cos2 racional, uma contradio. Portanto, tg irracional. Com repetidas aplicaes do resultado anterior mostra-se que cos, sen e tg so irracionais, para, por exemplo, os valores de : 5; 2 30; 1 15; 3730", etc.
Adaptado do artigo Valores irracionais de funes trigonomtricas Paulo A. da Mata Machado e Aldo Trajano Lourdo, RPM 46.

Painel VIII Mgica com nmeros


Truques de adivinhaes aritmticas tm sido apresentados a pessoas e alunos de vrios nveis de escolaridade e sempre causam surpresa e fazem muito sucesso. Vamos apresentar o truque da adivinhao egpcia com a subseqente explorao das propriedades aritmticas subjacentes a ele. Nesse truque o apresentador pede a um espectador que pense em um nmero de 10 a 100. O apresentador segue ento os seguintes passos: 1. Pergunta ao espectador se o nmero par ou mpar. Ouvida a resposta, se for par, pede ao espectador que divida o nmero por 2. Se for mpar, pede a ele que subtraia 1 e que ento divida o resultado por dois. 2. Pergunta se o resultado obtido par ou mpar e, ouvida a resposta, pede ao espectador para repetir o procedimento descrito no item 1. 3. O procedimento continua com cada novo resultado at o resultado (quociente de uma diviso por 2) tornar-se igual a 1, quando ento os clculos do espectador terminaro. Quando o apresentador informado de que o resultado igual a 1, ele revela imediatamente ao espectador o nmero pensado por ele.

RPM OBMEP

54

Como funciona o truque da adivinhao egpcia Suponhamos que o nmero pensado pelo espectador seja 52. Nas sucessivas etapas, ele efetuar as contas da coluna abaixo esquerda, enquanto simultaneamente o apresentador ir fazendo, secretamente, as anotaes da coluna direita. Aluno 52 (nmero pensado) 26 13 6 3 1 Professor 1 2 4 8 16 32

Para cada nmero mpar informado pelo espectador, o apresentador anota . Nos sucessivos estgios da brincadeira, o apresentador marca as potncias de 2, iniciando em 20 = 1. Em seguida, o apresentador soma as potncias de 2 correspondentes s marcas , 4 + 16 + 32 = 52, e resgata o nmero que foi pensado pelo espectador! O truque foi concebido observando o mtodo das divises sucessivas por 2, usado para representar um inteiro positivo no sistema binrio, isto , como soma de potncias (distintas) de 2, a partir de sua representao no sistema decimal. Nesse mtodo, tomando como exemplo o nmero 52, fazemos a seguinte escada de divises sucessivas por 2, at atingirmos quociente igual a 1, quando o algoritmo termina. Lendo da direita para a esquerda os 0s e 1s, que so o ltimo quociente e os restos das divises, obtemos a representao do nmero 52 (aqui representado no sistema decimal) no sistema de numerao de base 2:

RPM OBMEP

55

52 = (110100)2 = 1 25 + 1 24 + 0 23 + 1 22 + 0 21 + 0 20 = 22 + 24 + 25. Na seqncia das divises, um resto ser 0 quando o dividendo for par, e 1 quando o dividendo for mpar, da a importncia de tomar nota apenas das potncias de 2 correspondentes aos restos mpares. O ttulo adivinhao egpcia inspirado nos algoritmos de multiplicao dos antigos egpcios, baseados na decomposio de inteiros positivos como somas de potncias distintas de 2.
Adaptado do artigo Mgicas com nmeros Joo C. V. Sampaio, RPM 60.

Painel IX Destreza ou esperteza?


Certa vez, quando eu tinha 15 anos, um amigo da minha famlia afirmou que sabia fazer contas mentalmente e com muita rapidez. Para provar isso, props a seguinte brincadeira: Vou escrever um nmero com sete algarismos. Em seguida, voc escreve, abaixo do meu nmero, outro nmero com sete algarismos. Repetimos isso mais uma vez, eu escrevo meu terceiro nmero e, ento, eu direi a voc, sem fazer clculos, qual o valor da soma dos cinco nmeros. Eu, um tanto desconfiado, aceitei a proposta, ocorrendo o seguinte: 1o nmero escrito por ele: 1o nmero escrito por mim: 2o nmero escrito por ele: 2o nmero escrito por mim: 3o nmero escrito por ele: Soma fornecida por ele: 3 574 186 1 247 064 8 752 935 4 955 231 5 044 768 23 574 184

RPM OBMEP

56

Conferi a soma manualmente e constatei que estava correta. Fiquei atnito observando aqueles nmeros por alguns instantes, mas nada consegui concluir. Ele props outra conta e novamente acertou o resultado em poucos segundos. Claro que eu sabia (ou desconfiava) que existia algum truque por trs daquilo, mas fiquei por alguns anos sem saber qual era. Vamos agora mostrar que, na realidade, tudo no passa de um pouquinho de lgebra: observe que o segundo e o terceiro nmeros escritos por ele so construdos a partir do anterior, de modo que a soma com o anterior seja igual a 9 999 999. Veja: 1o nmero escrito por mim + 2o nmero escrito por ele 1 247 064 + 8 752 935 = 9 999 999 2o nmero escrito por mim + 3o nmero escrito por ele 4 955 231 + 5 044 768 = 9 999 999 Observe agora que, como 9 999 999 = 10 000 000 1, a soma total igual a: primeiro nmero somado + 2 (10 000 000 1) = 20 000 000 2, ou seja, (3 574 186 + 20 000 000) 2 . Para efetuar a soma entre parnteses, observando que o nmero de zeros em 20 000 000 igual ao nmero de dgitos do nmero inicial, basta acrescentar o dgito 2 na frente do nmero original, o que resulta em 23 574 186. Subtraindo 2, obtemos a soma. Note que, para realizar a ltima operao, no caso em que o algarismo das unidades do primeiro nmero maior do que ou igual a 2, basta subtrair 2 do algarismo das unidades, mantendo os outros dgitos inalterados. Se ele for 0 ou 1, ento a subtrao um pouco mais complicada, sendo necessrio emprestar 1 do algarismo das dezenas para depois subtrair 2. Como 10 2 = 8, isso equivalente a subtrair 1 do algarismo das dezenas e somar 8 ao algarismo das unidades, se esse no for nulo. Se o algarismo das dezenas for nulo, ento preciso emprestar 1 do algarismo das centenas e assim por diante.
RPM OBMEP

Observe que, no caso do desafio proposto pelo amigo de minha famlia, o nmero inicial 3 574 186. Colocando 2 no incio, obtemos 23 574 186. Subtraindo 2 do algarismo das unidades, obtemos 23 574 184, que a soma procurada.

57

Se algum o desafiar, voc pode tentar dificultar o trabalho para o desafiante dizendo: Quero ver se voc acerta o resultado no caso do primeiro nmero escrito ter o algarismo das unidades menor que 2, ou seja, igual a 0 ou 1, e o das dezenas nulo. Isso testar se ele entendeu realmente como funciona o truque, que pode ser adaptado facilmente para o caso de mais dgitos ou para um nmero maior de somandos. Deixamos para o leitor esse trabalho.
Adaptado do artigo Destreza ou esperteza? Vanderlei Nemitz, RPM 64.

Painel X Determinante para fatorar


H alguns anos, quando ainda existia a Unio Sovitica, submeteuse aos participantes de uma olimpada juvenil de Matemtica a seguinte questo, aparentemente simples: Fatorar a expresso a3 + b3 + c3 3abc Mesmo bons professores de Matemtica, se no conhecerem algum truque, tero dificuldade em resolver esse problema pelo mtodo direto. Quem duvidar, que o tente. Entretanto, a teoria dos determinantes d uma soluo fulminante ao problema. Vejamos: seja o determinante a b c c a b = a3 + b3 + c3 abc abc abc = a3 + b3 + c3 3abc , b c a exatamente a expresso que desejamos fatorar.
RPM OBMEP

O determinante no se altera se substitumos, por exemplo, a primeira linha da matriz por sua soma com as duas outras, ou seja

58

a b c (a + b + c) (a + b + c) (a + b + c) c a b = c a b = b c a b c a 1 1 1 (a + b + c) c a b = (a + b + c)(a 2 + b 2 + c 2 ab ac bc) . b c a e o problema foi resolvido. Muitos realmente so os caminhos da Matemtica e precisamos ter a mente aberta e desbloqueada para encontr-los.
Adaptado do artigo Usando determinantes para fatorar Gilberto Garbi, RPM 41.

RPM OBMEP

59

Funes interessantes

A aplicao de situaes do cotidiano na motivao, estudo e ensino de tpicos de contedos programticos aumenta, na maioria da vezes, o interesse e compreenso dos alunos da educao bsica, alm de evidenciar que a Matemtica faz realmente parte da vida de todos ns. No ensino de funes, que pode ser iniciado j no nvel fundamental, as aplicaes so muito indicadas para fugir do formalismo terico. Nessa direo, vou apresentar e estudar alguns aspectos de funes bastante simples que modelam situaes reais e comuns. I. Em uma capital brasileira, os preos das corridas de txi tiveram o seguinte aumento: bandeirada: passou de R$ 3,20 para R$ 3,50, tendo, portanto, um aumento de aproximadamente 9,3%; quilmetro rodado: passou de R$ 1,80 para R$2,20, tendo, portanto, um aumento de aproximadamente 22,2%. A determinao da funo que fornece o preo de uma corrida j suscita uma discusso interessante. Vrios textos didticos apresentam funes que modelam situaes desse tipo como polinomiais de primeiro grau, cujo grfico uma reta. No nosso caso seria: P1(x) = 3,20 + x 1,80 P2(x) = 3,50 + x 2,20,

RPM OBMEP

60

onde P1(x) e P2(x) denotam o preo da corrida de x km antes e depois do aumento, respectivamente. Essa interpretao pressupe uma variao contnua no preo da corrida em funo dos quilmetros rodados. Mas a realidade no assim. O taxmetro varia em fraes no valor de R$ 0,30, ou seja, supondo que o carro no pare durante a corrida:
os valores de P1(x) variam 1,80/0,30 = 6 vezes durante cada km

rodado, o que significa a cada intervalo de rodagem de aproximadamente 166,66 m;


os valores de P2(x) variam 2,20/0,30 = 7, 333... vezes durante cada

km rodado, o que significa a cada intervalo de rodagem de aproximadamente 136,36 m. Conversas com taxistas nos fizeram concluir que eles no tm em mente o valor exato do comprimento do trecho percorrido antes de cada mudana no preo, apenas deduzem valores aproximados (recebemos respostas de 200 m, 150 m, etc.); dizem que quem determina o valor exato o INMETRO ao ajustar os aparelhos dos txis. Aqui cabe uma observao interessante: no caso da P1, o taxmetro muda um nmero inteiro de vezes, 6, em cada km rodado, o que no acontece na P2, uma vez que 2,20 no mltiplo de 0,30. Nesse caso, para que o taxmetro mude um nmero inteiro de vezes, necessrio que

2, 20 x seja inteiro, isto , que x seja mltiplo de 0,30, sendo x o 0, 30


nmero de km rodados. Isso significa que a expresso afim da funo P1 ou P2 fornece o preo exato de uma corrida de x km, se x , respectivamente, inteiro ou inteiro mltiplo de 3. Voltemos ento s funes, P1 e P2 reais, que mudam de valor aos saltos, a cada intervalo de 166,66 m ou de 136,36 m. Seus grficos tm a forma de escada, um exemplo no usual de funo . Esboamos, tambm, os grficos das P1 e P2 afins.

RPM OBMEP

61

Perguntas 1. Quais os preos, antigo e depois do aumento, de uma corrida de 3,5 km = 3500 m? Como 3500/166,6 um valor entre 21 e 22, vemos que o preo antigo dado pelo 22o degrau do grfico da funo P1(x), ento P1(3500) = 3,20 + 21 0,30 = 9,50, ou seja, o preo R$ 9,50. Um clculo anlogo mostra que o preo novo dessa corrida seria R$ 11,00. Vamos responder s perguntas a seguir, considerando as aproximaes de P1(x) e P2(x) pelas funes afins anteriormente consideradas. Isso permite estabelecer expresses algbricas simples para as funes envolvidas, alm do que os grficos acima mostram que a funo afim uma aproximao razoavelmente boa. 2. Qual ser o aumento percentual no preo de uma corrida de 10 km? Considerando P1(10) = 3,20 + 10 1,80 = 21,20 e P2(10) = 3,50 + 10 2,20 = 25,50, vemos que o aumento percentual de 20,28%.
RPM OBMEP

3. Qual a funo que fornece o aumento percentual numa corrida de x km? Considerando as funes afins, queremos, em funo de x, o valor de p tal que P2 ( x) = P ( x) + 1

p P ( x) , sendo P1(x) = 3,20 + x 1,80 e 1 100

62

P2(x) = 3,50 + x 2,20. Substituindo os valores e fazendo os clculos,

30 + 40 x . Como x > 0, temos que o domnio 3, 20 + 1, 80 x dessa funo o intervalo [0, +]. interessante observar que, para x = 0, o aumento igual a 30/3,20, que aproximadamente 9,3%, aumento da bandeirada. Para valores de x muito grandes, observando
obtemos p ( x) =

30 + 40 , vemos que p tende para p = 40/1,80 = que p ( x) = x 3, 20 + 1, 80 x


22,22222... que o aumento percentual do km rodado, isto , para corridas muito grandes, o aumento da bandeirada no conta, valendo apenas o aumento do km rodado. O grfico da funo p(x), a seguir, ilustra esse resultado e tambm evidencia uma peculiaridade dos taxistas: eles no tm como receber aumentos de um percentual fixo.

II. O governo de um Estado brasileiro mudou a contribuio previdenciria de seus contribuintes: de 6% sobre qualquer salrio passou para 11% sobre o que excede R$ 1200,00 nos salrios. Por exemplo, sobre um salrio de R$ 1700,00, a contribuio anterior era 0,11 (R$ 1700,00 R$ 1200,00) = R$ 55,00. Provavelmente os alunos no tero dificuldades em determinar as funes que fornecem o valor das contribuies em funo do valor x
RPM OBMEP

0,06 R$1700,00 = R$ 102,00 e a atual

63

do salrio. Sendo C1(x) a contribuio anterior e C2(x) a atual, temos:

C1 ( x) =

6 x = 0, 06 x 100

0 se 0 x < 1200 . C2 ( x) = 11 ( x 1200) se x 1200 100


Os grficos dessas funes esto esboados a seguir e uma anlise deles permite tirar vrias concluses, por exemplo: 1. Para um salrio de, aproximadamente, R$ 2700,00, o valor da contribuio permanece o mesmo, por volta de R$ 160,00. Para obter o valor exato do salrio que mantm a contribuio, basta resolver a equao 0,06x = 0,11(x 1200), chegando a x = 2640 e C1(2640) = C2(2640) = 158,40. 2. Para salrios abaixo de R$ 2640,00, a contribuio previdenciria diminuiu, pois nesse caso temos, para um mesmo x, C2(x) menor do que C1(x). Fica interessante fazer simulaes com salrios e populao para calcular os valores das arrecadaes antes e depois da mudana da lei, verificando que em determinadas situaes, bastante provveis, a arrecadao estadual diminui consideravelmente.

RPM OBMEP

64

3. C2(x) maior que C1(x) para salrios maiores que R$ 2640,00, logo a nova lei aumenta a contribuio dos salrios maiores que esse valor. 4. A inclinao da reta do grfico de C2(x), x > 1200, maior que a da reta de C1(x); logo, a contribuio, com a nova lei, aumenta mais rapidamente do que antes, medida que o salrio aumenta.
Adaptado do artigo Funes interessantes Ana Catarina P. Hellmeister, RPM 63.

RPM OBMEP

65

A formiga inteligente

Um problema Imagine dois postes verticais AA e BB de tamanhos diferentes no plano horizontal . Para que posies uma formiga P, no plano, v os dois postes do mesmo tamanho?

RPM OBMEP

Em primeiro lugar, devemos pensar o que ocorre quando vemos dois objetos com o mesmo tamanho. Por exemplo, uma moedinha de 1 centavo segura entre os dedos com o brao esticado tem, aparentemente, o mesmo tamanho da lua cheia. A concluso a seguinte: dois objetos aparentam ter o mesmo tamanho para certo observador, quando os ngulos de visada so iguais. Portanto, observando a figura acima, a formiga v os postes AA e BB do mesmo tamanho se os ngulos de visada APA e BPB forem iguais. Mesmo sem pensar ainda como resolver o problema, a formiga inteligente pode verificar que existem dois

66

lugares onde isso acontece, ambos na reta AB. Andando na reta AB, de A para B, certamente encontraremos um ponto interior ao segmento AB, onde APA = BPB, como mostra a figura.

Se o poste A for maior que o poste B, esse ponto P estar obviamente mais prximo de B do que de A e, para obter exatamente a posio de P, basta ligar o ponto A ao simtrico de B em relao reta AB. Por outro lado, uma segunda posio para P a interseo da reta AB com a reta AB. A figura a seguir mostra uma outra posio de P, onde APA = BPB.

Alm dessas duas posies determinadas intuitivamente, existe certamente uma infinidade de outras, no plano, mas fora da reta AB. J sabemos que os ngulos de visada APA e BPB so iguais para que a formiga veja os dois postes do mesmo tamanho. Porm, como os postes so verticais, isso significa que os tringulos APA e BPB so

PA AA que constante, pois a razo entre = PB BB os comprimentos dos dois postes (veja novamente a primeira figura).
semelhantes. Portanto,
RPM OBMEP

Temos ento dois pontos fixos A e B no plano e buscamos o lugar geomtrico dos pontos cuja razo das distncias a esses pontos constante e igual razo entre os comprimentos dos postes.

67

Antes de resolver o problema, interessante recordar o teorema das bissetrizes: Uma bissetriz de um ngulo de um tringulo divide o lado oposto na mesma razo dos lados adjacentes. Veja a seguir uma demonstrao desse importante teorema. a) Na figura abaixo, AD bissetriz interna do ngulo A do ABC. Traando por D as perpendiculares DM e DN aos lados AB e AC, temos que DM = DN, pois o ponto D est na bissetriz do ngulo A. Por outro lado, como os tringulos ADB e ADC tm mesma altura a partir de A, ento a razo entre suas reas, A, igual razo entre suas bases, ou seja,
DB A( ADB ) ( AB DM ) / 2 AB = = = . DC A( ADC ) ( AC DN ) / 2 AC

b) Na figura a seguir, AE bissetriz externa do ngulo A do ABC.

RPM OBMEP

Traando por E as perpendiculares EP e EQ s retas AB e AC, temos que EP = EQ, pois o ponto D est na bissetriz do ngulo externo A. Por outro lado, como os tringulos AEB e AEC tm mesma altura a partir de A, ento a razo entre suas reas, A, igual razo entre suas bases, ou seja,

EB A( AEB ) ( AB EP ) / 2 AB = = = . EC A( AEC ) ( AC EQ) / 2 AC

68

Demonstrado o teorema das bissetrizes, importante lembrar que vale a sua recproca, ou seja, se D um ponto da base BC do ABC e
DB AB , ento AD bissetriz interna do ngulo A e, se E um = DC AC EB AB = , ento AE bissetriz EC AC

ponto do prolongamento de BC e externa. A circunferncia de Apolnio

Passamos agora a analisar o problema seguinte: Dados dois pontos A e B no plano e um nmero k > 0, determinar o lugar geomtrico do ponto P tal que

PA = k. PB

Em primeiro lugar, se k = 1, temos PA = PB e o lugar geomtrico de P naturalmente a mediatriz de AB. Em seguida, vamos imaginar k > 1 (o caso 0 < k < 1 inteiramente anlogo). Como j vimos antes, podemos encontrar com alguma facilidade dois pontos da reta AB que possuem a propriedade desejada. Sejam portanto M e N pontos da reta AB tais que

MA NA = = k. MB NB

Observe ento a figura a seguir em que as retas r e s so paralelas.

Sabemos que, dado um segmento unitrio, para cada real positivo k existe um segmento de comprimento k e a semelhana de tringulos

RPM OBMEP

69

permite concluir que, na figura acima, temos

MA NA = = k . Ateno: MB NB a figura acima mostra que os pontos M e N existem, mas a construo com rgua e compasso s possvel se k construtvel (racionais, por exemplo, so construtveis). Na situao acima, dizemos que os pontos M e N dividem harmonicamente o segmento AB.

Vamos agora considerar um ponto P fora da reta AB tal que PA = k PB e investigar seu lugar geomtrico. Observe a figura a seguir.

Como MA = PA , ento PM bissetriz interna do ngulo APB e MB PB como

NA PA , ento PN bissetriz externa. Mas essas bissetrizes = NB PB so perpendiculares (verifique!) e, como M e N so fixos, o lugar geomtrico de P a circunferncia de dimetro MN.

RPM OBMEP

Essa circunferncia chama-se circunferncia de Apolnio do segmento AB na razo k. Ela o lugar geomtrico dos pontos cuja razo das distncias a dois pontos fixos igual a uma constante dada.

70

O problema da formiga est resolvido. O lugar geomtrico dos pontos de onde a formiga v os postes de mesmo tamanho a circunferncia de Apolnio do segmento AB na razo AA/BB.

Nota Apolnio de Perga viveu no sculo 3 a.C. Foi clebre gemetra e astrnomo, mas a maior parte de sua vasta obra desapareceu. Felizmente, a sua obra-prima As Cnicas foi quase toda preservada. Entretanto, so conhecidos os ttulos e contedos dos muitos tratados que escreveu devido a relatos de matemticos posteriores. Sabemos por isso que Apolnio escreveu um livro chamado Lugares Planos dedicado anlise de diversos lugares geomtricos e que um deles era justamente o lugar geomtrico dos pontos cuja razo das distncias a dois pontos fixos constante. Esse lugar geomtrico ficou conhecido at hoje como Circunferncia de Apolnio, um tanto injustamente, pois Aristteles j o tinha descoberto anos antes. Adaptado do artigo
A formiga inteligente Eduardo Wagner, RPM 61.

RPM OBMEP

71

A demonstrao feita por Heron

Quando pequeno, li sobre Heron de Alexandria em uma enciclopdia biogrfica que havia em casa. Fiquei sabendo que ele viveu no sculo II d.C. na cidade de Alexandria (obviamente), que foi engenheiro e matemtico. No me lembro que outras coisas mais havia sobre Heron, mas ficou gravada em minha memria a frmula que l estava para calcular a rea de um tringulo:
A= p ( p a )( p b)( p c) ,

sendo p a metade do permetro do tringulo. O que me encantou nessa frmula? No sei. Talvez por ter uma raiz quadrada, que naqueles dias escolares lhe dava um ar de Matemtica superior; ou pelo fato de s usar os lados do tringulo, e no a altura, como na formulinha usada na escola. Anos mais tarde, aps ter encontrado vrias vezes a frmula e at depois de ter visto sua demonstrao como mero corolrio de um clculo de medianas, continuava intrigado: como Heron a havia demonstrado?
RPM OBMEP

Este ano comprei o livro Introduo Histria da Matemtica, de Howard Eves, e qual no foi minha surpresa ao encontrar no livro a meno de que, a demonstrao feita por Heron (que est em seu livro A

72

mtrica) estava esquematizada num dos exerccios do livro. Com algumas pequenas modificaes aqui vai ela:

BC = a AC = b AB = c

p=

a+b+c 2

1. rea ABC = rea ABI + rea IBC + rea AIC =


r ( AB + BC + CA) = rp , sendo r o raio da circunferncia inscrita. 2

2. Como ADI AIF, DBI IBE e FIC IEC, temos AD = AF, DB = BE e CE = CF. 3. Seja J o ponto da semi-reta AB tal que BJ = CE.

AJ =
Ento

AD + AF BD + BE CE + CF AB + BC + CA + + = = p. 2 2 2 2
p c = AJ AB = BJ; p b = AJ AC = DB e p a = AJ BC = AD.
RPM OBMEP

4. i) Seja K o ponto construdo como indicado na figura. O quadriltero AKBI inscritvel numa circunferncia de dimetro AK; logo AIB + AKB = 180 o e, como + + = 180, temos AIB + CIE = 180o, de onde AKB = CIE = .

73

Ento temos CIE AKB, o que implica AB = CE = BJ . BK r r ii) No tringulo retngulo ALI temos r2 = DL.AD e de DLI BLK (verifique) temos

BK r . = LB DL

iii) De i) e ii) temos AB = LB , o que implica AB + BJ = LB + DL BJ DL BJ DL ou


AJ AJ DB AD , que juntamente com r2 = DL.AD leva a . = . BJ AJ DL AD AJ2.r2 = BJ.AJ.BD.AD.

Usando-se as igualdades apresentadas em 3, obtemos p2r2 = (p c)p(p b)(p a), que, pela igualdade exibida em 1, demonstra a frmula.
Adaptado do artigo A demonstrao feita por Heron Mrio Dalcin, RPM 36.

RPM OBMEP

74

A Matemtica da folha de papel A4

Introduo O formato do papel que usamos rotineiramente nos servios de impresso ou fotocpia possui uma histria fascinante e repleta de Matemtica. Neste artigo, compartilho com o leitor algumas idias que esto por trs dessa histria. A intrigante folha de papel A4 O formato de papel mais usado para impresses e fotocpias, que recebe a denominao A4, tem 210 milmetros de altura por 297 milmetros de largura. Diferentemente do que se possa imaginar, a razo 297/210 no a razo urea (ver artigo Retngulo ureo, diviso urea e sequnci de Fibonacci). Analisaremos, a seguir, de onde vem essas estranhas medidas. Inicialmente padronizaremos neste artigo que as palavras largura e altura sempre sero usadas como referncia ao maior e ao menor lado de um retngulo, respectivamente. Imagine-se tendo que resolver o seguinte problema: qual deve ser a largura e a altura de uma folha retangular de modo que, quando ela for dividida ao meio, os dois novos retngulos obtidos mantenham a razo entre altura e largura da folha original?
RPM OBMEP

75

O problema de soluo simples, como se v a seguir:

L A = L2 = 2 A2 L = 2 A A L 2
Portanto, a folha retangular com razo L/A igual a 2 a nica que, quando dividida ao meio, conforme processo descrito, resultar em retngulos semelhantes ao da folha original. Lembramos que de forma diferente dos tringulos, onde bastam ngulos congruentes para que sejam figuras semelhantes, no caso dos quadrilteros a semelhana s se garante se os ngulos forem congruentes e se a razo entre os lados das figuras for preservada. No caso das medidas de uma folha A4, note que 297/210 uma tima aproximao racional para pequeno, da ordem de centsimo de milsimo.
2 , com erro muito

A classificao de papis da qual A4 faz parte chama-se srie A, que comea com o A0 e vai at o A10. Essas folhas tm em comum a razo 2 entre largura e altura. A srie comea com uma folha retangular de rea 1 m, definida como A0. A partir dela obtemos a folha do formato seguinte, A1, dividindo-se A0 ao meio. As dimenses da folha A0, em metros, podem ser obtidas a partir da soluo do seguinte sistema de equaes:
4 L = 2A 8 e L = 4 2 ou A = 2 4 e L = 2 4 . A= 2 L A = 1

Passando essas medidas para milmetros, e aproximando para o milmetro mais prximo, encontramos as dimenses da folha A0, que so 841 mm de altura por 1189 mm de largura.
RPM OBMEP

Faamos agora os clculos da folha A1, que obtida a partir da diviso ao meio da folha A0:

76

L = 2A 3 1 4 e L=2 4. 1 A=2 L A = 2
Adota-se, nesse caso, a aproximao 594 mm por 841 mm. Dividindo-se A1 ao meio, obtemos A2, que dividida ao meio resultar A3, e assim por diante at A10. Pode-se verificar de maneira simples que a altura e a largura de uma folha A(k), em metros, sero dadas, respectivamente, por 2 4 e 2 4 . Para o caso da folha A4, aplicando k = 4 na frmula, obtemos os misteriosos valores padronizados do formato, que so 210 mm por 297 mm. Qual a vantagem da proporo 1: 2 ? A literatura sobre artes grficas cita dois aspectos importantes sobre a convenincia do uso de uma folha retangular de razo 1 : 2 . As pginas de um livro so impressas em uma folha de mquina de grande formato. Nela so feitas dobras e cortes e, a partir disso, so montados os cadernos que, juntos, compem o livro. Normalmente as dobras so feitas ao meio, fazendo com que o nmero de pginas seja uma potncia de 2. Se o papel for dobrado ao meio por uma dobra, resultar em 2 folhas (chamado in-flio) que, quando impressas frente e verso, constituiro 4 pginas do livro. Se essa ltima folha for novamente dobrada ao meio, agora com dobras cruzadas, resultar em 4 folhas (in-quarto), ou seja, 8 pginas de livro. Com uma nova dobra teremos o in-oitavo: 3 dobras, 8 folhas e 16 pginas de livro; e assim sucessivamente. Uma vez que cada formato deriva do seu precedente fazendo uma dobra sobre o maior lado do retngulo, a razo inicial 1 : 2 sempre ser mantida em todas as pginas do livro, seja qual for o nmero de dobras feitas na composio. Outros formatos no permitiriam isso como, por exemplo, um retngulo de razo 3:4 (tambm usado na confeco de livros) que obedece a um padro de alternncia no decorrer das sucessivas dobras. A primeira dobra gera retngulos de razo 2:3; a
1+ 2 k 1 2 k

RPM OBMEP

77

segunda gera retngulos 3:4, a terceira retngulos 2:3, e assim sucessivamente. Deixo por conta do leitor a demonstrao do resultado: dada
y x , ento as razes se alternam entre x:y e y:2x no 2 decorrer das sucessivas dobras que dividem o lado maior do retngulo ao meio (obs.: o nico caso em que no h alternncia ser quando

a razo x:y, se

x y = , que justamente o caso em que temos a razo 1 : 2 ). y 2x


Vale citar que nem todos os estudiosos de composio em artes grficas esto de acordo sobre a relevncia da vantagem que acabamos de descrever da razo 1 : 2 sobre outras razes. Para um bom acabamento final das dobras de um livro recomenda-se que as dobras sejam feitas paralelamente s fibras do papel. Com isso, folhas de papel que, em virtude da direo das fibras, so adequadas ao in-quarto no poderiam ser usadas para livros in-oitavo porque a fibra correria em direo errada. Portanto, a vantagem da razo preservada em 1 : 2 aps as dobras fica comprometida quando levamos em considerao a direo das fibras [1]. Outra vantagem que os papis de razo 1 : 2 da srie A apresentam e essa aceita por todos os especialistas a de que evitam o desperdcio de papel nos trabalhos de fotocpias. Imagine que voc queira copiar duas folhas quadradas, juntas, em uma nova folha quadrada. Essa tarefa no pode ser realizada sem o desperdcio de papel. Se os quadrados tm lado 10 cm, lado a lado formaro um retngulo de 10 por 20 cm, o que exigir uma folha quadrada de 20 por 20 cm para que o servio seja feito. Nesse caso, haver desperdcio de metade da folha. O mesmo no ocorre, por exemplo, com duas folhas A4 lado a lado, que podem ser copiadas, sem desperdcio de papel, em uma folha A3.
RPM OBMEP

Se voc observar com ateno, as fotocopiadoras que fazem ampliao e reduo a partir das folhas da srie A possuem alguns comandos pr-definidos, como, por exemplo, os de reduo de 71%, 50%, 35%, 25%, 18% e 12,5%.

78

Voc j se perguntou de onde vm essas estranhas porcentagens? Responderemos essa pergunta calculando qual deve ser o fator de reduo usado na altura e na largura de uma folha A(k) para que ela seja reduzida a uma folha A(k +1):

2 0, 71 , uma reduo de 71% far o servio desejado. As 2 demais redues indicadas referem-se, respectivamente, s redues de A(k) para A(k + 2), A(k + 3), A(k + 4), A(k + 5) e A(k + 6).

Como

Outros formatos de papel: as sries B e C H registros do uso da razo 1 : 2 durante a Alta Idade Mdia, quando muitos livros eram escritos em duas colunas. Gutenberg (1398-1468), porm, preferia para suas pginas a razo 2:3, e, durante a Renascena, raramente se produziu livro na razo 1 : 2 . A idia de se padronizar um formato de papel surge no sculo XX, e tem a ver com aspectos relacionados praticidade e economia. Com o uso generalizado de um formato padro de papel o que se reflete diretamente na padronizao dos formatos de livros, revistas, jornais, envelopes , as bibliotecas podem planejar de forma mais eficiente as alturas de suas prateleiras, as grficas podem trabalhar com ajustes de mquina pr-definidos, as fotocopiadoras e impressoras podem padronizar programas para reduo e ampliao, etc. O padro internacional para o tamanho de papis o ISO 216 (International Organization for Standartization, norma 216), que adotado por todos os pases industrializados do mundo, exceto EUA, Canad e partes do Mxico. Essa norma regulamenta o formato de algumas sries bsicas de papel, como as sries A, B e C. As sries B
RPM OBMEP

79

e C destinam-se, entre outras aplicaes, aos formatos de envelopes que podem ser usados para conter folhas da srie A. O formato de uma folha B(k) definido como a mdia geomtrica entre A(k) e A(k 1), e o da folha C(k) como a mdia geomtrica entre A(k) e B(k). Usando a frmula que vimos anteriormente para altura de uma folha A(k), as frmulas de clculo da altura das folhas B(k) e C(k) sero:

B (k ) = A(k ) A(k 1) = 2

1+ 2 k 1+ 2 ( k 1) 4 4

= 2
1+ 4 k 4

k 22

C (k ) = A(k ) B (k ) = 2

1+ 2 k 4

k 2

= 2

=2

1+ 4 k 8

Ficam a cargo do leitor a formulao de B(k) e C(k) para a largura das folhas dessas duas sries, bem como a demonstrao de que tambm nas sries B e C a razo 1 : 2 se preserva. Seja qual for o nmero k da srie, sempre teremos, tanto para a altura quanto para a largura, a relao A(k) < C(k) < B(k). Verificaremos tal fato para a altura, cujos dados j foram calculados anteriormente:
2
1+ 2 k 4

<2

1+ 4 k 8

<2

k 2

2 + 4k 1 + 4k 4k < < 2 < 1 < 0 , 8 8 8

para qualquer k. Demonstrao anloga pode ser feita entre as larguras das trs sries. Os formatos das sries B e C so maiores que os da srie A e, por esse motivo, so usados nos envelopes que devero conter folhas da srie A. Como A(k) < C(k) < B(k), se queremos enviar pelo correio um documento com poucas folhas A4, devemos usar um envelope C4, porm, se a quantidade de folhas for muito grande, provvel que elas fiquem melhor acomodadas em um envelope B4. Se voc quiser enviar uma folha A4 dobrada uma nica vez, recomenda-se um envelope C5. Para uma folha A4 com duas dobras cruzadas, o envelope ideal o C6 e, se as duas dobras forem paralelas, o envelope ideal o DL (ilustrado na figura a seguir).

RPM OBMEP

80

Adaptado do artigo A Matemtica da folha de papel A4 Jos Luiz Pastore Mello, RPM 66.

RPM OBMEP

81

Retngulo ureo, diviso urea e sequncia de Fibonacci

O retngulo ureo Chama-se retngulo ureo qualquer retngulo ABCD (figura 1) com a seguinte propriedade: se dele suprimirmos um quadrado, como ABFE, o retngulo restante, CDEF, ser semelhante ao retngulo original. a F b C B Se a + b e a so os comprimentos dos lados do retngulo a original a definio acima se traduz na relao

a b = . (1) a+b a

E figura 1

Como veremos logo adiante, esse tipo de retngulo tem muitas propriedades interessantes que justificam o qualificativo ureo. Ele tem sido considerado por

RPM OBMEP

figura 2

arquitetos e artistas como o retngulo mais bem proporcionado e de grande valor esttico. A figura 2

82

reproduz a foto de uma residncia suburbana de Paris, projetada pelo famoso arquiteto Le Corbusier, na qual ele utiliza o retngulo ureo. H a dois retngulos ureos, um deles representado pelo corpo inteiro da casa e o outro, disposto verticalmente, representado pela parte da casa esquerda da escada.

figura 3

O Partenon (figura 3), ou templo da deusa Atena, uma das mais admiradas obras da arquitetura universal, revela, em seu frontispcio (figura 4) um quase exato retngulo ureo. Todavia no h evidencia histrica de que, ao construir o templo no 5o sculo a.C., os arquiteto de Pricles tenham conscientemente usado o retngulo ureo.

figura 4

Voltemos relao (1). Dela decorre, por uma propriedade bem conhecida das propores, que:

a b a b = = a + b a ( a + b) a

ou seja

b a b = . a b
a b
RPM OBMEP

Isto significa que se o retngulo de lados a + b e a ureo, ento tambm o o retngulo de lados a e b.

2b - a

a-b
figura 5

83

Evidentemente o mesmo raciocnio se aplica para mostrar que tambm so ureos os retngulos de lados b e a b, a b e 2b a, etc. (figura 5). Em outras palavras, dados os nmeros positivos a e b, satisfazendo a relao (1), formemos a seqncia a + b, a, b, a b, 2b a, 2a 3b, 5b 3a, 5a 8b, 13b 8a, ... (2) Sendo, a partir do terceiro, an = an2 an1. O raciocnio anterior estabelece que quaisquer dois elementos consecutivos dessa sequncia so os lados de um retngulo ureo. Portanto, o processo anterior de retirar quadrados de retngulos ureos conduz a uma sequncia infinita de retngulos ureos, com dimenses cada vez menores. A figura tambm sugere que a seqncia (2) tende a zero, e isso verdade porque as dimenses de cada retngulo da seqncia podem ser obtidas multiplicando as dimenses correspondentes do retngulo anterior por b/a, que menor que 1; ora, o termo geral de uma progresso geomtrica de razo menor do que 1 tende a zero quando o nmero de termos tende a infinito. Deve ser notado que o smbolo da Sociedade Brasileira de Matemtica utiliza a mencionada sucesso de retngulos ureos, unidas por quadrantes de circunferncias.

Os lados de um retngulo ureo so grandezas incomensurveis. (Veja a definio desse e de outros conceitos correlatos no artigo Grandezas Incomensurveis, desta apostila.) De fato, se fossem comensurveis, teriam um submltiplo comum s, e, com referencia figura 1, AD = (a + b)s e AB = as, onde a e b seriam ento nmeros inteiros. Em consequncia, todos os nmeros da sequncia (2) seriam inteiros e positivos. Isso um absurdo, pois no existe sequncia infinita e decrescente de nmeros inteiros positivos (Princpio da Descida Infinita de Fermat). Conclumos, ento, que os lados de um retngulo ureo so incomensurveis.

RPM OBMEP

84

A diviso urea O retngulo ureo est intimamente ligado com a chamada diviso urea de um segmento, ou diviso em mdia e extrema razo, que introduziremos a seguir. Diz-se que um ponto C de um segmento AB (figura 6) divide esse segmento em mdia e extrema razo se

AC CB . = AB AC

(3)

a
figura 6

A relao (3) precisamente a relao (1) se pusermos AC = a e CB = b, de sorte que os segmentos AC e CB da diviso urea (ou AB = a + b e AC = a) so os lados de um retngulo ureo. interessante notar que se C1 divide AB em mdia e extrema razo, e se marcarmos no segmento AB os pontos C2, C3, C4,... de tal maneira que AC2 = C1B, AC3 = C2C1, AC4 = C3C2, ... (figura 7), ento Cn divide ACn1 em mdia e extrema razo n = 2, 3, 4,... . Esse resultado segue facilmente do que j provamos antes sobre a sequncia infinita de retngulos ureos, donde segue tambm que os segmentos AC1 e C1B da diviso urea de AB so incomensurveis. Sugerimos que o leitor faa uma demonstrao completa destes resultados.
A C4 C3 C2
figura 7

C1

Como j observamos h pouco, as relaes (1) e (3) so idnticas quando pomos AC = a e CB = b. Delas segue-se que b2 + ab = a2. (4) O nmero m = b/a conhecido como a razo urea. Dividindo a equao anterior por a2 obtemos: m2 + m = 1. (5) A raiz positiva dessa equao do segundo grau :

M=

5 1 0, 618 . (6) 2

RPM OBMEP

85

A razo urea, como foi definida, a razo entre o menor e o maior lados de um retngulo ureo. Deve ser observado que muitos autores usam como razo urea (ou nmero ureo) o seu inverso u, que a razo entre o maior e o menor lados de um retngulo ureo:
u= 1 5 +1 = 1, 618 . m 2

O nmero u a raiz positiva da equao u2 = u + 1. Entre estes dois nmeros, existem as relaes: u = 1 + m e mu = 1. Construes geomtricas Vamos construir um retngulo ureo a partir de seu menor lado AE = a (figura 8). Para isso construmos EF = AE perpendicularmente a AE. Com centro em G, ponto mdio do segmento AE, traamos o

B a A a/2 G a/2

A
b
b

H
a-b

E
b

D
figura 9

figura 8

arco FD , com D na reta AE e E interno ao segmento AD. Como GF = GD = b + a/2, o teorema de Pitgoras aplicado ao triangulo retngulo GEF nos d: a a (b + ) 2 = a 2 + ( ) 2 . 2 2 Simplificando, obtemos daqui a relao (4) que, como vimos, equivale relao (1). Logo ABCD um retngulo ureo.
RPM OBMEP

Se o problema fosse dividir o segmento AE = EF em mdia e extrema razo, bastaria completar a construo anterior marcando, no segmento AE, o ponto H tal que AH = b (figura 9).

86

A sequncia de Fibonacci e a razo urea surpreendente que a razo urea esteja intimamente relacionada com a chamada sequncia de Fibonacci, como veremos a seguir, pois aparentemente uma coisa nada tem a ver com a outra. Leonardo de Pisa, muito conhecido como Fibonacci (filho de Bonaccio), viveu no perodo de aproximadamente 1170 a 1250. Ele foi educado na frica e viajou muito pela Europa e sia Menor. Tornou-se famoso por conhecer muito bem toda a Matemtica ento acumulada. Em 1202 ele publicou o Liber Abaci, ou Livro do Clculo, que teve importncia decisiva na tarefa de tornar conhecida na Europa a Matemtica dos rabes e hindus. Foi esse livro que popularizou no Ocidente o uso dos algarismos arbicos e os mtodos hindus de clculo com nmeros inteiros, fraes e razes. A seqncia de Fibonacci aparece num dos problemas tratados no Liber Abaci e que consiste no seguinte: Um casal de coelhos torna-se produtivo aps dois meses de vida e, a partir de ento, produz um novo casal a cada ms. Comeando com um nico casal de coelhos recm-nascidos, quantos casais existiro ao final de um ano? Vamos designar com fn o nmero de casais de coelhos existentes aps n meses. Evidentemente, f0 = f1 = 1. Por outro lado, o nmero de casais existentes no n-simo ms, fn, igual ao numero existente um ms antes, fn1, mais o numero de nascimentos novos. Ora, esse nmero precisamente o nmero de casais existentes h dois meses, fn2, que tm pelo menos dois meses de vida, portanto em condies de reproduzir. Ento, cada elemento da sequncia de Fibonacci a soma dos dois precedentes. Como j sabemos que f0 = f1 = 1, podemos construir toda a sequncia: f0 = 1, f1 = 1, f2 = f0 + f1 = 2, f3 = f1 + f2 = 3, f4 = f2 + f3 = 5, f5 = f3 + f4 = 8, f6 = f4 + f5 = 13, ... ou seja, 1, 1, 2, 3, 5, 8, 13, 21, 34, 55, 89, 144,...

RPM OBMEP

87

Que relao pode existir entre esta sequncia e a razo urea? Aparentemente nenhuma. No entanto, vamos demonstrar agora para surpresa de todos ns! ... que a razo urea m o limite, com n tende a infinito, da razo fn1/fn, isto ,
m= f 5 1 = lim n 1 . n f 2 n

(7)

De fato, pode-se observar (e demonstra-se por induo) que os coeficientes de a e tambm os de b na sequncia (2), a partir do quarto termo, a menos de sinal, so os nmeros de Fibonacci, isto , se a2 = a b, a3 = a + 2b, a4 = 2a 3b, ... ento an = (1)n(fn2a fn1b). (8) Lembrando que a sequncia (2) tende a zero quando n , tem-se: fn2a fn1b 0. Dividindo por afn1, v-se que

f b = lim n 2 . n f a n 1

A ttulo de curiosidade, vamos calcular fn1/fn, com trs casas decimais, para n = 1, 2, ..., 8:

1 1 2 3 5 = 1, = 0, 5, 0, 666, = 0, 6, = 0, 625, 1 2 3 5 8 8 13 21 0, 615, 0, 619, 0, 618. 13 21 34


Para n > 8, a razo fn1/fn sempre 0,618, com arredondamento na 3 casa decimal.
a

O pentagrama e observaes finais A diviso urea conhecida desde os pitagricos de cinco sculos a.C. Ao que tudo indica, essa diviso foi descoberta no pentgono regular, que exibe uma surpreendente profuso de segmentos na razo urea. Talvez este tenha sido o motivo que levou os pitagricos a adotarem o pentagrama (pentgono regular estrelado) como smbolo de sua seita (figura 10).

RPM OBMEP

88

Na figura, os vrtices da estrela dividem o crculo em cinco partes iguais. Portanto, cada arco mede 72. Os tringulos NBE e ABE so semelhantes porque seus ngulos internos medem 36, 72 e 72. Logo,
NB BE = . BE AB

D A M N B

E
figura 10

Mas AN = NE (porque NAE = NEA = 36 ) e NE = BE (porque


BNE = NBE = 72 ). Temos portanto,

NB BE = . AN AB

e consequentemente essas razes so ureas. Pela semelhana dos tringulos DMN e AND, prova-se, da mesma forma, que MN/AM tambm urea. muito improvvel que Pitgoras ou seus primeiros discpulos soubessem que os segmentos da diviso urea fossem incomensurveis, embora haja fundadas razes para se acreditar que a descoberta dos incomensurveis tenha ocorrido com o pentgono regular no fim do 5o sculo A.C. Certamente, Pitgoras e seus discpulos sabiam como construir geometricamente a soluo (6) da equao (5). As construes correspondentes s figuras 8 e 9 acima se encontram nos Elementos de Euclides, de cerca de 300 anos A.C. Na antiguidade, a diviso de um segmento em mdia e extrema razo tornou-se to familiar que era conhecida simplesmente como a seo, em qualquer qualificativo. O nome diviso urea lhe foi dado por Kepler (1571-1630), que escreveu: A Geometria possui dois grandes tesouros: um o Teorema de Pitgoras; o outro, a diviso de um segmento em mdia e extrema razo. Podemos comparar o primeiro a uma poro de ouro e o segundo a uma jia preciosa. Tanto a razo urea, como os nmeros de Fibonacci, aparecem numa variedade enorme de situaes inesperadas. Hoje a literatura sobre os nmeros de Fibonacci enorme. Existe at uma revista The Fibonacci

RPM OBMEP

89

Quartely fundada em 1963, dedicada pesquisa em torno desses nmeros! O Teorema de Lam Vamos mostrar agora uma aplicao surpreendente dos nmeros de Fibonacci. Para encontrar o o mximo divisor comum de dois inteiros, bastante conhecido o processo que exemplificamos no caso do mdc(243, 37):
6 243 37 21 16 1 21 5 1 16 1 3 5 0 5 1

As operaes efetuadas acima foram as seguintes: 243 = 6 37 + 21 37 = 21 1 + 16 21 = l 16 + 5 16 = 5 3+1 5 = 1 5. O algoritmo termina quando o resto da diviso nulo; o mximo divisor comum o ltimo divisor obtido. O algoritmo do processo apresentado acima o clebre algoritmo de Euclides, conhecido desde a antiguidade. Ele encontra-se exposto, para nmeros, na Proposio 1 do livro VII dos Elementos de Euclides, escritos em torno de 300 a.C. Os historiadores da Matemtica acreditam que esse algoritmo era conhecido j em 400 a.C. Ele de importncia fundamental em teoria dos nmeros. O que fizemos no exemplo acima pode ser generalizado: Sejam a e b inteiros positivos, com a > b. Usando sucessivamente o algoritmo da diviso, escreva a = bq1 + b1, 0 < b1 < b, b = b1q2 + b2, 0 < b2 < b1, b1 = b2q3 + b3, 0 < b3 < b2, ...

RPM OBMEP

90

bn2 = bnl qn + bn, 0 < bn< bn1, bnl = bnqn+l. Ento mdc(a, b) = bn. Com efeito, em primeiro lugar, o processo acima realmente chega ao fim. De fato, como 0 < bn < bn1 < ... < b1 < b, vemos que esse processo no pode repetir-se indefinidamente, pois temos uma seqncia estritamente decrescente de inteiros positivos e h um nmero finito de inteiros entre 0 e b. Alm disso, usando estas equaes de baixo para cima, constatamos que bn divide bn1, bn2, etc, at concluir que bn um divisor comum de a e b. Em seguida, usando estas mesmas equaes de cima para baixo, constatamos que todo divisor comum d de a e b tambm um divisor comum de b1 e b2, de b2 e b3, etc, at concluir que d tambm um divisor de bn, ou seja, sendo bn um mltiplo de d, bn maior que ou igual a d, o que mostra que bn o mximo divisor comum de a e b. Usando o algoritmo de Euclides, so necessrias n + 1 divises para vermos que mdc(a, b) = bn, pois s chegamos a uma concluso quando verificarmos que bn1 = bnqn+1 + bn+1 = bnqn+1 + 0 = bnqn+1. Chamaremos de comprimento do algoritmo de Euclides o nmero de divises necessrias para calcular o mdc(a, b). Usando a notao do teorema, o comprimento do algoritmo de Euclides n + 1. O algoritmo de Euclides bem eficiente. Por exemplo, se quisermos verificar que mdc(97, 24) = 1, sero necessrios apenas dois passos: 97 = 4 24 + 1 24 = 24 1. Agora, se queremos calcular mdc(21479, 24), temos
RPM OBMEP

21479 = 894 24 + 23, 24 = 1 23 + 1, 23 = 1 23. Ou seja, em 3 passos vemos que mdc(21479, 24) = 1. Por fim, como ltimo exemplo, para calcular mdc(49745692, 24), temos

91

49745692 = 2072737 24 + 4, 24 = 6 4; isto , em apenas 2 passos chegamos ao resultado desejado. Dados dois nmeros inteiros e positivos a e b, uma pergunta natural : qual o comprimento do algoritmo de Euclides aplicado a eles? Em outras palavras, quantas divises so necessrias para calcular o mximo divisor comum de a e b. imediato verificar que, se mantivermos b fixo, mesmo que a seja muito grande em relao a b, o nmero de divises no algoritmo de Euclides no pode crescer. Em verdade, esse nmero depende apenas de b. Com efeito, usando mais uma vez a notao acima, sabemos que, no algoritmo, mdc(a, b) = bn e que 0 < bn < bn1 < ... < b1 < b. Como h no mximo b 1 inteiros distintos no negativos entre 0 e b, vemos que n < b 1, donde n + 1 < b. Ora, como j vimos, so necessrias n + 1 divises para determinar o mximo divisor comum. Assim, so necessrias no mximo b divises para achar mdc(a, b). No entanto, esse resultado no muito bom. Por exemplo, se b = 99, devemos ter que n + 1 < 99 e chegamos concluso de que talvez tenhamos que efetuar 99 divises para calcular o mximo divisor comum! O Teorema de Lam melhora muito essa situao: Teorema (Lam). Sejam a e b inteiros positivos. Ento, o comprimento do algoritmo de Euclides aplicado aos nmeros a e b menor que ou igual a cinco vezes o nmero de dgitos na representao decimal de b. Segundo o teorema, se b igual a 99, que tem dois algarismos, ento o nmero de divises no algoritmo de Euclides no mximo 10, no sendo influenciado por a. Isso representa um progresso notvel em relao estimativa anterior. Esse teorema devido a Gabriel Lam (1795-1870) engenheiro e matemtico francs, conhecido por seus trabalhos sobre a equao do calor e criador das coordenadas curvilneas. Embora no tenha se dedicado sistematicamente teoria dos nmeros, ele deixou algumas jias sobre o assunto, uma das quais o teorema anterior.

RPM OBMEP

92

A demonstrao do Teorema de Lam um exemplo de utilizao inteligente dos nmeros de Fibonacci. Em verdade, essa foi, em 1844, a primeira aplicao significativa desses nmeros. Para efetuarmos a demonstrao, voltemos ao algoritmo de Euclides. Em primeiro lugar, b n > 1, pois bn um nmero inteiro. De bn1 = bnqn+1, vemos que bn1 > 2, pois bn1 > bn. Assim, bn > f1 e bn1 > f2. Ento, bn2 = bn1qn + bn > f2 + f1 = f3, pois qn > 1. Analogamente, bn3 = bn2 qn1 + bn1 > f3 + f2 = f4 , pois qn1 > 1. Continuando dessa maneira, vemos, de maneira geral, que bn-k > fk+1 para k = 0, 1, 2, ..., n 1, e, enfim, b = b1q2 + b2 > fn + fn1 = fn+1, ou seja, fazendo b0 = b, temos: bnk > fk+1, para k = 0, 1, 2, ..., n. Ilustrando:
bn > f1 bn -1 > f2 bn -2 > f3 ... ... ... b1 > fn b > fn + 1

Esse resultado nos mostra que o comprimento do algoritmo de Euclides menor ou igual ao nmero de ordem do maior nmero de Fibonacci menor ou igual a b. Podemos ver que esse resultado o melhor possvel achando o mximo divisor comum entre dois nmeros de Fibonacci consecutivos. Calculemos, por exemplo, mdc(21,13) = mdc(f7, f6): 21 = 13 + 8 13 = 8 + 5 8=5+3 5=3+2 3=2+1 2 = 1 2 + 0. Nesse exemplo, f7 e f6 no desempenham nenhum papel essencial,

RPM OBMEP

93

pois o mesmo acontece no caso geral, para achar mdc(fn1, fn). Consideremos agora o nmero ureo u =
5 +1 1, 618... . 2 Temos, como visto anteriormente, u2 = 1 + u. Logo,

u2 = u + 1 < 2 +1 < f2 + f1 = f3. u3 = u2 + u < f3 + 2 < f3 + f2 = f4, u4 = u3 + u2 < f4 + f3 = f5, e assim sucessivamente, chegando enfim a uj < fj+1, j = 2, 3, 4, ... .
Em particular, un < fn +1 < b.

Como a funo log10x estritamente crescente, temos que nlog10u < log10b, ou, equivalente, n <

log10 b . log10 u

Ora, calcula-se facilmente, usando uma tbua de logaritmos ou uma mquina de calcular, que

log10 u = log10
Assim, n <

1 1+ 5 1 < 5. = 0, 20898 > 0, 20 = , ou seja, log10 u 2 5

log10 b < 5 log10 b log10 u

Se o nmero de algarismos na representao decimal de b s, ento b = ts110s1 + ts210s2 + ... + t110 + t0, e, portanto, b < 10s, donde log10b < s, e vemos que n < 5s. Como n um inteiro estritamente menor do que 5s, temos que n + 1 < 5s, o resultado procurado.
Adaptado dos artigos Retngulo ureo, diviso urea e seqncia de Fibonacci. Geraldo vila, RPM 06.
RPM OBMEP

O smbolo da SBM Eduardo Wagner, RPM 20. Euclides, Fibonacci e Lam Joo Bosco Pitombeira de Carvalho, RPM 24.

94

Usando Geometria para somar

Introduo Um dos maiores prazeres da Matemtica o da descoberta (ou da redescoberta) de resultados matemticos mesmo que j conhecidos. Iniciamos este artigo com o problema clssico de calcular a soma dos n primeiros nmeros naturais pelo mtodo que Gauss teria utilizado aos dez anos para somar de 1 a 100 de cabea, para surpresa do seu professor. Mostraremos ainda como calcular a soma dos quadrados dos n primeiros naturais e outros resultados interessantes. Seja S = 1 + 2 + 3 + ... a soma dos n primeiros nmeros naturais. O mtodo utilizado pelo jovem Gauss para calcular essa soma bastante simples, embora engenhoso. Ele escreveu a soma pedida e, embaixo, escreveu a mesma soma ao contrrio S = 1 + 2 + (n 1) + n S = n + (n 1) + ... + 2 + 1 Somando, obtemos 2S = (n + 1) + (n + 1) + ... + (n + 1)
RPM OBMEP

n(n + 1) . 2 A demonstrao a seguir usa essencialmente a mesma ideia.


com n parcelas, ou seja, S =

95

A soma dos n primeiros nmeros naturais I. A soma dos n primeiros nmeros naturais pode ser visualizada geometricamente atravs da figura abaixo. Nela v-se um retngulo formado por bolinhas. A base do retngulo possui n + 1 bolinhas e a altura tem n bolinhas. No total, temos ento n(n + 1) bolinhas. Observe agora que elas esto divididas em duas partes iguais pela linha poligonal e em cada uma delas aparece a soma S = 1 + 2 + 3 + ... + n. Obtm-se, ento, a frmula da soma dos n primeiros nmeros naturais.

1 1 + 2 + ... + n = n(n + 1) 2

Nota da RPM: A figura acima apareceu na coluna de Martin Gardner Mathematical Games, da Scientific American de outubro de 1973, junto com vrias outras chamadas look-see diagrams (diagramas olhe-veja) e o autor cita que ela j era conhecida pelos gregos antigos. Posteriormente, figuras demonstrando resultados matemticos conhecidos apareceram em vrias revistas. Em 1993, as melhores demonstraes dessas revistas foram reunidas por Roger Nelsen e publicadas no livro Proofs without words Exercices in visual thinking pela MAA (Mathematical Association of America). Esse livro ser a nossa principal referncia neste artigo, doravante designado por [Pww] (Pww - Proofs without words Demonstraes sem palavras) II. Uma outra forma de obter a soma dos n primeiros nmeros naturais utiliza a figura a seguir e o conceito de rea. Observe que a soma 1 + 2 + 3 + ... + n igual rea do tringulo grande (metade de um Quadrado de lado n) mais a metade de n quadrados.
RPM OBMEP

1 + 2 + ... + n =

n 2 n n(n + 1) + = 2 2 2

96

A soma dos nmeros mpares A soma dos n primeiros nmeros mpares pode ser visualizada atravs da figura a seguir. O fato de que essa soma igual a n2 j era do conhecimento dos antigos pitagricos, mas a figura da autoria de Nicmaco de Gerasa (um pitagrico tardio), que viveu em torno do ano 100 d.C. [Pww].

1 + 3 + 5 + ... + (2n 1) = n 2

A soma dos quadrados dos n primeiros nmeros naturais I. Considere inicialmente trs castelos iguais como os da figura A. Cada um deles formado por 1 + 4 + 9 + ... + n2 cubos unitrios. Na figura B os trs castelos foram reunidos e nota-se que o ltimo andar possui apenas a metade dos cubos necessrios para completar um paraleleppedo.

figura A figuras de [Pww]

figura B

RPM OBMEP

97

A figura C mostra os cubos do ltimo andar cortados horizontalmente pela metade, sendo a parte de cima (mais escura) utilizada para completar o paraleleppedo. Pela figura D vemos que a soma 1 + 22 + ...+ n2, que o volume dos castelos iniciais, um tero do volume de um paraleleppedo de base n por n + 1 e altura n +

1 . Portanto, 2

1 1 n(n + 1)(2n + 1) 1 + 22 + ... + n 2 = n(n + 1)(n + ) = . 3 2 6

figura C figuras de [Pww]

figura D

A soma de uma srie geomtrica Sendo r um nmero positivo menor que 1, quanto vale a soma infinita 1 + r + r2 + r3 + ...? A resposta pode ser obtida atravs de um desenho bastante engenhoso. Na figura seguinte, ASPQ um quadrado de lado 1 e AR = r. A reta PR forma o tringulo PTS no qual ST = 1 + r + r2 + r3 + ... (Essa ltima igualdade pode ser verificada construindo-se um novo quadrado de lado r obtendo o segmento de medida r2 uma vez que a razo entre as medidas dos segmentos
RPM OBMEP

1 r . Continuando o = r r2 processo, construindo-se quadrados de lados r2, r3, ... obtemos a medida indicada para o segmento ST.)

98

Da semelhana entre os tringulos PTS e RPQ temos ou seja, 1 + r + r 2 + r 3 + ... = 1 . 1 r

ST QP , = SP QR

Para voc pensar: O que cada umas das figuras seguintes pode mostrar?

Sugesto: figura da esquerda: 4(1 + 3 + ...) figura da direita: 1 + 2.22 + 3.32 + 4.42 + 5.52 = ...

Observao certo que, as demonstraes visuais podem parecer, em um primeiro momento, ter algo de magia. importante perceber que a mgica apenas aparente: a descoberta de um resultado matemtico fruto de experimentao, dedicao e compreenso dos conceitos envolvidos. Deve ficar clara tambm a necessidade, no caso de nmeros naturais, por exemplo, do axioma da induo, para provar definitivamente certos resultados.
Adaptado do artigo Usando Geometria para somar Eduardo de Campos Valadares e Eduardo Wagner, RPM 39.
RPM OBMEP

99

Mdias

As mdias mais conhecidas pelos estudantes e professores de Matemtica so a mdia aritmtica, a mdia geomtrica e a mdia harmnica. Para dois nmeros a e b, estas mdias so, respectivamente:

A=

a+b 2

G = ab

H=

2ab . a+b

Para calcular a mdia geomtrica G, costuma-se exigir que a e b sejam positivos e, para calcular a mdia harmnica H, exige-se que a e b sejam no nulos. De agora por diante, consideraremos apenas nmeros positivos. A mdia geomtrica tambm pode ser escrita como: G2 = ab ou a = G . A mdia harmnica tambm pode G b ser vista como o inverso da mdia aritmtica dos inversos, isto : 1 = 1 a + 1 b . H 2
RPM OBMEP

Decorre imediatamente das definies que AH = G2, o que mostra que a mdia geomtrica de dois nmeros tambm a mdia geomtrica entre a mdia aritmtica e a mdia harmnica destes nmeros.

100

Para ter uma viso unificada destas trs mdias, considere as relaes seguintes envolvendo os nmeros reais a, b e c, positivos e distintos:
ac a = cb a ac a = cb b ac a = cb c

(1) (2) (3)

Estas equaes diferem apenas nos segundos membros: na equao (1) o denominador do quociente a, na (2) b, e na (3) c. Isolando c na equao (1), obtemos c = (a + b)/2, ou seja, c a mdia aritmtica de a e b; isolando c em (2), obtemos c = 2ab/(a + b), ou seja, c a mdia harmnica de a e b; isolando c em (3), obtemos c = ab , ou seja, c a mdia geomtrica de a e b. Por exemplo, para os nmeros 2 e 18, temos:

A=

2 + 18 = 10 2

G = 2.18 = 6

H=

2.2.18 = 3, 6 . 2 + 18

Neste exemplo, observa-se que qualquer destas mdias est entre o menor e o maior dos nmeros. Mais ainda, no exemplo: 2 < 3,6 < 6 < 10, ou seja a < H < G < A < b. Vamos mostrar agora que este fato geral, isto : Dados os nmeros positivos a e b, com a < b, tem-se: a < H < G < A < b. Alm disso, se a < b, ento: a < H < G < A < b. De fato, se a = b, ento A = isto , a = H = G = A = b. Por outro lado, se a < b, temos, sucessivamente: a + b < b + b = 2b; a(a + b) < 2ab; a <
a+a 2aa = a; G = a.a = a; H = = a, 2 a+a
RPM OBMEP

2ab , isto : a < H. a+b

101

2 (a b)2 = a2 2ab + b2 > 0; a2 + 2ab + b2 > 4ab; (a + b) >

4a 2 b 2 ; ab

ab >

4a 2 b 2 ( a + b)
2

ab >

2ab , ou seja: H < G. a+b ab < a+b , ou seja: G < A. 2

0 < ( a b )2 = a + b 2 a b ;

a + b < b + b; a + b < 2b;

a+b < b , isto : A < b. 2

possvel tambm visualizar geometricamente essas desigualdades. Para isso, como na figura 1, colocamos consecutivamente numa mesma reta os segmentos PQ = a e QS = b, com Q entre P e S. Com centro no ponto mdio M de PS, construmos uma semicircunferncia K, e os segmentos MT e QD, perpendiculares a PS, com T e D em K. T D K A H E G
P
a

Q
b

figura 1

Construimos tambm o segmento DM e o ponto E, projeo ortogonal de Q sobre DM. Como a + b dimetro e MT o raio de K, ento MT a mdia aritmtica de a e b, isto , MT = A. Alm disso, o tringulo PDS retngulo em D, por estar inscrito em uma semicircunferencia de dimetro PS; logo, a altura QD a mdia geomtrica de PQ = a e QS = b, ou seja: QD = G.
RPM OBMEP

Finalmente, no tringulo retngulo DQM, DE a projeo ortogonal do cateto DQ sobre a hipotenusa DM = A como DE; logo: DQ2 = DM.DE, isto : DE = DQ2/DM = G2/A = H. Logo: DE = H a mdia harmnica de a e b.

102

Na figura 1, pode-se verificar que H < G < A. De fato, H < G porque H cateto e G hipotenusa no tringulo DEQ, enquanto G < A porque G cateto e A hipotenusa no tringulo DQM. Note que quando a = b, o ponto Q coincide com M. Neste caso, H = G = A, como era de esperar. Outra maneira de visualizar as desigualdades entre as mdias considerar um trapzio com bases a e b. a

b
figura 2

Observamos que conforme um segmento paralelo s bases a e b, com extremidades nos dois lados transversos, caminha se afastando de a e se aproximando de b, sua medida assume todos os valores entre a e b, e, consequentemente, todas as mdias consideradas. Veremos a seguir que os valores A, G e H aparecem conforme o segmento paralelo s bases assume alguma posio notvel no trapzio. 1) Suponhamos o segmento de medida m equidistando das bases. a
m
h h

b figura 3

A soma das reas dos dois trapzios menores a rea do trapzio inicial:

(a + m)h (m + b)h (a + b)2h o que implica m = a + b = A . + = 2 2 2 2

103

RPM OBMEP

2) Consideremos agora que o segmento de medida m divida o trapzio inicial em dois trapzios semelhantes. a
m A b
h1 h2

figura 4 Temos

a m = , implicando m = ab = G . m b
h a = 1 , pois os trapzios so semelhantes, ab h2

Note que, sendo

temos h1 < h2, pois a < ab , e, por isso, h1 < h. Logo, m = G < A. 3) Finalmente, consideremos o segmento de medida m passando pelo encontro das diagonais. a

h3 h4

b
figura 5

Por semelhana de tringulos temos:

h3 h4 h4 x x y = = ; ; e = , a h4 + h3 b h4 + h3 a h4 + h3
RPM OBMEP

implicando x = y e

x y + = 1. a b

104

Substituindo x = y na ltima igualdade, encontramos x = portanto, m = 2 x =

2ab =H . a+b

ab e, a+b

Ainda, usando semelhana de tringulos, temos

h a a a h h = 1 e , ou seja, 3 < 1 e como h3 + h4 = h1 + h2 < b ab h2 h4 h2 ab

a h3 = . Mas b h4

tem-se h3 < h1 e, portanto, H < G. Caso permitssemos que as bases do trapzio se igualassem, ou seja, ter a = b, o trapzio se transformaria num paralelogramo e assim, obviamente, as trs mdias se igualariam a a e b. Onde aparece a mdia harmnica So inevitveis as perguntas pragmticas que alunos e professores costumam fazer: Para que serve o estudo da mdia harmnica? Onde se aplica a mdia harmnica? Sem a pretenso de responder cabalmente a essas perguntas, vou apenas salientar a importncia da mdia harmnica, assinalando a sua presena em alguns problemas da vida prtica. O problema das velocidades O sr. Mrio, um imprudente vendedor de filtros de gua, costuma acordar cedo e viajar de carro, da cidade A at a cidade B, com a velocidade mdia de 120 km/h. Depois de visitar seus clientes e tomar com eles algumas garrafas de cerveja, ele volta de B para A, com a velocidade mdia de 60 km/h. Qual a velocidade mdia que o sr. Mrio desenvolve no percurso todo? A resposta mais imediata que surge em nosso crebro que a velocidade mdia no percurso todo a mdia aritmtica das velocidades na ida e na volta, o que daria 90 km/h. Essa resposta, embora intuitiva, est errada! Temos que estar sempre alertas, maneira dos escoteiros, para no deixar a razo matemtica ser desgovernada por falsas intuies.

105

RPM OBMEP

A resoluo correta do problema a seguinte. Sejam: d: a distncia entre as cidades A e B v1: a velocidade mdia na ida v2: a velocidade mdia na volta t1: o tempo de viagem na ida t2: o tempo de viagem na volta

Temos ento que d = v1t1 = v2t2. Se v a velocidade mdia no percurso todo, temos: 2d = v(t1 + t2). Logo, 2d = v(d/v1 + d/v2). Simplificando: v = 2v1v2/(v1 + v2). Substituindo os valores v1 = 120 km/h e v2 = 60 km/h, obtemos v = 80 km/h. Moral da histria: a velocidade mdia no percurso todo a mdia harmnica das velocidades na ida e na volta. A mdia harmnica geralmente aparece em problemas que envolvem velocidades, vazes, frequncias e taxas. O exemplo seguinte uma verso simples de um problema de vazo bastante conhecido. O problema das torneiras Se uma torneira enche um tanque em 60 minutos e uma outra torneira enche o mesmo tanque em 30 minutos, em quanto tempo as duas torneiras juntas enchem o tanque? Os leitores esto convidados a resolver mais esse problema, e para isso damos uma pequena dica: a resposta no a mdia harmnica de 60 min e 30 min, mas est relacionada a ela. Problemas de torneiras so antiqussimos. Uma de suas verses aparece por exemplo na Antologia grega organizada por Metrodoro, um matemtico grego que vivia por volta do ano 500 depois de Cristo. A traduo para o portugus seria mais ou menos a seguinte: Eu sou um leo de bronze; de meus olhos, boca e p direito jorra gua. Meu olho direito enche uma jarra em dois dias, meu olho esquerdo em trs dias, e meu p direito em quatro dias. Minha boca capaz de ench-la em seis horas, diga-me quanto tempo os quatro juntos levaro para ench-la? Para finalizar esta seo, mais um problema.

106

RPM OBMEP

O problema do usque Durante 4 meses consecutivos, o sr. Mrio comprou usque para o bar de sua casa aos preos, respectivamente, de 16, 18, 21 e 25 reais por garrafa. Qual foi o custo mdio do usque para o sr. Mrio nesse perodo todo? Esse um daqueles problemas que nos deixam frustrados, pois s depois de muita batalha notamos que faltam dados; temos necessariamente que introduzir alguma hiptese para poder resolver o problema. (i) Uma hiptese plausvel que, talvez por ser um bebedor regular, o sr. Mrio tenha comprado a mesma quantidade x de usque a cada ms. Logo, ele despendeu 16x + 18x + 21x + 25x = 80x reais para comprar usque no perodo. Da, o custo mdio no perodo de 4 meses foi de 80x/4x = 20 reais por garrafa. Portanto, caso essa hiptese seja verdadeira, o custo mdio no perodo a mdia aritmtica dos custos mensais. (ii) Uma outra hiptese plausvel que, talvez por no ter tido aumento de salrio nesse perodo, o sr. Mrio tenha gasto a mesma quantia y de reais a cada ms. Logo, ele consumiu y/16 + y/18 + y/21 + y/25 garrafas no perodo. Assim, o custo mdio nesse perodo foi, aproximadamente: 4y/(y/16 + y/18 + y/21 + y/25) = 19,5 reais por garrafa. Portanto, neste caso, o custo mdio no perodo a mdia harmnica dos custos mensais. Mdias para mais de dois nmeros As mdias que vimos para dois nmeros podem ser generalizadas para mais nmeros. Vamos fazer isto aqui somente para as mdias aritmtica e geomtrica.
RPM OBMEP

Dados n nmeros positivos x1, x2, ... , xn, definimos.


A= x1 + x2 + ... + xn e G = n x1 x2 ... xn . n

107

Deixamos para o leitor verificar que, se todos os nmeros forem iguais a um valor v, ento A = G = v. Porm, se eles no forem todos iguais, a mdia geomtrica sempre menor que a mdia aritmtica, mas a demonstrao, nesse caso, no to fcil como no caso n = 2. Existem demonstraes de vrios tipos, de diversos graus de sofisticao e baseadas em diferentes teorias. A mais conhecida a demonstrao de Cauchy (1789-1857), que pode ser encontrada no livro Meu Professor de Matemtica e outras histrias de Elon Lages Lima, p. 153, publicado pela SBM. Vamos dar aqui a demonstrao concebida por Polya, que se baseia na desigualdade ex > 1 + x, justificada a seguir: Observemos que se pode definir o logaritmo (natural) de um nmero positivo a como sendo a rea limitada pelo eixo das abscissas, pela curva y = l/x e pelas y=1 x retas verticais x = 1 e x = a. Como essa regio est contida no retngulo de altura 1 e base a 1, temos, claro, ln a < a 1. 1 Fazendo a = 1 + x, obtemos ln(1+ x) < x ou lna ex > 1 + x, valendo a igualdade apenas se a 1 a = 1, ou seja, x = 0. Resolvida essa parte, podemos ento demonstrar a desigualdade das mdias para n nmeros. Na desigualdade ex > 1 + x vamos substituir x por i = 1, 2, ..., n, obtendo as relaes

xi 1 , com A

eA

x1 1

x1 A x2 A xn A
x1 x2 ... xn An

eA
RPM OBMEP

x2 1

xn 1 eA

Multiplicando, obtemos

x1 + x2 +...+ xn n A e

108

ou A > G. An como queramos demonstrar. claro que a igualdade vale se, e somente se,
xi 1 = 0 ou seja, xi = A para todo i = 1, 2, ..., n. A

Mas, x1 + x2 + ... + xn = nA, logo 1

Gn

Aplicaes das desigualdades das mdias

1 2. x Soluo: Aplicando a desigualdade das mdias aos nmeros x e l/x,


Exemplo 1: Mostre que se x > 0, ento: x +

x+
obtemos

1 x x. 1 = 1 ou seja, x + 1 2 , ocorrendo a igualdade x 2 x

se e somente se, x = 1. Exemplo 2 Para todos os valores das variveis x, y, z, w, reais positivas, qual o menor valor da expresso
E= x y z w + + + y z w x ?

Soluo:

x y z w + + + x y z w y z w x 4 . . . =1 4 y z w x
Logo, E > 4, ocorrendo a igualdade se x = y = z = w. Exemplo 3
2 Para x > 0, qual o valor mnimo de y = x +

1 ? x
RPM OBMEP

Soluo: Escrevemos y = x 2 +

1 1 . Desta forma temos + 2x 2x

109

x2 +

1 1 + 2 x 2 x 3 x2 . 1 . 1 = 3 1 . 3 2x 2x 4

Portanto, y

x=

1 . 2

3 1 , ocorrendo a igualdade quando x 2 = , ou seja, 2x 4

Exemplo 4 Se x, y e z so positivos, qual o valor mnimo de

1 1 1 ( x + y + z )( + + ) ? x y z
Este exemplo ser deixado como exerccio para o leitor. A resposta 9. A desigualdade entre as mdias aritmtica e geomtrica tem como consequncia as seguintes afirmaes: I) Se a soma de n nmeros positivos for constante, ento o produto ser mximo quando todos os nmeros forem iguais. II) Se o produto de n nmeros positivos for constante, ento a soma ser mnima quando todos os nmeros forem iguais. Daremos mais dois exemplos para mostrar como funciona a afirmao I). Exemplo 5 Sendo x e y nmeros reais positivos, determinar o mximo de E = xy(1 x y). Soluo: Consideremos apenas os valores de x e y tais que x + y < 1 (se x + y > 1, teremos 1 x y < 0 e o mximo que estamos procurando obviamente positivo). Ento, os nmeros x, y e 1 x y so positivos e possuem soma igual a 1. Logo, o produto ser mximo quando todos forem iguais ou seja,

RPM OBMEP

1 1 1 1 . Emax = . . = 3 3 3 27

110

Exemplo 6 Provar que, de todos os tringulos de mesmo permetro, o equiltero possui a maior rea. Soluo: Consideremos um tringulo de lados a, b e c com a + b + c = 2p. A rea S desse tringulo dada pela frmula de Heron,
S= p.( p a )( p b)( p c)

Para poder aplicar a afirmativa I, devemos escrever


S= p . ( p a )( p b)( p c)

Ora, o semipermetro p constante. Ento S ser mximo quando (p a)(p b)(p c) for mximo. Mas p a + p b + p c = 3p 2p = p (constante) logo o produto ser mximo quando

p a = p b = p c = a=b=c=

p , ou seja 3

2p como queramos demonstrar. 3


Adaptado dos artigos Duas mdias
Eduardo Wagner, RPM 18.

Mdia harmnica
Seiji Hariki, RPM 32.

Uma aula sobre mdias


Chico Nery, RPM 68.

111

RPM OBMEP

Problemas diversos resolvidos com Geometria Analtica

A Geometria Analtica, ou melhor o mtodo das coordenadas, uma ferramente til para resolver problemas diversos, mesmo aqueles que no contm equaes ou coordenadas. interessante observar problemas que permitem a introduo de um sistema adequado de coordenadas e conseguir assim, solues simples e convincentes. A seguir, mostraremos alguns problemas desse tipo. Para cada um deles outras formas de resoluo so possveis, mas o mtodo das coordenadas uma boa opo. Problema 1 Um pesado caminho parte ao meio dia da cidade A para a cidade B viajando com velocidade constante de 40 km/h, e s 6 horas da tarde chega cidade B. Um automvel parte da cidade B s 2 horas da tarde desse dia e, viajando com velocidade constante pela mesma estrada, chega cidade A tambm s 6 da tarde. Perguntase em que momento o caminho e o automvel se cruzaram na estrada.
RPM OBMEP

Soluo A distncia entre as cidades A e B, ao longo da estrada de 6 40 = 240 km. Vamos introduzir o seguinte sistema de coordenadas: para um objeto qualquer que se mova

112

ao longo da estrada, seja x o tempo (em horas) decorrido aps o meio dia e seja y (em quilmetros) a sua distncia cidade A. Os grficos correspondentes aos movimentos do caminho e do automvel so retas, uma vez que eles viajam com velocidades constantes. De acordo com os dados do problema, o grfico que mostra o movimento do caminho um segmento de reta cujos extremos so os pontos (0, 0) e (6, 240), e o grfico que mostra o movimento do automvel um segmento de reta cujos extremos so os pontos (2, 240) e (6, 0). As equaes das retas do grfico abaixo so y = 40x e y = 60x + 360. Resolvendo o sistema encontramos x = 3,6 e y = 144. Conclu- 240 mos ento que o encontro se deu 3,6 horas aps o meio dia, ou seja s 3 horas e 36 minutos da tarde e, nesse momento, ambos estavam a 144 km da cidade A.
y

Esse problema nada tem de original, mas serve para ilustrar que diversos problemas de cinemtica escalar podem ser resolvidos com o mtodo analtico. Problema 2 Considere todos os nmeros reais x e y tais que x + 2y = 10. Para que valores de x e y a expresso E = x2 + y2 assume menor valor? Soluo Este um problema de lgebra. Sua soluo, uma vez que o enunciado esteja bem entendido, no difcil. Entretanto, a soluo analtica interessante. Estabelecendo um sistema de coordenadas, todos os pontos P(x, y) tais que x + 2y = 10 pertencem a uma reta r e o valor de E o quadrado da distncia de P origem do sistema de coordenadas.
y s

P
r
RPM OBMEP

113

Precisamos ento encontrar o ponto de r cuja distncia ao ponto (0, 0) mnima. A reta s, perpendicular a r e passando pela origem tem equao 2x y = 0 e a interseo dessas retas o ponto que procuramos. Resolvendo o sistema formado pelas duas equaes encontramos x = 2 e y = 4 que a soluo do problema. Conclumos ainda que o valor mnimo de E 22 + 42 = 20. Problema 3 Na molcula do metano (CH4) o tomo de carbono ocupa o centro de um tetraedro regular em cujos vrtices esto os tomos de hidrognio. Determine o ngulo entre duas das valncias do carbono. Soluo O resultado deste problema est presente em todos os cursos de qumica orgnica. O estranho nmero fornecido aceito por todos, mas, em geral, no se tem a menor idia de como esse resultado foi obtido. Para calcular esse ngulo, a Geometria Analtica um mtodo imbatvel, aliada claro, com alguma inventividade. Vamos utilizar um sistema de coordenadas no espao e usar a frmula que fornece o cosseno do ngulo entre dois vetores u e v :

cos =

u v . Consideremos inicialmente um cubo de aresta 2 (para | u || v |

facilitar) com um vrtice na origem, outro no eixo X, outro no eixo Y e outro no eixo Z. No difcil escolher quatro vrtices deste cubo que formem um tetraedro regular. Os pontos A = (0, 0, 0), B = (2, 2, 0), C = (0, 2, 2) e D = (2, 0, 2) formam um tetraedro regular (uma vez que as distncias entre dois quaisquer deles so diagonais de faces do cubo) e so ocupados pelos hidrognios.
RPM OBMEP

O ponto P = (1, 1, 1), centro do cubo e tambm centro do tetraedro, est ocupado pelo carbono. O resto fcil. Para calcular, por exemplo o ngulo APB, consideremos os vetores u = PA = (1, 1, 1) e v = PB = (1,1, 1) .

114

O cosseno do ngulo entre eles :


cos = 1 1 + 1 1 = . 3 3 3

C D P B

Com uma calculadora, determinamos um valor muito aproximado para esse ngulo: = 1092816,395".

Vamos, nos prximos exemplos, resolver vrios problemas de Geometria Plana, usando Geometria Analtica. Naturalmente, todos os problemas apresentados podem ser resolvidos utilizando-se geometria sinttica. Sugerimos que os leitores tentem obter essas solues. Problema 4 O quadrado ABCD tem lado 10. Sendo M o ponto mdio de BC, trace DP perpendicular a AM. Qual o comprimento do segmento DP? Soluo A soluo pela geometria sinttica passa pela descoberta que os tringulos ABM e DPA so semelhantes. Com isso, e mais uma aplicao do teorema de Pitgoras se resolve o problema. Entretanto, com os recursos da Geometria Analtica, a soluo no depende da descoberta dessa semelhana. Podemos escolher um sistema de coordenadas com o eixo X passando por AB e com o eixo Y passando por AD. Na figura ao lado temos: A = (0, 0), B = (10, 0), C = (10, 10), D = (0, 10) e M = (10, 5). A equao da reta AM x 2y = 0 e o comprimento do segmento DP a distncia do ponto D reta AM. A distncia do ponto (x0, y0) reta ax + by + c = 0 dada por: d= Temos, ento, DP = | ax0 + by0 + c | . a 2 + b2
D C M P A B
RPM OBMEP

| 0 2.10 | 12 + (2) 2

20 =4 5. 5

115

Problema 5 Na figura, o quadrado ABCD tem lado 9 e os pontos P e Q dividem o lado CD em trs segmentos congruentes. Calcule a distncia do vrtice A ao baricentro G do tringulo BPQ. Soluo

G D P Q C

Encaixemos o quadrado ABCD no primeiro quadrante do plano cartesiano, com o vrtice D coincidindo com a origem. Sendo A= (0, 9), B = (9, 9), P = (3, 0) e Q = (6, 0), conhecido que as coordenadas do baricentro G so:
y

9+3+ 6 = 6, 3 3 yB + yP + yQ 9 + 0 + 0 yG = = = 3, 3 3 portanto, G = (6, 3). xG = =

xB + xP + xQ

G D P Q C
x

A distncia procurada :

d AG = ( xG x A ) 2 + ( yG y A ) 2 = (6 0) 2 + (3 9) 2 = 72 = 6 2 .

Problema 6 As medianas AM e BN de um tringulo ABC so perpendiculares e medem, respectivamente, 9 cm e 12 cm. Calcule o comprimento da terceira mediana desse tringulo. Soluo O encontro das medianas o baricentro G do tringulo ABC. Usando o fato de que as medianas AM e BN se cortam perpendicularmente em G, coloquemos esse tringulo no plano cartesiano

A N

G B
y

M
6 A

RPM OBMEP

B
-8

G
-3

N M
4

116

com origem em G. Usando a conhecida proporo em que G divide as medianas, temos:


AM = 9 AG = 6 e GM = 3 , ou seja, A = (0, 6), M = (0, 3), BN = 12 BG = 8 e GN = 4

B = (8, 0) e N = (4, 0). As equaes segmentrias das retas AN e BM so, respectivamente,


x y + =1 e 4 6

x y + = 1. 8 3

Resolvendo o sistema anterior, encontramos o ponto C = (8, 6). O comprimento da terceira mediana 3/2 da distncia entre C e G:

3 3 ( xC xG ) 2 + ( yC yG ) 2 = (8 0) 2 + (6 0) 2 = 15 cm . 2 2
Problema 7 Calcule a rea do tringulo ADE, retngulo em E, inscrito num trapzio retngulo ABCD, com AB = 10 cm, AD = 30 cm e CD = 20 cm (figura). Soluo Encaixemos o trapzio ABCD no primeiro quadrante do plano cartesiano, fazendo os lados AD e AB ficarem contidos, respectivamente, nos eixos x e y. Como a reta BC tem coeficiente angular m =
yC yB 20 10 1 = = e xC xB 30 0 3
y

C B A E

C E
20

B
10

M
15

D x

coeficiente linear 10, sua equao reduzida

circunferncia de dimetro AD, com centro M = (15, 0), passa pelo ponto E e tem equao: (x 15)2 + y2 = 152.

117

RPM OBMEP

1 y = x + 10 . A 3

1 y = x + 10 O ponto E dado pela soluo do sistema: , 3 ( x 15) 2 + y 2 = 225


ou seja, E = (6, 12) ou E = (15, 15). Portanto, a rea do tringulo ADE 0 0 1 0 0 1 1 1 2 = 30 0 1 = 180 cm ou 30 0 1 = 225 cm 2 . 2 2 6 12 1 15 15 1

: S ADE

Adaptado dos artigos Sobre o ensino de Geometria Analtica Eduardo Wagner, RPM 41 A Geometria Analtica no ensino mdio Chico Nery, RPM 67

118

RPM OBMEP

A sombra do meu abajur

Introduo A fotografia abaixo reproduz o abajur do meu quarto e a sombra que ele projeta na parede. Que curvas so essas?

A cpula do abajur um tronco de cone, com altura h e raios das bases R e r. A lmpada centralizada, de modo a ficar no eixo do tronco de cone. Para simplificar, podemos imaginar a lmpada concentrada em um ponto, situado a uma distncia b da base inferior e a uma distncia c da base superior do tronco, sendo b + c = h (ver figura 1).
RPM OBMEP

figura 1

119

A funo da cpula barrar uma parte dos raios de luz, evitando que a luz atinja diretamente a vista. Os raios de luz que escapam dessa barragem formam um par de cones, ambos com vrtice na lmpada (ver figura 2). Devemos imaginar esses cones prolongados para alm das bases da cpula, um para cima e outro para baixo.

figura 2

Equacionamento e resoluo do problema Para descobrir a natureza da sombra do abajur, vamos cuidar primeiro da parte superior da sombra, que a interseo da parede com o cone de luz superior. Esse cone fica caracterizado pelo ngulo da figura 3, que define sua abertura e tal que: m = tg = r/c.

figura 3

120

RPM OBMEP

Criemos um sistema de coordenadas em trs dimenses OXYZ, de modo que a origem O do sistema esteja sobre a lmpada (concebida como um ponto) e o eixo do cone coincida com o semi-eixo positivo OZ, como na figura 4.

figura 4

Nessa figura, vemos que um ponto genrico P = (x, y, z) pertence superfcie do cone se e s se, enquanto ele distar z = AP do plano XOY, sua distncia PB ao eixo permanecer igual a OA = x 2 + y 2 . Porm, a figura 5, vista no plano AOB, mostra que: PB/PA = tg = m, ou seja: PB = mPA = mz. Logo, a equao do cone : mz = x 2 + y 2 .

figura 5

A parede um plano paralelo ao eixo do cone. Podemos ajustar os eixos OX e OY de modo que o plano XOZ fique paralelo parede. Nesse

121

RPM OBMEP

caso, a parede tem equao y = d, onde d a distncia da lmpada parede. Finalmente, a curva que procuramos a interseo do cone de equao
mz = x 2 + y 2 com o plano de equao y = d, isto , o conjunto dos
2 2 pontos (x, y, z) que so solues do sistema: mz = x + y . y = d 2 2 Esse sistema obviamente equivalente ao sistema: mz = x + d , y = d

e, se olharmos esses pontos no plano y = d, poderemos ficar somente com a equao mz = x 2 + d 2 , que a equao dessa curva plana nesse plano. Lembrando que m = r/c, essa equao fica: z =

c 2 x + d2 . r b x2 + d 2 . R

Para a parte inferior da sombra, um raciocnio inteiramente anlogo concluiria que a equao dessa parte da sombra : z =

Para o abajur do meu quarto, essas dimenses so, aproximadamente: r =10 cm, R = 25 cm, b = 10 cm, c = 20 cm e (quando o abajur est no seu lugar mais usual) d = 40 cm. Nesse caso, as equaes da sombra so, em centmetros:

z = 2 x 2 + 1600
e z = 0, 4 x 2 + 1600 .
RPM OBMEP

A figura 6 mostra esses grficos obtidos por um programa de computador.


figura 6

122

Identificao das curvas Essas curvas que obtivemos so uma novidade ou ser que j as vimos por a no ensino mdio? Na verdade, elas so velhas conhecidas. A sombra superior tem equao z =

c 2 x + d 2 . Elevando essa equao ao r


z2

quadrado e manipulando, obtemos:

( )
cd r

x2 d2

= 1 , que a equao de

uma hiprbole com centro na origem do plano XZ, eixo transverso sobre o eixo Z, de comprimento comprimento 2d. Como a equao z =
c 2 x + d 2 equivalente ao sistema, r

2cd , e eixo no transverso sobre o eixo X, de r

z2 x2 2 =1 cd 2 d r , v-se que a sombra superior o ramo positivo dessa z>0

( )

hiprbole. Analogamente, a sombra inferior o ramo negativo da hiprbole de equao:

z2 ( bd ) 2 R

x2 d2

= 1.

interessante observar que, quando a lmpada se situa exatamente

c b = r R (verifique!), de modo que a sombra superior e a sombra inferior so os dois ramos de uma mesma hiprbole.
no encontro das diagonais do trapzio da figura 1, ento
RPM OBMEP

O leitor pode verificar tambm que, se o abajur for cilndrico, as duas partes da sombra sero tambm os dois ramos de uma mesma hiprbole. Mais ainda: se o cilindro for eqiltero (altura igual ao dimetro da base) e a lmpada estiver centralizada, a resultante hiprbole

123

ser equiltera (eixos transverso e no transverso de mesmo comprimento). Comentrios 1. No deve ser novidade para muitos leitores que a interseo de um cone e um plano possa ser, em certos casos, uma hiprbole. Na verdade, o nome cnicas vem justamente do fato de que a seo de um cone de duas folhas por um plano , em geral, uma elipse, uma parbola ou uma hiprbole, conforme o plano seccionador forme com o eixo do cone um ngulo maior, igual ou menor do que o ngulo que a geratriz do cone forma com o seu eixo. No caso em questo, o plano da parede paralelo geratriz do cone, formando portanto um ngulo nulo, menor do que o ngulo que a geratriz do cone forma com o seu eixo. Esse teorema j era conhecido por Apolnio de Perga (sc. III a.C.) e j foi algumas vezes citado na RPM. 2. Um tema-chave no ensino mdio o ensino de funes (reais de uma varivel real) e seus grficos. Dentre essas, ningum pode negar a grande importncia das funes polinomiais de 1o e 2o graus, das funes logaritmo e exponencial e das funes trigonomtricas. Muitas vezes, porm, queremos sair um pouco da rotina e apresentar outras funes que tenham uma definio simples e estejam ligadas a aplicaes prticas. Est a um interessante exemplo: as funes da forma f ( x) = k x 2 + d 2 , cujo grfico um ramo de hiprbole. Alis, uma boa ocasio para pensar tambm nos grficos de funes da forma f ( x) = k x 2 d 2 ou f ( x) = k d 2 x 2 , cujos grficos so tambm partes de cnicas (desafio: quais?). Adaptado do artigo
RPM OBMEP

A sombra do meu abajur


Jos Paulo Q. Carneiro, RPM 59.

124

A ilha do tesouro
Dois problemas e duas solues

Problema 1 O problema a seguir foi inspirado numa histria do livro Um, dois, trs, ..., infinito de George Gamow. Era uma vez dois irmos aventureiros que encontraram, no ba das lembranas de seu bisav, o mapa de um tesouro, juntamente com as instrues para localiz-lo. O tesouro estava numa ilha, cuja localizao estava descrita de forma clara; encontrada a ilha, deveriam procurar um campo aberto com um grande espao arenoso, perfeitamente circular. No exterior do dito crculo encontrariam numerosas palmeiras alinhadas ao longo de uma reta. Deveriam, ento, procurar a palmeira com um desenho geomtrico no seu tronco e, partindo de sua base, traar as tangentes pista circular, chamando de T1 e T2 os pontos de tangncia. A seguir, deveriam traar tambm o dimetro, AM, da circunferncia fronteira da clareira, perpendicular reta das palmeiras. Encontrariam o tesouro enterrado exatamente no ponto de interseco de AM com T1T2. Os jovens viajaram muito contentes at a ilha, levando cordas e outras ferramentas necessrias. L estavam a formosa plancie, a grande clareira circular e a comprida fila de belas palmeiras. Mas todas as palmeiras

125

RPM OBMEP

apresentavam figuras geomtricas nos seus grossos troncos! Esse inesperado fato derrubou todos os planos. No sabiam qual era o ponto inicial e, sem ele, imaginaram que o trabalho seria gigantesco ou impossvel. Dessa forma tiveram de voltar com as mos vazias Entretanto, se aqueles aventureiros soubessem um pouco de Geometria, teriam escolhido uma palmeira qualquer da fila, como ponto inicial, e teriam encontrado o tesouro. Vejamos por qu.

Na figura: O e r so, respectivamente, o centro e o raio da circunferncia fronteira da clareira circular; H o ponto de interseco da reta determinada por AM com a reta das palmeiras; P o ponto que representa a palmeira escolhida, eleita para iniciar a procura do tesouro; B o ponto de interseco de OP com T1T2; T interseco de T1T2 com AM, ponto onde deveriam cavar para encontrar o tesouro. Temos ento: Os tringulos retngulos OBT1 e OT1P so semelhantes. Logo,
OT1 OP , ou seja, r 2 = OB.OP . = OB OT1

RPM OBMEP

Analogamente, os tringulos retngulos OBT e OHP so semelhantes, o que implica:

OT OB = , ou seja, OT .OH = OB.OP . OP OH

126

r2 , o que mostra que a posio OH do ponto T independe do ponto P, ou seja, independe da palmeira escolhida inicialmente.

Assim, OT .OH = r 2 ou OT =

Problema 2 O problema a seguir foi inspirado em um exerccio do livro Polynomials, de E. J. Barbeau, e foi apresentado a professores do ensino mdio, alunos de um curso, de formao continuada, sobre nmeros complexos. Dois piratas decidem enterrar um tesouro em uma ilha. Escolhem, como pontos de referncia, uma rvore e duas pedras. Comeando na rvore, medem o nmero de passos at a primeira pedra. Em seguida, dobram, segundo um ngulo reto, direita e caminham o mesmo nmero de passos at alcanar um ponto, onde fazem uma marca. Voltam rvore, medem o nmero de passos desde a rvore at a segunda pedra, dobram esquerda, segundo um ngulo reto, e caminham o mesmo nmero de passos at alcanar um ponto, onde fazem outra marca. Finalmente, enterram o tesouro exatamente no ponto mdio entre as duas marcas. Anos mais tarde, os dois piratas voltam ilha e decidem desenterrar o tesouro, mas, para sua decepo, constatam que a rvore no existe mais (o vento, a chuva e os depredadores a haviam arrancado). Ento um dos piratas decide arriscar. Escolhe ao acaso um ponto da ilha e diz: Vamos imaginar que a rvore estivesse aqui. Repete ento os mesmos procedimentos de quando havia enterrado o tesouro: conta os passos at a primeira pedra, dobra direita, etc., e encontra o tesouro. A pergunta : esse pirata era sortudo ou um matemtico? Mesmo tendo sido apresentado em um curso sobre nmeros complexos, e para alunos que tinham bastante experincia eram professores de Matemtica , o problema da ilha do tesouro causou uma comoo. Na verdade, todos admitiram que, se o curso no fosse sobre nmeros complexos, a nenhum dos presentes teria ocorrido a ideia de resolver esse problema usando a lgebra dos nmeros complexos. E, mesmo depois da sugesto para faz-lo, quase ningum conseguiu.

127

RPM OBMEP

Qual a relao entre o problema e os nmeros complexos? Bem, tudo se baseia em dois fatos fundamentais: 1) no plano complexo, a diferena entre dois complexos traduz o vetor com origem no primeiro ponto e extremidade no segundo; o que se costuma formular por: AB = B A ; 2) multiplicar um complexo pelo nmero i (a unidade imaginria) equivale a gir-lo de um ngulo reto positivo. A figura ilustra a situao do problema. Sendo A a rvore, e P e Q as pedras, o tesouro est no ponto T mdio dos pontos P e Q. Considerando os pontos pertencentes ao plano complexo, no importando onde esteja a origem, tem-se:

T=

P + Q P i ( P A) + Q + i (Q A) P + Q Q P . = = +i 2 2 2 2

Observando que P + Q o ponto mdio 2 de PQ e que Q P = PQ , esse resultado no s demonstra que a localizao do tesouro independe da posio da rvore (o pirata era um matemtico...), como tambm permite localiz-lo como o terceiro vrtice de um dos tringulos retngulos issceles com hipotenusa PQ.
Adaptado do artigo A ilha do tesouro. Dois problemas e duas solues
RPM OBMEP

Jess A. P. Snchez e Jos Paulo Q. Carneiro, RPM 47.

128

Qual o mesmo a definio de polgono convexo?

Quando pensamos num polgono convexo, imaginamos seus vrtices todos apontando para fora, ou seja, que ele no possui vrtices reentrantes. Como os dois polgonos da esquerda na figura 1.

figura 1: Dois polgonos convexos e dois no convexos.

Essa idia intuitiva necessita, entretanto, uma formulao mais precisa, para poder ser usada com segurana e generalidade. Alm disso, h outras maneiras de pensar num polgono convexo. Conforme o contexto, uma dessas definies pode ser mais adequada do que as outras. Por isso conveniente conhecer as principais alternativas e saber mostrar que elas so equivalentes. A seguir, daremos trs definies diferentes de polgono convexo e provaremos a equivalncia entre elas.
RPM OBMEP

Chamamos polgono a uma linha poligonal fechada sem auto-intersees, isto , cada lado tem apenas um ponto comum com o lado anterior e com o seguinte, mas no com os demais. s vezes, a palavra polgono tambm designa a regio do plano limitada por essa linha poligonal fechada

129

sem auto-intersees. Por exemplo, quando falamos da rea de um polgono, claro que nos referimos regio poligonal, no linha que a limita. Um subconjunto F do plano chama-se uma figura plana convexa quando, para quaisquer dois pontos X e Y em F, o segmento de reta XY est inteiramente contido em F.

figura 2: Duas figuras planas convexas e duas no convexas.

Primeira definio Um polgono diz-se convexo quando a regio por ele limitada uma figura plana convexa. Segue-se desta definio que toda diagonal de um polgono convexo est inteiramente contida na regio por ele limitada. Para a segunda definio, lembremos que toda reta r decompe o plano em duas regies que tm r como fronteira comum. Chamaremos essas regies as margens de r. As margens de uma reta so figuras planas convexas. Se os pontos X e Y esto em margens opostas da reta r, o segmento de reta XY corta r. Diz-se que r uma reta de apoio do polgono P quando P tem pelo menos um ponto em comum com r e situa-se inteiramente numa das margens de r.

RPM OBMEP

figura 3: r a reta de apoio dos polgonos P1 e P2 mas no de P3 e P4.

130

Segunda definio Um polgono chama-se convexo quando a reta que contm qualquer dos seus lados uma reta de apoio. Por exemplo, dos polgonos na figura 3, apenas P4 convexo. Para formular a terceira definio de polgono convexo, definimos um ziguezague ABCD como uma poligonal com trs lados, AB, BC e CD, dispostos de modo que AB e CD se situem em margens opostas da reta (que contm o segmento) BC.

figura 4: A poligonal ABCD um ziguezague mas ABCD no .

Terceira definio Um polgono diz-se convexo quando no contm ziguezagues. Notemos que se ABCD um ziguezague contido no polgono P, ento um dos vrtices B, C saliente e o outro reentrante.
figura 5: No ziguezague ABCD, o vrtice C saliente para o polgono P e reentrante para Q. O contrrio ocorre com o vrtice B.

Para demonstrar a equivalncia entre estas trs definies de polgono convexo, usaremos a noo de ponta de um polgono. Sejam A, B, C vrtices consecutivos do polgono P. Diz-se que B uma ponta de P quando o segmento AC uma diagonal interna desse polgono.
figura 6: Os vrtices B e D (mas no A e C) so pontas de ABCD.
RPM OBMEP

131

Lema Todo polgono tem pelo menos uma ponta. Demonstrao Um polgono P, de n lados, decompe-se, mediante diagonais internas, em n 2 tringulos justapostos (RPM 18, p. 36). Cada um dos n lados de P pertence a pelo menos um desses n 2 tringulos. Pelo princpio da casa dos pombos (RPM 8, p. 21) h 2 lados de P no mesmo tringulo. O vrtice comum a esses dois lados uma ponta de P. O teorema seguinte estabelece a equivalncia entre as trs definies de polgono convexo dadas acima. Teorema 1 Cada uma das seguintes afirmaes a respeito de um polgono P implica, as demais: 1) A regio limitada por P uma figura plana convexa; 2) A reta que contm qualquer lado de P uma reta de apoio; 3) P no possui ziguezagues. Demonstrao Provaremos as implicaes 1) 2) 3) 1). 1) 2). Admitindo 1), suponhamos, por absurdo, que 2) seja falsa, isto , que exista um lado AB do polgono P e pontos X, Y da regio F limitada por P situados em margens opostas da reta AB, como na figura 7. Sendo F convexa, todos os pontos do segmento XY, e da todos os pontos do tringulo AXY, obtidos ligando A aos pontos de XY, esto contidos em F. Ento AB no lado de P. Contradio.
RPM OBMEP

2) 3). Se ABCD um ziguezague, AB e CD esto em margens opostas da reta BC. Portanto, um polgono onde a reta que contm qualquer dos seus lados de apoio no pode conter ziguezagues. 3) 1). Para provar esta ltima implicao suponhamos, por absurdo, que exista um polgono P, com n lados, que no contm

132

ziguezagues mas a regio F, por ela limitada, no uma figura plana convexa. Tomemos P de modo que n seja o menor possvel. Ento 3) 1) para polgonos com menos de n lados. Pelo lema, existem vrtices consecutivos L, A, B, C, D de P tais que B uma ponta. A diagonal AC decompe P em dois polgonos justapostos: o tringulo ABC e um polgono Q, de n 1 lados, que no contm ziguezagues, logo limita uma figura plana convexa G.

figura 8: O polgono P decomposto no tringulo ABC e no polgono Q, de n 1 lados.

Assim, para provar que F uma figura plana convexa, basta tomar um ponto X no tringulo ABC, um ponto Y na regio G e mostrar que o segmento de reta XY est contido em F. Como j vimos que 1) 2), sabemos que AC uma reta de apoio para Q, logo X e Y esto em margens opostas de AC. Alm disso, como LABC e ABCD no so ziguezagues, X e Y esto na mesma margem em relao s retas AB e BC. Tudo isto significa que o segmento XY corta a reta AC mas no as retas AB ou BC. Noutras palavras, o segmento XY sai do tringulo ABC por um ponto Z do segmento AC. Ento XZ est contido no tringulo ABC e ZY est contido na regio G, logo XY est contido na regio F, como queramos demonstrar. Para finalizar, breves observaes sobre as definies acima propostas: 1. A primeira definio a que melhor se adapta aos padres atuais da Matemtica, tanto Pura, como Aplicada. Ela se aplica literalmente a figuras slidas com um nmero qualquer de dimenses. Dela resulta facilmente que a interseo de duas ou mais figuras convexas uma figura convexa. Por isso simples deduzir dela que um polgono convexo se, e somente se, tem exatamente dois pontos em comum com qualquer reta que passa pelo seu interior. (Isto seria uma quarta definio de polgono convexo.)

133

RPM OBMEP

2. A segunda definio tambm se estende a poliedros em espaos com um nmero qualquer de dimenses. Ela permite caracterizar um polgono convexo como o conjunto das solues (x, y) de um sistema de desigualdades lineares do tipo ax + by < c. Por isso desempenha papel fundamental em Programao Linear. 3. As duas primeiras definies tm carter global enquanto a terceira nitidamente local. Para verificar se um dado polgono convexo no sentido das duas primeiras definies necessrio examinar (vrias vezes) todos os seus lados ao mesmo tempo. J na terceira definio, para cada lado, olha-se apenas para o lado sua esquerda e para o lado sua direita. Do ponto de vista computacional, isto bem mais simples. Por outro lado, a no existncia de ziguezagues s faz sentido no plano. Alm disso, trata-se de uma hiptese da qual, em que pese seu grande apelo geomtrico, difcil deduzir conseqncias. (Compare 3) 1) com as outras implicaes.)
Adaptado do artigo Qual mesmo a definio de polgono convexo? Elon Lages Lima, RPM 21.

134

RPM OBMEP

A soluo de Tartaglia para a equao do 3o grau e a emergncia dos nmeros complexos

Introduo A histria da resoluo da equao de terceiro grau muito pitoresca, plena de lances dramticos, paixes e disputas pela fama e a fortuna que seu achado poderia trazer a seus autores. Uma das personagens dessa histria Niccol Fontana (1500-1557 aprox.). Em 1512 os franceses saquearam Brescia, sua cidade natal. Sua me buscou refgio para o filho na igreja, mas os soldados tambm invadiram o santurio, e a criana foi ferida no rosto. O ferimento lhe causou uma gagueira permanente, que lhe valeu o apelido de Tartaglia (gago, em italiano), pelo qual se tornou conhecido. Ele no foi o primeiro a obter o mtodo de resoluo dessas equaes; Scipione del Ferro (14651562 aprox.), que foi professor na Universidade de Bolonha e cuja biografia pouco conhecida, foi o verdadeiro descobridor. Antes de morrer, del Ferro ensinou seu mtodo a dois discpulos, Annibale della Nave - seu futuro genro e sucessor na ctedra em Bolonha - e Antnio Maria Fior (ou Floridus, em latim). Em 1535 houve uma disputa matemtica entre Fior e Tartaglia. Tais confrontos intelectuais no eram infrequentes na poca e, muitas vezes, a permanncia de um matemtico numa ctedra dependia de seu bom

135

RPM OBMEP

desempenho nesses encontros. Cada um dos adversrios props ao outro trinta problemas e foi combinado que o perdedor deveria pagar trinta banquetes ao ganhador. Tartaglia preparou questes variadas, mas todos os problemas propostos por Fior implicavam equaes do tipo X3 + aX = b. Precisamente na noite de 12 para 13 de fevereiro, Tartaglia conseguiu descobrir o mtodo de resoluo de tais equaes e, na hora do confronto, verificou-se que Tartaglia tinha resolvido todas as questes propostas por Fior, enquanto este no tinha conseguido resolver a maioria das questes submetidas por Tartaglia. Declarado vencedor, Tartaglia voluntariamente renunciou aos trinta banquetes. A notcia do triunfo de Tartaglia logo se espalhou e chegou aos ouvidos de Girolamo Cardano (1501-1576), que, na poca, ocupava uma cadeira de medicina na Universidade de Pavia e era membro do Colgio Mdico de Milo. De todos os participantes da nossa histria, talvez seja Cardano o mais enigmtico, aquele cuja vida mais pitoresca e, certamente, que teve uma formao mais universal. Para termos uma idia de quo extenso e profundo era seu conhecimento, citamos a seguir os comentrios de Gabriel Naud (16001653), que publicou a autobiografia de Cardano pela primeira vez em 1643: No somente era ele inquestionavelmente um mdico notvel, como foi tambm provavelmente o primeiro e nico homem a se distinguir em todas as cincias ao mesmo tempo. uma das ilustraes da Natureza daquilo que um homem capaz de atingir. Nada de significativo lhe era desconhecido em filosofia, medicina, astronomia, matemtica, histria, metafsica ou as cincias sociais, ou em outras reas mais remotas do conhecimento. Ele tambm errava, claro, isto apenas humano; maravilhoso, porm, quo raramente ele errava. Na poca da descoberta de Tartaglia, Cardano gozava de boa posio em Milo e o convidou a sua casa. Uma vez l, com muita insistncia Cardano conseguiu que lhe fosse revelado o segredo da resoluo das equaes do terceiro grau. Tartaglia consentiu em lhe ensinar a regra de resoluo (embora no lhe ensinasse a demonstrao da mesma), sob forma de versos, em troca

136

RPM OBMEP

do juramento solene de que Cardano jamais publicaria esse segredo. Conhecendo um mtodo de resoluo, Cardano procurou e achou uma demonstrao que o justificasse. De posse da soluo, Cardano deve ter se sentido fortemente tentado a public-las. Em 1544 fez uma viagem a Florena e, no caminho, visitou Annibale delia Nave, em Bologna, que lhe mostrou um manuscrito de del Ferro que continha a famosa regra de Tartaglia, manuscrito este que ainda se conserva. Aparentemente, ao saber que a frmula de Tartaglia existia j desde trinta anos antes, Cardano se sentiu desobrigado de cumprir seu juramento e publicou, em 1545, em Nuremberg, uma obra intitulada Ars Magna, que o tornou verdadeiramente famoso em todo o continente. Nas palavras de C. Boyer, ele provavelmente era o matemtico mais competente da Europa. Nessa obra aparecem, pela primeira vez, as regras de resoluo das equaes do terceiro e quarto graus. A seu favor, podemos dizer que Cardano no esquece de fazer as devidas atribuies de mrito aos respectivos descobridores. A seguir, faremos uma anlise do mtodo que Tartaglia confiou a Cardano. Os Versos de Tartaglia Como dissemos acima, Tartaglia comunicou a Cardano o segredo da sua descoberta por meio de versos. Tal idia no to estranha quanto pode parecer a princpio; devemos lembrar que, na poca, os autores no dispunham ainda de uma notao adequada para tratar as equaes em sua generalidade e no podiam, portanto, expressar seus mtodos resumidamente mediante frmulas, como fazemos hoje em dia. A seguir, reproduzimos os versos na sua verso original, tal como transcritos na edio de 1554 dos Quesiti et inventione diverse de Tartaglia. 1. Quando chel cubo con le cose appreso Se aggaglia a qualque nmero discreto Trovati due altri differenti in esso 2. Depoi terrai questo por consueto Chel lor produtto sempre sia eguale Al terzo cubo delle cose neto

137

RPM OBMEP

3. El resduo poi suo generale Delli lor lati cubi ben sostratti Verra la tua cosa principale 4. In el secondo de coiesti aiti Quando chel cubo restasse lui solo Tu osserverai questaltri contratti 5. Del nmero farai due, tal parta volo Cha luno e laltro si produca schietto El terzo delle cose in stelo 6. Delle qual poi, per commun precetto Torrai li lati cubi incieme gionti Et cotal somma sera il tuo concetto 7. El terzo poi de questi nostri conti Se solve con secondo, se ben guardi Che ser natura son quasi congiontri 8. Questi trovai, et non con passi tardi nel mille cinquecento quatro et trinta Nella citt dal mare intorno centa. Uma traduo para o portugus ficaria, mais ou menos, assim: 1. Quando o cubo com a coisa em apreo Se igualam a qualquer nmero discreto Acha dois outros diferentes nisso 2. Depois ters isto por consenso Que seu produto seja sempre igual Ao cubo do tero da coisa certo 3. Depois, o resduo geral Das razes cbicas subtradas Ser tua coisa principal 4. Na segunda destas operaes, Quando o cubo estiver sozinho Observars estas outras redues

138

RPM OBMEP

5. Do nmero fars dois, de tal forma Que um e outro produzam exatamente O cubo da tera parte da coisa 6. Depois, por um preceito comum Toma o lado dos cubos juntos E tal soma ser teu conceito 7. Depois, a terceira destas nossas contas Se resolve como a segunda, se observas bem Que suas naturezas so quase idnticas 8. Isto eu achei, e no com passo tardo No mil quinhentos e trinta e quatro Com fundamentos bem firmes e rigorosos Na cidade cingida pelo mar Analisaremos, a seguir, esses versos numa linguagem acessvel ao leitor contemporneo. Antes de tudo, conveniente lembrar que Tartaglia (assim como depois faria tambm Cardano) no utiliza coeficientes negativos em suas equaes. Ento, em vez de uma equao geral do terceiro grau, ele deve considerar trs casos possveis: x3 + ax = b x3 = ax + b x3 + b = ax . Tartaglia chama cada um desses casos de operaes e afirma que ir considerar, de incio, equaes do primeiro tipo: cubo e coisa igual a nmero. No quarto verso comea a considerar o segundo tipo quando o cubo estiver sozinho e, no stimo, faz referncia ao terceiro caso. Vejamos agora como se prope a resolver o primeiro caso, nos trs versos iniciais, para depois justificar seu mtodo, de uma forma simples. O nmero se refere ao termo independente, que ns denotamos aqui por b. Quando diz acha dois outros diferentes nisso, est sugerindo tomar duas novas variveis cuja diferena seja precisamente b, i.e., escolher U e V tais que: U V = b.

139

RPM OBMEP

A frase ... que seu produto seja sempre igual a cubo da tera parte da coisa significa que U e V devem verificar:
a UV = ( )3 . 3 Finalmente, o resduo geral das razes cbicas subtradas ser tua coisa principal significa que a soluo estar dada por

x = 3U 3V .
Os outros dois casos carecem de interesse para o leitor moderno, uma vez que podemos reduzi-los ao primeiro, mudando termos de um membro a outro da equao. A frase final ... a cidade cingida pelo mar uma referncia a Veneza, onde realizou suas descobertas. A Resoluo da Equao do Terceiro Grau Nesta seo veremos como justificar a frmula de Tartaglia para resolver equaes do terceiro grau. Naturalmente, utilizaremos mtodos e notaes modernos, o que nos permitir dar uma exposio relativamente simples. Vamos considerar uma equao do terceiro grau escrita na forma: x3 + ax = b. para compar-la com a primeira destas operaes . . . cubo e coisa igual a nmero, discutida nos trs primeiros versos de Tartaglia. Na verdade, h um caminho muito simples para ach-la. Comecemos por lembrar a frmula do cubo de um binmio: (u v)3 = u3 3u2v + 3uv2 v3. Pondo em evidncia o produto uv, temos: (u v)3 = 3uv(u v) + (u3 v3), isto , (u v)3 + 3uv(u v) = u3 v3 .
RPM OBMEP

Se podemos escolher, de alguma forma, u e v de modo que verifiquem: uv = a/3 u v3 = b,


3

140

a relao acima se transformar em: (u v)3 + a(u v) = b o que significa que x = u v ser uma soluo da equao dada. Em outras palavras, se conseguirmos achar u e v que sejam solues do sistema acima, tomando x = u v obter-se- uma soluo da equao proposta. Resta-nos ento o problema de resolver o sistema. Para isso, observemos que, elevando ao cubo a primeira equao, ela se transforma em: u3v3 = (a/3)3 u3 v3 = b. Finalmente, fazendo u3 = U e v3 = V, temos: UV = (a/3)3 U V = b. Isso muito fcil de resolver; U e V so as razes da equao: X2 bX + (a/3)3 = 0 que so dadas por:
b b 2 4( 2 a 3 ) b b a 3 = ( ) 2 + ( )3 . 2 2 3

X=

Podemos tomar uma dessas razes como sendo U e a outra como V, logo temos u = 3 U e v = 3 V . Portanto, obtemos precisamente a soluo enunciada por Tartaglia:

x = 3U 3V .
Mais explicitamente, substituindo U e V pelos seus respectivos valores, resulta a conhecida frmula que, nos textos, chamada de frmula de Cardano ou de Tartaglia:
RPM OBMEP

x=3

b b a b b a + ( ) 2 + ( )3 + 3 ( ) 2 + ( )3 . 2 2 3 2 2 3

Uma observao final: a equao geral do terceiro grau, que podemos escrever na forma:

141

x3 + a1x2 + a2x + a3 = 0, pode-se reduzir ao caso acima, mediante a mudana de varivel x = y (a1/3). Alis, essa reduo era conhecida por Tartaglia, mas no por Fior, e foi justamente esse fato que determinou a vitria do primeiro. Isso significa que, na verdade, Tartaglia conhecia um mtodo geral para resolver qualquer equao do terceiro grau. A emergncia dos nmeros complexos Os nmeros complexos desempenham um papel sumamente importante nos mais diversos ramos da Matemtica e, atravs destes, em muitas das aplicaes a outras reas do conhecimento. Em geral, o estudante se depara com eles, pela primeira vez, ainda no curso secundrio e sua introduo justificada pela necessidade de resolver equaes de segundo grau com discriminante negativo. Isso cria uma falsa impresso, j que, historicamente, no foram as equaes de segundo grau que levaram introduo dos nmeros complexos. Neste texto analisaremos essa questo e alguns outros aspectos ligados ao desenvolvimento do assunto. O fato de que um nmero negativo no tem raiz quadrada parece ter sido sempre claro para os matemticos que se depararam com a questo. As equaes de segundo grau apareceram na Matemtica j nas tabuletas de argila da Sumria, aproximadamente 1700 anos antes de Cristo e, ocasionalmente, levaram a radicais de nmeros negativos; porm, no foram elas, em momento algum, que sugeriram o uso de nmeros complexos. Em rigor, uma equao era vista como a formulao matemtica de um problema concreto; assim, se no processo de resoluo aparecia uma raiz quadrada de um nmero negativo, isso era interpretado apenas como uma indicao de que o problema originalmente proposto no tinha soluo. Como veremos adiante, foram s as equaes de terceiro grau que impuseram a necessidade de trabalhar com esses nmeros. Vejamos inicialmente alguns antecedentes. Na Arithmetica, de Diophanto, aproximadamente no ano de 275 d.C. ele considera o seguinte problema:

142

RPM OBMEP

Um tringulo retngulo tem rea igual a 7 e seu permetro de 12 unidades. Encontre o comprimento dos seus lados. Chamando de x e y o comprimento dos catetos desse tringulo, temos, na nossa notao atual:

1 xy = 7; x 2 + y 2 = (12 x y ) 2 . 2
Substituindo y em funo de x, obtemos a equao 24x2 172x + 336 = 0. Nesse ponto Diophanto observa que s poderia haver soluo se
172 2 ( ) 24 336 . Nesse contexto, claro que no h necessidade 2

alguma de introduzir um sentido para a expresso o discriminante da equao.

167 , sendo 167

Na verdade, o primeiro registro de um radical de um nmero negativo um pouco anterior: ele aparece na Estereometria de Heron, matemtico grego do perodo Alexandrino, publicada aproximadamente em 75 d.C. Num clculo sobre o desenho de uma pirmide surge a necessidade de

81 144 . A questo parece no causar nenhum problema avaliar simplesmente porque logo em seguida os nmeros apresentam-se
trocados:

144 81 , resultando
15 . 16

63 , que calculado como aproxima-

damente igual a 7

Encontram-se novas referncias questo na Matemtica indiana. Aproximadamente no ano de 850 d.C, o matemtico indiano Mahavira afirma:
... como na natureza das coisas um negativo no um quadrado, ele no tem, portanto, raiz quadrada.
RPM OBMEP

J no sculo XII, o famoso matemtico Bhaskara (1114-1185 aprox.) escreve:


O quadrado de um afirmativo afirmativo; e a raiz quadrada de um afirmativo dupla: positiva e negativa. No h raiz quadrada de um negativo; pois ele no um quadrado.

143

Tambm na Matemtica europia aparecem observaes dessa natureza; Luca Paccioli, na sua Summa de arithmetica, geomtrica, proportioni et proportionalita, publicada em 1494, escreve que a equao x2 + c = bx solvel somente se

1 2 b c , e o matemtico francs 4 Nicolas Chuquet (1445-1500 aproximadamente) faz observaes semelhantes sobre solues impossveis num manuscrito, no publicado, de 1484.
O prprio Cardano se deparou com esse tipo de questes e, embora mantivesse a atitude dos seus contemporneos, no sentido de entender que razes de nmeros negativos indicavam apenas a no-existncia de solues de um determinado problema, pelo menos em um caso ele deu um passo a mais. No Captulo 37 do Ars Magna, ele considera o problema de dividir um segmento de comprimento 10 em duas partes cujo produto seja 40.

Se chamamos de x o comprimento de uma das partes, a outra ter comprimento 10 x, e a condio do problema se traduz na equao: x(10 x) = 40. Isso leva equao x2 l0x + 40 = 0, cujas solues so 5 15 . Cardano reconhece que o problema dado no tem soluo mas, talvez a ttulo de curiosidade, observa que, trabalhando com essas expresses como se fossem nmeros, deixando de lado as torturas mentais envolvidas e multiplicando 25 (15), que igual a 40.
5 + 15 por 5 15 , obtm-se

Em consequncia, ele chama essas expresses de razes sofsticas da equao e diz, a respeito delas, que so to sutis quanto inteis.
RPM OBMEP

A necessidade dos nmeros complexos Raphael Bombelli (1526-1573) era um admirador da Ars Magna de Cardano, mas achava que seu estilo de exposio no era claro (ou, em suas prprias palavras, ma nel dire f oscuro). Decidiu, ento, escrever

144

um livro expondo os mesmos assuntos, mas de forma tal que um principiante pudesse estud-los sem necessidade de nenhuma outra referncia. Publicou lAlgebra, em trs volumes, em 1572, em Veneza, obra que viria a se tornar muito influente. No captulo II dessa obra, ele estuda a resoluo de equaes de grau no superior a quatro. Em particular na pgina 294 e nas seguintes, ele considera a equao x3 = 15x+ 4. Ao aplicar a frmula de Cardano para o clculo de uma raiz, ele obtm:
x = 3 2 + 121 + 3 2 121 .

Seguindo Cardano, ele tambm chama essa expresso de sofstica, mas, por outro lado, ele percebe que x = 4 , de fato, uma raiz da equao proposta. Assim, pela primeira vez, nos deparamos com uma situao em que, apesar de termos radicais de nmeros negativos, existe verdadeiramente uma soluo da equao proposta. necessrio, ento, compreender o que est acontecendo. Bombelli concebe ento a possibilidade de que exista uma expresso da forma a + b que possa ser considerada como raiz cbica de

2 + 121 i.e., que verifique (a + b )3 = 2 + 121 . A forma em que ele calcula essa raiz um tanto peculiar; ele assume que a raiz cbica
de 2 121 seja da forma a b . Como ele sabe que 4 deve ser raiz da equao, necessariamente a + b + a b = 4 . Nesse ponto, felizmente, as quantidades no existentes se cancelam e obtemos a = 2. Com esse resultado, muito fcil voltar equao
(a + b )3 = 2 + 121 e deduzir que b = 1. Assim, ele obtm que
3

2 + 121 = 2 + 1 e que:
RPM OBMEP

x = 2 + 1 + 2 1 = 4

uma soluo da equao dada. Bombelli percebeu claramente a importncia desse achado. Ele diz:
Eu achei uma espcie de raiz cbica muito diferente das outras, que aparece no captulo sobre o cubo igual a uma quantidade e um nmero.

145

... A princpio, a coisa toda me pareceu mais baseada em sofismas que na verdade, mas eu procurei at que achei uma prova... . Isto pode parecer muito sofisticado mas, na realidade, eu tinha essa opinio, e no pude achar a demonstrao por meio de linhas [i.e. geometricamente], assim, tratarei da multiplicao dando as regras para mais e menos.

Ele utiliza a expresso pi di meno para se referir ao que ns denotaramos como +i e meno di meno para i. Ele enuncia ento o que chama de regras do produto, que citamos abaixo junto com sua traduo na nossa simbologia:
Pi via pi di meno fa pi di meno, Meno via pi di meno fa meno di meno, Pi via meno di meno fa meno di meno, Meno via meno di meno fa pi di meno, Pi di meno via pi di meno fa meno, Meno di meno via pi di meno fa pi, Meno di meno via meno di meno fa meno. +.(+i) = +i .(+i) = i +.(i) = i .(i) = +i (+i).(+i) = (i).(+i) = + (i).(i) =

E interessante notar que Bombelli se deparava com a dificuldade adicional de no dispor de uma boa notao. Ele utilizava p (plus) para indicar a soma; m (minus) para a subtrao; R (radix) para raiz quadrada e R3 para a raiz cbica. Tambm no dispunha de parnteses; nos seus manuscritos sublinhava expresses para indicar quais os termos afetados por um radical. Assim, por exemplo, a expresso escrita na forma R3 | 2 pR| 0 121 | | . Note que, como no escrevia diretamente nmeros negativos, ele escreveu 121 como 0 121. Dessa forma, a soluo da equao discutida acima aparecia como: R3 | 2 pR| 0 121 | | pR3 | 2mR| 0 121 | | .
RPM OBMEP
3

2 + 121

era

Progressos ulteriores Faremos aqui um pequeno resumo da evoluo dos nmeros complexos, para que o leitor tenha uma viso global da histria do

146

assunto. Comearemos listando alguns progressos na notao para depois nos ocuparmos da evoluo dos conhecimentos. O smbolo

1 foi introduzido em 1629 por Albert Girard.

O smbolo i foi usado pela primeira vez para representar 1 por Leonhard Euler em 1777, apareceu impresso pela primeira vez em 1794 e se tornou amplamente aceito aps seu uso por Gauss em 1801. Os termos real e imaginrio foram empregados pela primeira vez por Ren Descartes em 1637. O expresso nmero complexo foi introduzida por Carl Friederich Gauss em 1832. Como observamos na seo anterior, a partir do trabalho de Bombelli, os nmeros complexos comearam a ser utilizados devido a sua bvia utilidade para resolver equaes de terceiro grau mas, ao mesmo tempo, era claro que tais nmeros no poderiam existir. A primeira tentativa de legitimao, via uma interpretao geomtrica, devida a John Wallis (1616-1703), contemporneo de Newton e professor na Universidade de Oxford. Em 1673 ele publicou um tratado intitulado lgebra, em cujo captulo LXVI discute a impossibilidade da existncia de quantidades imaginrias e compara essa questo com a da existncia de quantidades negativas.
Essas quantidades imaginrias (como so freqentemente chamadas) surgem das supostas razes de um quadrado negativo (quando aparecem) e se considera que implicam que o caso proposto impossvel. E assim , de fato, no sentido estrito do que foi proposto. Pois no possvel que qualquer nmero (negativo ou afirmativo), multiplicado por si mesmo, possa produzir (por exemplo) 4. Pois sinais iguais (tanto + quanto ) produziro +; e portanto no 4. Mas tambm impossvel que qualquer quantidade (embora no um suposto quadrado) possa ser negativa. Pois no possvel que qualquer magnitude possa ser menos que nada, ou qualquer nmero menor que nada. Porm, no esta suposio (das quantidades negativas) nem intil nem absurda, quando corretamente compreendida. E, embora para a simples notao algbrica representa uma quantidade menor do que

147

RPM OBMEP

nada, quando se trata de uma aplicao fsica, denota uma quantidade to real como se o sinal fosse +; mas interpretada no sentido contrrio.

Depois de considerar diversos exemplos de nmeros negativos interpretados em termos de segmentos sobre uma reta orientada, ele tenta uma interpretao para as quantidades imaginrias:
Suponhamos que num local ganhamos do mar 30 acres, mas perdemos em outro local 20 acres: se agora formos perguntados quantos acres ganhamos ao todo a resposta 10 acres, ou +10 (pois 30 20 = 10). ... Mas se num terceiro local perdemos mais 20 acres, a resposta deve ser 10 (pois 30 20 20 = 10) ... . Mas agora, supondo que esta plancie negativa de 1600 square perches [20 acres correspondem a 1600 square perches, uma outra medida inglesa da poca] tem a forma de um quadrado, no devemos supor que este quadrado tem um lado? E, assim, qual ser esse lado? No podemos dizer que 40, nem 40 ... Mas sim que
1600 (a

suposta raiz de um quadrado negativo) ou 10 16 ou 20 4 ou


40 1 .

Como era de se esperar, essa interpretao no teve uma grande acolhida entre seus contemporneos e nenhuma repercusso posterior. Notemos que, at aqui, nada garante que razes cbicas - ou, em geral, razes n-simas de complexos sejam, de fato, complexos. Tal como assinala M. Kline, no comeo do sculo XVIII, a maioria dos matemticos ainda acreditava que razes de diferente ordem de nmeros complexos levariam introduo de diferentes tipos de complexos. Jean Le Rond dAlembert (1717-1783), aps estudar Direito e Medicina, decidiu dedicar sua vida Matemtica. Trabalhou em lgebra, clculo e suas aplicaes, equaes diferenciais ordinrias e parciais, funes de varivel complexa, mecnica e dinmica. Em 1747 publicou Reflxions sur Ia cause gnrale des vents, em que afirmou que toda expresso construda algbricamente a partir de um nmero complexo (onde inclua tambm a extrao de razes) da forma a + b 1 . No formulou uma prova satisfatria no caso de expresses da forma (a + bi)c+di, tarefa que seria completada por Euler.

148

RPM OBMEP

DAlembert foi amigo de Voltaire e colaborou com diversos artigos para a Enciclopdie, mas manteve nessa um discreto silncio sobre os nmeros complexos. Roger Cotes (1682-1716) foi um jovem professor no famoso Trinity College de Cambridge e, aps sua prematura morte, dele disse Newton: Se Cotes tivesse vivido, teramos aprendido alguma coisa. Em 1714 ele obteve um importante resultado, relacionado com a obteno de razes n-simas da unidade que, na notao moderna, poderamos explicitar como: loge(cos + isen) = i. Isso poderia ter levado famosa relao de Euler: cos + isen = ei. que, por sua vez, implica a frmula de De Moivre: (cos + isen)n = cos(n) + isen(n) o que resolveria o problema de achar razes. Porm, o caminho foi outro. Abraham De Moivre (1667-1754) nasceu na Frana, mas viveu na Inglaterra a partir dos dezoito anos. Estudou Matemtica sozinho, aps ler os Principia de Newton, chegando a se tornar membro da Royal Society e das academias de Paris e Berlim. Seu trabalho versou fundamentalmente sobre trigonometria, probabilidade e clculo de anuidades. Em 1722, utilizando fatos que j havia publicado em 1707, ele obteve um resultado que implicou a frmula que leva seu nome, embora tenha se limitado a casos particulares e nunca tenha chegado a enunciar ou demonstrar a frmula no caso geral. Essa tarefa coube a Leonhard Euler (1707-1783), considerado o mais prolfico matemtico de todos os tempos. Numa carta endereada a Jean Bernoulli, datada de 18 de outubro de 1740, ele afirma que y = 2 cos e y = eix + eix eram ambas solues da mesma equao diferencial (o que reconheceu atravs do desenvolvimento em srie das solues) e que, portanto, deviam ser iguais. Publicou esse resultado em 1743; explicitamente:
cos = ei + e i ei e i e sen = . 2 2i

149

RPM OBMEP

Em 1748 ele redescobriu o resultado de Cotes, demonstrou a frmula de De Moivre e estendeu sua validade para todo exponente n real. Com isso, a existncia de razes no campo complexo ficou definitivamente estabelecida. Obviamente, Euler compreendia e utilizava muito bem os nmeros complexos. O fato de ele prprio ter grandes dvidas quanto a sua legitimidade ilustra claramente o status desse corpo numrico na poca. Em Vollstndige Anleitung zur Algebra, publicada primeiro em russo, em 1768-69, e depois em alemo, em 1770, que se tornou uma referncia clssica nessa rea nos dois sculos seguintes, Euler escreve:
Uma vez que todos os nmeros concebveis so maiores do que 0, ou menores do que 0 ou iguais a 0, claro que a raiz quadrada de um nmero negativo no pode ser includa entre os nmeros possveis. Consequentemente, devemos dizer que esses so nmeros impossveis. E essa circunstncia nos conduz a tais nmeros, que por sua natureza so impossveis, e que so chamados costumeiramente de imaginrios, pois eles s existem na imaginao.

A representao grfica A representao geomtrica dos nmeros complexos mediante pontos do plano foi decisiva para sua aceitao. A possibilidade dessa representao era clara para vrios autores, como Cotes, De Moivre, Euler e Vandermonde; todos eles tentaram resolver a equao xn 1 = 0 pensando em suas solues como vrtices de um polgono regular de n lados. Essa ideia era ainda incompleta, pois nenhum desses autores achou tambm uma interpretao geomtrica para as operaes com complexos. O primeiro a formular uma tal interpretao foi um agrimensor noruegus chamado Caspar Wessel (1745-1818), um autodidata. Ele autor de um artigo intitulado Sobre a representao analtica da direo: uma tentativa, que foi publicado em 1799 nas memrias da Real Academia da Dinamarca. Ali, escreveu:
RPM OBMEP

Vamos designar por +1 a unidade retilnea positiva, por + outra perpendicular primeira, com a mesma origem; ento o ngulo de direo de +1 ser 0, o de 1 ser 180, o de ser 90 e o de ser 90 ou 270.

150

Tal como fazemos hoje em dia, ele representa o complexo a + bi pelo vetor do plano com origem O a origem do sistema de eixos coordenados e com extremo no ponto P de coordenadas (a, b). Depois d uma representao geomtrica da soma de dois complexos a + bi e c + di, representando-os pelos vetores OP e OQ, respectivamente, e observando que a soma estar respresentada pela diagonal do paralelogramo construdo sobre OP e OQ. De forma anloga, o produto desses complexos estar representado por um vetor OR tal que o comprimento de OR o produto dos comprimentos de OP e OQ, e o ngulo que OR forma com o eixo Ox igual soma dos ngulos formados por OP e OQ com esse eixo. Uma representao semelhante foi dada por Jean-Robert Argand (1768-1822), um bibliotecrio suo, tambm autodidata, que em 1806 publicou um pequeno livro intitulado Essai sur la manire de reprsenter les quantits imaginaires dans les constructions gomtriques. Ele observa que se multiplicamos +1 por i obtemos i e se multiplicamos esse resultado novamente por i obtemos 1. Ele pensa, ento, em representar i por uma operao que aja de modo anlogo. Assim, podemos representar i por uma rotao de 90 em sentido anti-horrio. A partir daqui, tal como Wessel, ele d interpretaes para nmeros da forma a + bi e para as operaes com complexos, aplicando seus resultados demonstrao de teoremas de lgebra, geometria e trigonometria. Esses trabalhos tiveram pouco ou nenhum efeito sobre os matemticos da poca; a memria de Wessel s foi notada quando publicada em traduo francesa em 1897, e o livro de Argand, embora causasse uma certa controvrsia, teve pouco impacto, talvez por ser a nica contribuio de seu autor Matemtica. Quem verdadeiramente tornou a interpretao geomtrica amplamente aceita foi Carl Friederich Gauss (1777-1855). A julgar pelas suas demonstraes do teorema fundamental da lgebra, ele j conhecia a interpretao grfica dos complexos em torno de 1815, embora escrevesse, numa carta de 1825, que a verdadeira metafsica de
1 elusiva. Finalmente, em 1831, ele escreveu um artigo muito explcito sobre a questo. Diz na introduo:

151

RPM OBMEP

O autor tem considerado h vrios anos essa parte importante da Matemtica sob um ponto de vista diferente, que permite conferir s quantidades imaginrias, como as negativas, uma existncia objetiva. O significado intuitivo dos nmeros complexos fica completamente estabelecido e no se precisa mais para admitir estas quantidades no domnio da aritmtica.

Ele observa tambm que se as unidades 1, 1, 1 no fossem chamadas de positiva, negativa e imaginria, mas direta, inversa e lateral, as pessoas no teriam tido a impresso de que h algo de misterioso nesses nmeros. A observao de Gauss a respeito da existncia objetiva dos nmeros complexos ilustra a viso da Matemtica na poca. Parece que o fato de esses nmeros poderem ser representados geometricamente lhes d essa existncia. Em outras palavras, parece que, para os matemticos daquele perodo, os entes geomtricos tinham um tipo de realidade que faltava aos objetos da aritmtica. Finalmente, a formalizao completa dos nmeros complexos como pares ordenados de nmeros reais ser desenvolvida por William Rowan Hamilton (1805-1865) em 1833, e ainda Agustin Cauchy (1789-1857) daria outro tipo de formalizao em 1847.
Adaptado dos artigos A soluo de Tartaglia para a equao do terceiro grau Csar Polcino Milies, RPM 25. A emergncia dos nmeros complexos Csar Polcino Milies, RPM 24.

152

RPM OBMEP

Grandezas incomensurveis e nmeros irracionais

Existem, em Matemtica, conceitos que parecem muito simples a uma viso superficial, mas que, submetidos a uma anlise mais cuidadosa, revelam aspectos verdadeiramente surpreendentes. Vamos tratar aqui da reta na sua representao numrica em termos das abscissas de seus pontos para mostrar que esses conceitos de reta e de nmero no tm uma simplicidade to inocente como parecem revelar a uma viso menos profunda. Exploraremos alguns fatos notveis e inesperados, que esto ligados primeira grande crise do desenvolvimento da Matemtica, ocorrida no final do 5o sculo a.C. Uma questo com que lidavam os matemticos gregos daquela poca era a de comparar grandezas da mesma espcie, como dois segmentos de reta, duas reas ou dois volumes. No caso de dois segmentos retilneos AB e CD, dizer que a razo AB/CD o nmero racional m/n, significava para eles (e ainda significa para ns) que existia um terceiro segmento EF tal que AB fosse m vezes EF e CD n vezes esse mesmo segmento EF. Na figura 1 ilustramos essa situao com m = 8 e n = 5. A B
figura 1

E F

AB 8 = CD 5

153

RPM OBMEP

No tempo de Pitgoras (580 500 a.C. aproximadamente) e mesmo durante boa parte do 5o sculo a.C. pensava-se que os nmeros racionais fossem suficientes para comparar segmentos de reta; isto , dados dois segmentos AB e CD, seria sempre possvel encontrar um terceiro segmento EF contido um nmero inteiro de vezes em AB e outro nmero inteiro de vezes em CD, situao esta que descrevemos dizendo que EF um submltiplo comum de AB e CD. Uma simples reflexo revela que essa uma idia muito razovel. Afinal, se EF no serve, podemos imaginar um segmento menor, outro menor ainda, e assim por diante. Nossa intuio geomtrica parece dizer-nos que h de existir um certo segmento EF, talvez muito pequeno, mas satisfazendo aos propsitos desejados. Na figura 2 ilustramos uma situao com segmento EF bem menor que o da figura 1. O leitor deve ir muito alm, imaginando um segmento EF to pequeno que nem se possa mais desenhar, para se convencer, pela sua intuio geomtrica, da possibilidade de sempre encontrar um submltiplo comum de AB e CD. B A

C
figura 2

DE F

AB 29 = CD 26

Dois segmentos nessas condies so ditos comensurveis, justamente por ser possvel medi-los ao mesmo tempo, com a mesma unidade EF. Entretanto, no verdade que dois segmentos quaisquer sejam sempre comensurveis. Em outras palavras, existem segmentos AB e CD sem unidade comum EF, os chamados segmentos incomensurveis. Esse um fato que contraria nossa intuio geomtrica, e por isso mesmo a descoberta de grandezas incomensurveis na Antiguidade representou um momento de crise no desenvolvimento da Matemtica. Foram os prprios pitagricos que descobriram grandezas incomensurveis, provavelmente entre 450 e 400 a.C.; e, ao que tudo indica, isto se fez atravs de um argumento geomtrico, como o que apresentaremos a seguir, demonstrando que o lado e a diagonal de um quadrado so segmentos incomensurveis.

154

RPM OBMEP

C E F B D
figura 3

Na figura 3 representamos um quadrado com diagonal = AB e lado = AC. Suponhamos que e sejam comensurveis. Ento existir um terceiro segmento que seja submltiplo comum de e . Fazemos agora a seguinte construo: traamos o arco CD com centro em A e o segmento ED tangente a esse arco em D. Ento, nos tringulos retngulos ACE e ADE, os catetos AC e AD so iguais e a hipotenusa AE comum, logo so tambm iguais os catetos CE e DE (= BD). Portanto, = AB = AD + BD = + BD = BC = BE + EC = BE + BD ou seja, = + BD = BE + BD. (1) (2)

155

RPM OBMEP

Se um segmento submltiplo comum de e , conclumos, por (1), que tambm submltiplo de BD. Daqui e de (2) segue-se que tambm submltiplo de BE. Provamos assim que se houver um segmento que seja submltiplo comum de = AB e = AC, ento o mesmo segmento ser submltiplo comum de BE e BD, segmentos esses que so a diagonal e o lado do quadrado BDEF. Ora, a mesma construo geomtrica que nos permitiu passar do quadrado original ao quadrado BDEF pode ser repetida com este ltimo para chegarmos a um quadrado menor ainda; e assim por diante, indefinidamente; e esses quadrados vo se tornando arbitrariamente pequenos, pois, como fcil ver, as dimenses de cada quadrado diminuem em mais da metade quando passamos de um deles a seu sucessor. Dessa maneira, provamos que o

segmento dever ser submltiplo comum do lado e da diagonal de um quadrado to pequeno quanto desejemos. Evidentemente, isso um absurdo! Somos, pois, levados a rejeitar a suposio inicial de que o lado AC e a diagonal AB do quadrado original sejam comensurveis. Conclumos, pois, que o lado e a diagonal de qualquer quadrado so grandezas incomensurveis C.Q.D. A descoberta dos incomensurveis representou, no 5o sculo a.C., uma derrota para os pitagricos. De fato, para eles o nmero era a essncia de tudo. Eles acreditavam na possibilidade de explicar todos os fenmenos do mundo sensvel em termos dos nmeros e de suas relaes, tanto na Geometria como na Msica, na Astronomia ou na Fsica, enfim, o nmero seria a essncia ltima do ser e de todos os fenmenos. Mas por nmero eles entendiam apenas o que chamamos hoje de nmeros naturais, ou inteiros positivos: 1, 2, 3, 4, .... Nem as fraes eram nmeros, j que elas apareciam como relaes entre grandezas da mesma espcie. Agora que haviam sido descobertas grandezas incomensurveis, estava claro que os nmeros (naturais) eram insuficientes at mesmo para definir a razo entre duas grandezas, o que se constitua num srio entrave Filosofia Pitagrica. Ao mesmo tempo em que essas coisas aconteciam, outros argumentos propostos pelos filsofos da poca dentre os quais os de Zeno so os mais famosos tambm apontavam dificuldades na suposta harmonia entre a Geometria e os nmeros. Tudo isso culminou numa crise no desenvolvimento da Matemtica, crise essa que s foi definitivamente superada com a criao da teoria dos nmeros reais (racionais e irracionais) no sculo passado, devido, sobretudo aos trabalhos do matemtico alemo Richard Dedekind (1831-1916). Uma consequncia da existncia de grandezas incomensurveis a existncia de pontos na reta sem abscissas racionais. A
RPM OBMEP

U
figura 4

156

De fato, com referncia figura 4, basta tomar OP = AO, onde AO a diagonal de um quadrado de lado unitrio OU. Como OP e OU so incomensurveis, no possvel expressar a razo OP/OU como um nmero racional. Que nmero seria a abscissa de P? Pelo teorema de Pitgoras, OA2 = OU2 + UA2. Como AO = OP e UA = OU = 1, obtemos OP2 = 2OU2 = 2 ou seja, OP = 2 . essa a abscissa de P, tomando OU como unidade de comprimento. interessante analisar essas questes do ponto de vista moderno dos nmeros como abscissas dos pontos de uma reta. Para maior simplicidade, vamos restringir-nos apenas a uma semirreta OU, tomando o segmento OU como unidade de comprimento (figura abaixo).
O
0

U
1

P
x

figura 5

Ento, todo ponto P da semirreta, que no seja a origem O, tem abscissa positiva x, que a razo OP/OU. Evidentemente, se todos os pares de segmentos OU e OP fossem comensurveis, bastariam os nmeros racionais no-negativos para caracterizar os pontos da semirreta, isto , os nmeros da forma m/n, com m e n inteiros, m > 0 e n > 0. E bom observar que isso condiz muito bem com nossa intuio geomtrica: afinal, esses nmeros ficam densamente distribudos ao longo da semirreta, de tal forma que entre dois deles h sempre uma infinidade de nmeros do mesmo tipo. Assim, entre os pontos A e B de abscissas 7 e 8 existem 9 nmeros do tipo
RPM OBMEP

n , 10 com n variando de 1 a 9. Isso porque dividimos o intervalo AB em 10 subintervalos de comprimento 1/10 cada um (figura 6). Mas podemos 7+

157

dividir esse intervalo em 100 subintervalos, cada um de comprimento 0,01; ou 1000 subintervalos, cada um de comprimento 0,001;
A
7 7,09 7,5 7,630589

B
8

figura 6

e assim por diante. Se, digamos, adotarmos a diviso em 1.000.000 subintervalos iguais, encontraremos entre A e B, 999.999 pontos com abscissas racionais do tipo
7+ n , 1.000.000

com n variando de 1 at 999 999. Na figura 6 ilustramos um desses pontos, aquele que tem abscissa 7,630598. Pois bem, vamos confiar ainda que provisoriamente na suposio de que todos os pontos da semirreta tenham abscissas racionais e ver onde isso nos leva. Uma primeira consequncia que os pontos da semireta formam um conjunto enumervel, pois o conjunto dos nmeros racionais enumervel (ver textos de Anlise Real). Se r1, r2, r3, ... uma enumerao dos racionais, faremos uma cobertura da semirreta por meio de segmentos, da seguinte maneira: cobrimos o ponto r1 com um segmento de comprimento c/2, centrado em r1; cobrimos r2 com um segmento de comprimento c/22, centrado em r2; fazemos o mesmo com r3, utilizando agora um segmento de comprimento c/23; com r4 utilizamos um segmento de comprimento c/24; e assim por diante. Dessa maneira a semirreta ficar toda coberta com uma famlia infinita de segmentos no necessariamente disjuntos. Vamos agora somar os comprimentos dos segmentos dessa famlia. Por simplicidade e para enfatizar a visualizao geomtrica colocamos os segmentos em fila, um em seguida ao outro e na ordem em que aparecem, como ilustra a figura 7. Isso o bastante para nos convencer de que a soma de todos os seus comprimentos exatamente igual a c, pois comeamos com um segmento de comprimento c/2, adicionamos sua metade, depois a metade deste ltimo e assim por diante.

158

RPM OBMEP

O
c/2 c/22 c/23

figura 7

O que acabamos de demonstrar uma impossibilidade! Certamente no possvel cobrir a semirreta com um a famlia de segmentos cuja soma total dos comprimentos seja um nmero finito c! (E o nmero c arbitrrio!) Afinal, a semirreta tem comprimento infinito! Para sairmos dessa contradio temos de voltar atrs em nossa hiptese inicial de que os pontos da reta numrica tm todos eles abscissas racionais. Em outras palavras, os nmeros racionais so insuficientes para marcar todos os pontos de uma reta; ou ainda, em termos mais inteligveis aos gregos da Antiguidade, existem segmentos AB e CD para os quais impossvel encontrar um segmento EF que seja submltiplo comum de AB e CD. Como se v, acabamos de estabelecer a existncia de segmentos incomensurveis com um raciocnio tpico da Anlise moderna! Ele certamente causaria, na Antiguidade, tanta controvrsia quanto causaram os famosos argumentos de Zeno. Talvez mais ainda, pois os argumentos de Zeno foram rebatidos por Aristteles que, atravs de seus escritos, f-los chegar at ns. Mas como rebater o argumento que demos acima sobre a cobertura dos pontos de abscissas racionais? Seria necessrio admitir a existncia de uma infinidade muito maior (uma infinidade no enumervel) de pontos sem abscissas racionais! claro que isso seria totalmente inaceitvel para quem j tinha srias objees ao infinito enumervel. Mesmo hoje muito surpreendente que se possa cobrir todos os pontos de abscissas racionais numa reta com uma famlia de segmentos cuja soma total dos comprimentos seja to pequena quanto desejemos! Esses pontos da reta sem abscissas racionais tm por abscissas
RPM OBMEP

nmeros irracionais (desde que esses nmeros sejam criados!) e 2 um deles, como decorre do argumento que demos antes referente figura 4. No entanto, para completar este artigo, vamos reproduzir aqui a demonstrao desse fato com um argumento puramente numrico e bem conhecido.

159

Comeamos supondo que existisse uma frao irredutvel m/n tal que
2 = m / n . Ento

2=

m 2 m2 = 2n2. n2

Daqui segue-se que m2 um nmero par, portanto o mesmo verdade para m, isto , m = 2r, sendo r outro nmero inteiro. Substituindo m = 2r em m2 = 2n2 obtemos 4r2 = 2n2 n2 = 2r2. Mas essa ltima relao nos diz que n2 nmero par, logo n tambm par. Chegamos a um absurdo, pois m/n frao irredutvel, no sendo possvel que m e n sejam ambos pares. Somos, assim, forados a rejeitar a suposio inicial de que
2 seja um nmero racional m/n.

A demonstrao que acabamos de dar est baseada num argumento que, segundo Aristteles, teria sido usado na descoberta de grandezas incomensurveis. um argumento que encerra um alto grau de abstrao, razo pela qual muitos historiadores da Cincia acreditam que a descoberta dos incomensurveis tenha ocorrido com um raciocnio mais concreto, como o argumento geomtrico da figura 3. Demonstraes como as que apresentamos, da incomensurabilidade do lado e da diagonal do quadrado, ou da irracionalidade de 2 , foram as primeiras demonstraes por reduo ao absurdo que se fizeram na Antiguidade. notvel que por volta de 400 a.C. a Matemtica j tivesse alcanado to avanado grau de sofisticao. O mesmo no aconteceu com outras cincias, como a Fsica, que somente no sculo XVII, com os trabalhos de vrios cientistas, notadamente Galileu e Newton, alcanaria desenvolvimento comparvel ao da Matemtica de dois milnios antes. Finalmente, um ltimo comentrio sobre a crise desencadeada com a descoberta dos incomensurveis. De imediato isso tornou impossvel falar em razo entre duas grandezas quando essas fossem incomensurveis.

160

RPM OBMEP

Havia a necessidade de se inventarem os nmeros irracionais, o que s ocorreu nos tempos modernos. Mas os gregos souberam contornar esse problema, logo na primeira metade do 4o sculo a.C., e com muita genialidade! Foi Eudoxo (408? 355? a.C.), da escola de Plato, quem desenvolveu, de maneira brilhante, uma teoria das propores, com a qual foi possvel superar a dificuldade dos incomensurveis, usando apenas os nmeros inteiros positivos. Mas isso uma outra histria...
Adaptado do artigo Grandezas incomensurveis e nmeros irracionais Geraldo vila, RPM 05.

161

RPM OBMEP

A outra face da moeda honesta

No outro dia, brincava com a minha filha Ana Letcia o seguinte jogo: cada uma tinha um peo avanando por uma linha de um tabuleiro de xadrez. A regra para avanar baseava-se no lanamento de uma moeda honesta: se sasse cara, o peo avanava uma casa, se fosse coroa, avanava duas casas. Quem chegasse primeiro ao fim (ou passasse) da linha, ganhava. O jogo muito rpido, de modo que depois de algumas revanches, a Ana me disse: u, no ltimo lanamento s sai coroa?. De fato, comecei a perceber que, no ltimo lanamento, o nmero de coroas era muito maior que o de caras. No entanto, ao observar todos os lanamentos, o nmero de caras e o de coroas eram praticamente iguais, o que indica a honestidade da moeda. A Ana com seus sete anos foi brincar de outra coisa e eu fiquei pensando no assunto: o lanamento final poder ser cara somente se o peo estiver na penltima casa, j que com cara o peo avana apenas uma casa. Enquanto que poder ser coroa, tanto se o peo estiver na penltima quanto na antepenltima casa. Portanto, a face coroa tem realmente mais chance de aparecer no ltimo lanamento que a face cara!!!

162

RPM OBMEP

Mais formalmente, denotemos por N o nmero de lanamentos necessrios para chegar oitava casa (ou passar), por SN1 a posio do peo antes do lanamento final, e por XN a face nele observada. Se SN1 = 7, ento o lanamento seguinte pode ser cara ou coroa, com mesma chance. Por outro lado, SN1 = 6 implica que o lanamento seguinte deva ser coroa, seno este no seria o ltimo. Assim, pela lei da probabilidade total, P ( X N = cara ) = P ( S N 1 = 7) P( X N 1 2 1 = coroa ) = P ( S N 1 = 7) + P ( S N 1 = 6) 1 2 > P ( X N = cara ). (1)

A ltima desigualdade encerra o assunto, mas eu ainda queria conhecer a probabilidade de obter coroa no ltimo lanamento. O evento [SN1 = 6] ocorre se e somente se forem observadas as seguintes seqncias nos lanamentos: {(2, 2, 2, 2), (1, 1, 2, 2, 2), (1, 1, 1, 1, 2, 2), (1, 1, 1, 1, 1, 1, 2)} ou suas possveis permutaes (lembrando que o ltimo lanamento deve ser coroa). Portanto,
P ( S N 1 = 6) = 4 1 5 1 1 + 5 + 6 + 7 , 4 2 2 1 2 2 2 1

que aproximadamente 1/3. Como a penltima posio s pode ser 6 ou 7, ento P(SN1 = 7) aproximadamente 2/3. Substituindo esse resultado em (1), temos que a probabilidade de obter coroa no lanamento final aproximadamente 2/3. A primeira concluso disso tudo que minha filha algo exagerada em seus comentrios. A segunda que moeda honesta quando chega ao fim da linha... Comentrios O problema abordado neste texto uma verso discreta do conhecido Paradoxo do tempo esperado, definido originalmente no contexto de processos de renovao a tempo contnuo.
RPM OBMEP

163

A novidade nesta apresentao que inspira a construo fsica de um mecanismo que permite visualizar esse resultado: por exemplo, mediante uma trilha na qual se avana de acordo com o lanamento de um dado equilibrado, ou a prpria corrida de oito passos no tabuleiro de xadrez. A reao usual das pessoas de espanto ao constatar o desequilbrio da distribuio obtida, ou de dvida a respeito da honestidade da moeda. Mesmo sendo contra-intuitivo, o resultado natural se levarmos em considerao que h outra varivel aleatria envolvida no processo, que o nmero de lanamentos necessrios para se alcanar o fim da linha, N. Se bem verdade que o resultado de qualquer lanamento tpico, XN, uniforme no conjunto {cara, coroa} e independente dos demais lanamentos, o mesmo no ocorre com o lanamento final, XN. A varivel aleatria XN no independente dos lanamentos anteriores, depende da penltima posio. Uma outra forma de entender o resultado que mais fcil cobrir a ltima casa com um passo grande (de duas casas) que com um passo pequeno (de uma casa). Assim, a diferena entre a frequncia de caras e coroas fica ainda mais evidente se avanarmos uma casa ao obter cara e trs ao obter coroa, por exemplo. No caso extremo de avanarmos uma casa com cara e oito casas com coroa, a chance de obter cara no ltimo lanamento 1/256, menos que 4 em 1000!!!
Adaptado do artigo A outra face da moeda honesta Laura L. R. Rifo, RPM 64.

164

RPM OBMEP

Nmeros de regies: um problema de contagem

Muitos problemas em Matemtica envolvem processos adequados de contagem que, frequentemente, conduzem a frmulas gerais extremamente teis; por exemplo, para contar de quantas maneiras podemos combinar n objetos em grupos de r desses objetos, usamos a conhecida frmula que d o nmero de combinaes de n objetos tomados r a r, a saber:

n n! . C ( n, r ) = = r r !(n r )!
Vamos analisar um problema de contagem do nmero de regies no plano que pode ser resolvido de maneira direta, simples e interessante. Trata-se do seguinte: Considere 100 pontos distribudos sobre uma circunferncia, de tal modo que o segmento ligando dois quaisquer desses pontos no passe pelo ponto de interseco de outros dois segmentos. Calcular o nmero R de regies obtidas no crculo quando todos os 100 pontos estiverem ligados.
RPM OBMEP

Inicialmente, tentamos resolver o problema com um nmero menor de pontos. Examinando os casos 2, 3, 4 e 5 pontos, temos:

165

R2 = 2 Observamos que:

R3 = 4
figura 1

R4 = 8

R5 = 16

2 3 4 5

pontos: pontos: pontos: pontos:

21 22 23 24

regies; regies; regies; regies.

Os resultados levam a acreditar que 6 pontos fornerceriam 25 = 32 regies, logo 100 pontos forneceriam 299 regies, e, por analogia (incorreta, como veremos) n pontos determinariam 2n1 regies! Mas, ao verificar diretamente o que acontece com 6 pontos, vemos que ficam determinadas 31 regies, e no 32. Logo, a generalizao pretendida no verdadeira.
1 A B 2

5
figura 2

RPM OBMEP

Como determinar uma frmula que fornea o nmero de regies obtidas com 100 (ou um outro nmero qualquer) de pontos? Soluo 1 Os segmentos ligando dois a dois os 100 pontos sero chamados diagonais; como para cada dois pontos temos uma diagonal, o nmero

166

100 delas C(100, 2) = , e o nmero de pontos de interseco das 2 100 diagonais C(100, 4) = , visto que cada 4 pontos determinam 4 duas diagonais, as quais tm um ponto em comum.

Vamos descrever um processo que nos permite obter o nmero de regies pela eliminao sucessiva de diagonais. Ao retirarmos uma das diagonais, o nmero de regies vai diminuir, visto que duas regies que tm em comum um segmento da diagonal retirada fundem-se em uma nica regio. Por exemplo, na figura 2, a retirada da diagonal D12, que liga os pontos 1 e 2, faz com que as regies A e B se transformem em uma nica regio; a retirada da diagonal D35 transforma em quatro as oito regies que tm partes dessa diagonal como arestas. Podemos observar que, ao retirarmos uma diagonal, o nmero de regies decresce conforme o nmero de pontos de interseco dessa diagonal com aquelas que ainda no foram removidas, mais um. Com efeito, esse o nmero de segmentos nos quais os referidos pontos de interseco dividem a diagonal, e a remoo de cada um desses segmentos transforma duas regies em uma. Assim, a remoo da diagonal D12, que no tem ponto de interseco com as demais, produz um decrscimo de apenas um no nmero total de regies; j a retirada da diagonal D35, que tem 3 pontos de interseco com as demais diagonais, produz um decrscimo de 4 regies. Notemos que, no processo de retirada sucessiva das diagonais, considera-se o nmero de pontos de interseco de cada diagonal com aquelas que ainda no foram retiradas; no final do processo, ao serem retiradas, sucessivamente, todas as diagonais, tal nmero igual ao nmero total de pontos de interseco de todas as diagonais, ou seja
100 C(100, 4) = ; ao mesmo tempo, o nmero de regies decresce at 4 reduzir-se a uma nica regio, quando todas as diagonais tiverem sido

167

RPM OBMEP

eliminadas. Podemos ento concluir que o nmero de regies eliminadas no processo de retirada sucessiva de todas as diagonais dado pelo nmero total de pontos de interseco de todas as diagonais, ou seja
100 C(100, 4) = , acrescido de tantas parcelas iguais a 1 quantas so 4 100 as diagonais, ento, C(100, 2) = . Portanto, o nmero inicial de 2 regies, que igual ao nmero de regies eliminadas mais uma, a que restou no final do processo, dado por

100 100 100 R100 = + + = 3.926.176. 0 2 4


Observe que, para n pontos, temos a mesma expresso, apenas trocando o 100 por n. E, para 6 pontos, a frmula obtida fornece
6 6 6 R6 = + + = 31, como havamos verificado! 0 2 4

Soluo 2 (No volume anterior (2007) do nmero especial da RPM para a OBMEP esta soluo, com menos detalhes, aparece na pgina 93.) Em Geometria, uma das frmulas mais notveis a chamada frmula de Euler, que estabelece uma relao entre o nmero de vrtices, arestas e faces de um poliedro: V A + F = 2. Mostraremos, em seguida, como a frmula que fornece o nmero de regies determinadas por n pontos, distribudos em uma circunferncia, pode ser obtida a partir da frmula de Euler, o que era de se esperar, pois a demonstrao mais conhecida da frmula de Euler, devida a Cauchy, comea removendo uma face do poliedro e deformando a parte restante em uma regio plana que um polgono subdividido pelas arestas do poliedro. Para poliedros planos, como o da figura 2, obtidos pela interligao de n pontos na circunferncia, a frmula de Euler se reduz a V A + F = 1. (1) Vamos calcular, separadamente, V, A e F em funo de n e substitu-los na frmula (1) para obter Rn .

168

RPM OBMEP

Clculo do nmero de vrtices Para cada 4 vrtices na circunferncia existem dois, e apenas dois, segmentos que se cruzam, e portanto determinam um vrtice chamado

n interno, de modo que o nmero desses vrtices C (n, 4) = , ou 4 n seja: V = n + . 4


(2)

Clculo do nmero de arestas Cada vrtice externo contribui com (n 1) arestas, cada vrtice interno com 4 arestas e como cada aresta contm 2 vrtices:

n 2 A = n(n 1) + 4 e, portanto, 4 A= n n n n(n 1) + 2 = + 2 . 2 4 2 4


(3)

Clculo do nmero de regies O nmero Rn obtido acrescentando-se a F o nmero n de regies compreendidas entre o poliedro plano e a circunferncia, de modo que F = Rn n. (4) Basta agora substituir (2), (3) e (4) na frmula (1) para se obter o valor de Rn , na mesma expresso da soluo 1. Observamos que o argumento usado na soluo 1 serve tambm para demonstarr a frmula de Euler. Adaptado do artigo
RPM OBMEP

Nmero de regies: um problema de contagem Antnio C. Patrocnio, RPM 12.

169

Intuio e Probabilidade

Em muitas situaes do cotidiano estamos interessados no clculo de determinadas probabilidades. Ocorre que, em certos casos, especialmente aqueles envolvendo o conceito de probabilidade condicional, esses clculos levam a concluses que podem contrariar a intuio. Apresentamos, neste texto, um exemplo dessa situao. Num pas, 10% da populao portadora de um vrus. Um teste para detectar a presena do vrus d 90% de acertos quando aplicado a portadores e 80% de acertos quando aplicado a no portadores. Qual o percentual de pessoas realmente portadoras do vrus, dentre aquelas que o teste classificou como portadoras? Vejamos uma soluo que no cita teoremas de Probabilidade ou Estatstica. Considere que o teste foi aplicado aos I habitantes do pas. O nmero de testes que indicou a presena do vrus foi: 0,9 0,1 I + 0,2 0,9 I = 0,09 I + 0,18 I = 0,27 I, sendo que a primeira parcela representa os 90% que so realmente portadores e a segunda parcela representa os 20% que no so portadores. Logo, do total, 0,27 I, so portadoras 0,09 I.
RPM OBMEP

170

Assim, so realmente portadoras do vrus 0,09I / 0,27 I = 1/3 33,3% das pessoas que o teste classificou como portadoras. Esse nmero no mnimo curioso e mostra que uma pessoa que fez o teste e foi classificada como portadora tem grande possibilidade de ser um falso-positivo (normalmente, quando uma pessoa faz um teste desse tipo e o resultado positivo, os mdicos recomendam um novo teste). Por exemplo, o nmero de testes que indicaram a ausncia do vrus foi 0,73 I e, dentre esses, 0,72 I no so portadores, o que d 0,72 I / 0,73I 98,6% de no portadores dentre os classificados como no portadores. Algumas variaes nos dados tambm originam resultados interessantes. Por exemplo: Se 0,5% da populao portadora e o teste acerta em 98% dos casos, ento somente 20% das pessoas que o teste classificou como portadoras so realmente portadoras.
Adaptado do artigo Intuio e Probabilidade Raul F. W. Agostino, RPM 27.

Resposta do Desafio
Como cada um dos pontos marcados na figura pertence a duas circunferncias, a soma total dos nmeros colocados nas quatro circunferncias (contando cada circunferncia em separado) igual a 2(1 + 2 + 3 + 4 + 5 + 6 + 7 + 8 + 9) = 90.
RPM OBMEP

Para que a soma S dos nmeros pertencentes circunferncia externa seja exatamente igual soma (S) dos nmeros pertencentes a cada uma das circunferncias internas, deveramos ter 90 = 4S, o que impossvel, uma vez que 90 no divisvel por 4. O problema no tem soluo.

171

Problemas I: problemas interessantes


com nmeros primos

P1. Escreva o nmero 91 como soma de dois nmeros primos. P2. Eu e meu irmo caula temos idades entre 10 e 20 anos e hoje nossas idades so expressas ambas por nmeros primos, fato que se repetir pela prxima vez daqui h 18 anos. Determine minha idade sabendo que a idade de nosso irmo mais velho, que, hoje, tambm um nmero primo, uma unidade maior do que a soma das nossas idades. P3. Uma equao do 2o grau, cujos coeficientes so todos nmeros primos, pode apresentar duas razes iguais? P4. Os nmeros a, b e logba podem ser todos primos? A resposta aos dois problemas acima no, e eles no devem apresentar maiores dificuldades ao leitor. P5. Quantos pontos da reta y = x + 51 so tais que as suas duas coordenadas so nmeros primos? Observe-se que, trocando o nmero 51 por outro valor, o problema pode tornar-se muito mais difcil. Para a reta y = x + 2 somos conduzidos ao conceito de primos gmeos (diferem por 2 unidades). At hoje um problema em aberto saber se existem ou no infinitos pares de primos gmeos. Se tomssemos a reta y + x = 40 obteramos seis solues: (3, 37), (37, 3), (11, 29), (29, 11), (17, 23) e (23, 17), todas no primeiro quadrante e que podem ser obtidas por inspeo direta.
RPM OBMEP

Neste instante natural lembrar que a famosa conjectura de Goldbach todo nmero natural par pode ser escrito como soma de dois nmeros primos ainda no foi provada e nem se encontrou um contra-exemplo. P6. As medidas dos lados de um tringulo retngulo (numa mesma unidade) podem ser nmeros primos?

172

P7. Para quantos pontos da circunferncia x2 + y2 = 361 as duas coordenadas so nmeros primos? P8. Para quantos pontos da circunferncia x2 + y2 = 461 as duas coordenadas so nmeros inteiros? Esse problema se assemelha ao anterior, embora seja mais difcil que ele. Para resolv-lo sugerimos a leitura de um artigo de Gilberto Garbi, Outro belo teorema de Fermat, publicado na RPM 38. P9. Determine as medidas, em graus, dos ngulos internos de um tringulo acutngulo, sabendo que elas so expressas por nmeros primos. A mesma pergunta sem a hiptese de ser acutngulo exige um pouco mais de trabalho. P10. Quantos divisores positivos possui o nmero 2 420? Esse exerccio uma aplicao clssica do Teorema Fundamental da Aritmtica e do Princpio Fundamental da Contagem. P11. Verifique que todos os n 1 nmeros da sequncia n! + 2, n! + 3, ..., n! + n so nmeros compostos so nmeros compostos, isto , nenhum deles um nmero primo. P12. Quantos so os nmeros naturais, de 1 a 100, que podem ser escritos como um produto de dois nmeros naturais distintos entre si e diferentes de 1? P13. Apresente algum nmero natural n para o qual o valor numrico p(n) do polinmio p(n) = x2 + x + 41 no seja um nmero primo. P14. Quantos polgonos regulares, com nmero par de lados, podem ter todas as diagonais expressas (numa mesma unidade) por nmeros primos? P15. H dois anos, ano em que finalmente conclu meu Doutorado em Matemtica, nasceu meu segundo filho e ocorreu uma notvel coincidncia: eu e meus dois filhos passamos a fazer aniversrio no mesmo dia do ano. A partir da outras coincidncias aconteceram. No
RPM OBMEP

173

ano passado nossas trs idades foram representadas por quadrados perfeitos e hoje, dia em que estamos comemorando mais um aniversrio, percebo que nossas idades so representadas por trs nmeros primos. Supondo que vivamos cem anos cada um, pergunto: qual minha idade hoje? Nos prximos anos, quantas vezes todas as nossas idades voltaro a ser representadas por nmeros primos?
Adaptado do artigo Os primos esquecidos Chico Nery e Cludio Possani, RPM.

174

RPM OBMEP

Problemas II: problemas do PISA*


*Programme for International Student Assessment Programa Internacional de Avaliao de Alunos 2000/2003 http://www.inep.gov.br/internacional/pisa/novo (julho de 2009)

Questes Estantes Para construir uma estante completa, um marceneiro precisa do seguinte material: 4 pranchas grandes de madeira, 6 pranchas pequenas de madeira, 12 braadeiras pequenas, 2 braadeiras grandes e 14 parafusos. O marceneiro possui em estoque 26 pranchas grandes de madeira, 33 pranchas pequenas de madeira, 200 braadeiras pequenas, 20 braadeiras grandes e 510 parafusos. Quantas estantes completas o marceneiro poder fazer? Bombons coloridos A me de Roberto permite que ele pegue um bombom de um saco. Ele no consegue ver os bombons. O grfico ao lado mostra o nmero de bombons de cada cor contidos no saco. Qual a probabilidade de Roberto pegar um bombom vermelho? A) 10% B) 20% C) 25% D) 50% Carpinteiro Um carpinteiro tem 32 metros de tbua para cercar um canteiro em uma horta. Ele est pensando em utilizar um dos seguintes modelos para o canteiro.
RPM OBMEP

175

Na tabela abaixo, faa um crculo em Sim ou No para cada modelo, indicando se ele pode ou no ser feito com 32 metros de tbuas.
Modelo de canteiro modelo A modelo B modelo C modelo D Usando esse modelo, o canteiro pode ser construdo com 32 metros de tbua? Sim/No Sim/No Sim/No Sim/No

Dados O desenho da direita representa dois dados. Os dados so cubos com faces numeradas de acordo com a seguinte regra: em um mesmo dado, o nmero total de pontos de duas faces opostas sempre sete. Questo 1 direita, vemos trs dados empilhados um sobre o outro. O dado 1 possui quatro pontos na face superior. Quantos pontos h, no total, nas cinco faces horizontais que no podemos ver (face inferior do dado 1, faces superiores e inferiores dos dados 2 e 3)? Questo 2 Voc pode fazer um dado, cortando, dobrando e colando uma cartolina. Isso pode ser feito de diversas maneiras. Na figura a seguir, h quatro

176

RPM OBMEP

opes que podem ser utilizadas para fazer dados. Quais dos formatos podem ser dobrados para formar um dado que obedea regra na qual a soma das faces opostas 7? Para cada formato, faa um crculo em Sim ou No na tabela abaixo.

Formato

Obedece regra na qual a soma dos lados opostos 7?

I II III IV

Sim/No Sim/No Sim/No Sim/No

Sequncia em escada Roberto constri uma sequncia com o formato de uma escada, utilizando quadrados. Ele segue as seguintes etapas:

Como voc pode ver, ele utiliza um quadrado na Etapa 1, trs quadrados na Etapa 2 e seis na Etapa 3. Quantos quadrados ele vai utilizar na Etapa 4? Terremoto Foi divulgado um documentrio sobre terremotos e a frequncia com que eles ocorrem. Essa reportagem inclui uma discusso sobre a previsibilidade dos terremotos. Um gelogo declarou: Nos prximos vinte anos, a probabilidade de que ocorra um terremoto em Zedpolis de dois sobre trs. Qual das opes a seguir exprime melhor o significado da declarao do gelogo?

177

RPM OBMEP

A) 2/3 20 = 13,3, portanto no perodo de 13 a 14 anos, a partir de hoje, haver um terremoto em Zedpolis. B) 2/3 maior que 1/2, portanto podemos ter certeza de que haver um terremoto em Zedpolis nos prximos 20 anos. C) A probabilidade de haver um terremoto em Zedpolis nos prximos 20 anos maior do que a probabilidade de no haver um terremoto. D) No se pode afirmar o que acontecer porque ningum pode ter certeza de quando ocorrer um terremoto. Feira de Vero Uma barraca de uma feira de vero prope um jogo no qual se utiliza primeiro uma roleta. Em seguida, se a roleta parar em um nmero par, o jogador poder pegar uma bolinha de gude de dentro de um saco. A roleta e as bolinhas de gude contidas no saco esto representadas na ilustrao. Os prmios so distribudos s pessoas que pegam uma bolinha de gude preta. Qual a probabilidade de Sueli ganhar um prmio? A) Impossvel. B) Pouco provvel. C) Cerca de 50% de probabilidade. D) Muito provvel. E) Certeza. Bate-papo pela Internet Mark (de Sydney, na Austrlia) e Hans (de Berlim, na Alemanha) comunicam-se com frequncia por meio de uma sala de bate-papo da Internet. Para isso, eles precisam conectar-se Internet ao mesmo tempo. Para determinar um horrio apropriado para bater papo, Mark consultou uma tabela de fusos horrios do mundo e encontrou o seguinte:

178

RPM OBMEP

Questo 1 Que horas so em Berlim quando so 19 horas em Sydney? Questo 2 Mark e Hans no podem bater papo das 9h s 16h30 de seus horrios locais respectivos, porque eles devem ir para a escola. Alm disso, no podero bater papo entre 23h e 7h porque estaro dormindo. Qual seria um bom horrio para Mark e Hans baterem papo? Escreva os horrios locais na tabela ao lado: Assaltos Um reprter de TV apresentou o grfico ao lado e disse: O grfico mostra que, de 1998 para 1999, houve um grande aumento no nmero de assaltos. Voc considera que a afirmao do reprter uma interpretao razovel do grfico? D uma explicao que justifique a sua resposta. Colnia de frias Os Servios Comunitrios de Zedpolis esto organizando um acampamento para um perodo de cinco dias. Inscreveram-se 46 crianas (26 meninas e 20 meninos) e 8 adultos (4 homens e 4 mulheres) apresentaramse como voluntrios para acompanh-las e organizar o acampamento.
Tabela 1: Adultos Sra. Marlia Sra. Carolina Sra. Graa Srta. Ktia Sr. Slvio Sr. Nelson Sr. William Sr. Pedro Tabela 2: Dormitrios Nome Nmero de camas Vermelho Azul Verde Roxo Laranja Amarelo Branco 12 8 8 8 8 6 6 Local Sydney Berlim Horrio

179

RPM OBMEP

Regra dos dormitrios 1. Os meninos e as meninas devem dormir em dormitrios separados. 2. Em cada dormitrio deve dormir, pelo menos, um adulto. 3. O(s) adulto(s) que ficar(em) nos dormitrios deve(m) ser do mesmo sexo que as crianas. Preencha a tabela abaixo, distribuindo as 46 crianas e os 8 adultos nos dormitrios, de maneira que todas as regras sejam obedecidas.
Nome Vermelho Azul Verde Roxo Laranja Amarelo Branco Nmero de meninos Nmero de meninas Nome(s) do(s) adulto(s)

O leitor deve ter observado que, em linhas gerais, as questes so diferentes das que normalmente so apresentadas aos nossos alunos em sala de aula. Elas exigem pouco contedo, pouca memria, mas, nas palavras dos idealizadores do PISA, examinam a capacidade dos alunos de analisar, raciocinar e refletir ativamente sobre seus conhecimentos e experincias, enfocando competncias que sero relevantes para suas vidas futuras. No site indicado, alm das questes de 2003, esto algumas questes de 2000, e tambm uma descrio dos mecanismos utilizados para a seleo dos estudantes que participaro do teste, normas de avaliao, algumas explicaes sobre o mau desempenho dos brasileiros e outros temas correlatos.

180

RPM OBMEP

PROBLEMAS III

A numerao entre parnteses a original dos exemplares da RPM 1. (184) Os nmeros reais a, b e c so tais que a + b + c = 3, a2 + b2 + c2 = 13 e a3 + b3 + c3 = 27. Determine a4 + b4 + c4. 2. (205) Trs aranhas caminham pelos lados de um tringulo ABC e movimentam-se de modo que em qualquer instante formam um tringulo e o baricentro de todos os tringulos formados sempre o mesmo ponto (fixo) P. Sabendo-se que uma das aranhas percorre todo o tringulo ABC, mostrar que P tambm o baricentro do tringulo ABC. 3. (207) Um rapaz esqueceu o ltimo algarismo do telefone da namorada e resolveu tentar falar com ela escolhendo ao acaso o ltimo dgito. Se ele est num telefone pblico e s tem duas fichas, qual a probabilidade de que ele consiga conversar com a namorada? 4. (219) Dados x e y nmeros inteiros positivos, mostre que, se x2 + y2 + xy divisvel por 10, ento divisvel por 100. 5. (220) Considere duas retas paralelas que distam a entre si e um quadrado ABCD, de lado a, situado no plano das paralelas numa posio tal que os vrtices A e C estejam em lados opostos do plano dividido pela faixa das paralelas. Calcule a soma dos permetros dos tringulos sombreados. 6. (236) Achar todos os nmeros m e n naturais que resolvam
RPM OBMEP

n2n1 + 1 = m2. 7. (265) Seja ABC um tringulo tal que = 60o. Seja H o seu ortocentro e J AC tal que AJ = 2JC e JC = JH. Mostre que o tringulo ABC equiltero.

181

8. (266) Seja p(x) o polinmio de grau 2007, com coeficientes reais,

2 2 2007 + cos ) . 2007 2007 Determine o resto da diviso de p(x) por x2 + 1. p ( x) = ( x sen
9. (267) Numa folha quadrada de papel desenhe ou dobre um ngulo , marque a metade da folha e a metade da metade. Dobre a folha de modo que A caia em um ponto A pertencente a r e B em um ponto B pertencente a s (ver figura). Marque os pontos A, B e C, o correspondente de C na dobra. Prove que AB, AA e AC trisseccionam o ngulo .

10. (268) Seja f: N N uma funo tal que f(f(x)) = x para todo x N. a) Mostre que f bijetora. b) Exiba uma funo f com a propriedade acima e tal que f(x) x para todo x N. 11. (270) Dispe-se de 2007 moedas viciadas M1, M2, ..., M2007. Sabe-se que, em um lanamento, a probabilidade de se obter cara na moeda Mi, i = 1, 2, ... 2007,
1 . Se as 2007 moedas (2i + 1)

so lanadas simultaneamente, qual a probabilidade de que o nmero de caras obtidas seja mpar?

A
RPM OBMEP

12. (271) Na figura, AD = 2BD. Determine .

D B
45
o

15

182

13. (272) Encontre as razes reais da equao

x + 2 x 1 + x 2 x 1 = 2.
14. (274) Prove que: a) num tringulo retngulo a medida da mediana relativa hipotenusa igual metade da medida da hipotenusa. b) todo tringulo pode ser decomposto em n tringulos issceles, para todo n > 4. 15. (276) Determine todas as funes f: R R satisfazendo 2f(x) + f(1 x) = x2008 para todo x R. 16. (277) Qual a maior potncia de 2 que divide 32008 1? 17.
(280) (Jogo de Kontsevich) Consideremos o tabuleiro infinito, ilustrado na figura, que ocupa o primeiro quadrante. Inicialmente, h 6 peas nas casas sombreadas no canto inferior esquerdo. Um movimento consiste em escolher uma pea sem vizinhos nas casas imediatamente acima e direita e substituir essa pea por duas, colocandoas nas casas vizinhas vagas:

movimento

O objetivo do jogo , a partir da configurao inicial, realizar uma sequncia de movimentos de modo a deixar as 6 casas sombreadas sem peas. a) Atribua o peso 2ij para a casa na posio (i, j), como mostra a figura (a casa inicial, inferior esquerda, situa-se na posio (0,0)). Mostre que a soma dos pesos das casas ocupadas por peas, depois de se realizarem movimentos, constante, isto , no se altera aps qualquer sequncia de movimentos.
RPM OBMEP

183

1 8 1 16 1 32 1 64 14 12
1

1 8 1 16 1 32 14 12 1 8 1 16 14 18

... ... ... ... ...

b) Determine a soma de todos os pesos do tabuleiro. c) Mostre que impossvel realizar o objetivo do jogo: no existe nenhuma sequncia de movimentos que deixa as 6 casas iniciais, sombreadas, livres de peas! 18. (281) Simplifique:
2 n 2008 n

n3 1
3

+1

23 1 33 1 43 1 20083 1 . 3 . 3 .. . 23 + 1 3 + 1 4 + 1 20083 + 1

19. (282) Prove que, se x e y so inteiros tais que N = (x + 6y)(2x + 5y)(3x + 4y) mltiplo de 7, ento N mltiplo de 343. 20. (284) Dado um ngulo qualquer AOB com AO = BO, dividimos AB em 3 partes iguais: AC = CD = BD. Pergunta-se: existe algum

AOB para o qual os ngulos AOC , congruentes?

COD e BOD

so

184

RPM OBMEP

Solues dos Problemas I


P1. Os alunos no devero ter dificuldade em perceber que, como a soma de dois mpares par e como 2 o nico primo par, os nmeros so 2 e 89. Alis, esse pode ser um bom momento para recordar com os alunos os testes de primalidade, para verificar que 89, efetivamente, primo. P2. As duplas de primos entre 10 e 20 so: 11 e 13, 11 e 17, 11 e 19, 13 e 17, 13 e 19 e 17 e 19. Como a soma dos nmeros adicionada de 1 deve resultar um primo, descarto as duplas 11 e 13 e 13 e 19. Como daqui a 18 anos as idades voltam a ser representadas por nmeros primos, descarto as duplas que incluem o 17. Resta apenas uma possibilidade: minha idade 19 anos e a do meu irmo 11 anos. P3. Para que a equao ax2 + bx + c = 0 (com a, b e c primos) admita duas razes iguais, devemos ter b2 4ac = 0 ou b2 = 4ac, o que implica b2 par. Logo, b tambm par e, como primo, b = 2. De b2 = 4ac, com b = 2, temos ac = 1, o que absurdo para a e c primos. Portanto, nas condies impostas, a equao no pode admitir duas razes iguais. P4. Seja x = logba, portanto bx = a. Se b e x so nmeros primos, ento bx no primo; logo, a no primo. P5.
RPM OBMEP

Se x = 2 , temos y = x + 51 = 53, que primo. Se x for qualquer outro primo, ser um nmero mpar, implicando y par maior que 2, logo, no primo. Assim, existe um nico par, (2, 53), da reta de equao y = x + 51 que tem ambas as coordenadas dadas por nmeros mpares.

185

P6. Soluo: A resposta no. Do teorema de Pitgoras temos a igualdade a2 = b2 + c2. Sendo a, b e c primos, no podem ser todos mpares e, como a > b e a > c, devemos ter b = 2 ou c = 2. Digamos c = 2. Teremos ento: a2 b2 = 4 (a + b)(a b) = 4 e analisando os possveis valores de a + b e a b, que so 1, 2 ou 4, conclumos que a situao impossvel. P7. Se x e y satisfazem a equao x2 + y2 = 361, sendo 361 mpar, devemos ter x par e y mpar ou x mpar e y par. Se x par e primo, ento, x = 2; logo, y2 = 357 e y no , ento, um nmero inteiro. Do mesmo modo verificamos ser impossvel ter y par e x mpar; logo, nenhum ponto da circunferncia de equao x2 + y2 = 361 tem ambas as coordenadas dadas por nmeros primos. P8. Observamos, inicialmente, que 461 = 100 + 361 = 102 + 192, logo os seguintes oito pontos, de coordenadas inteiras, pertencem circunferncia de equao x2 + y2 = 461: (10, 19), (10, 19), (10, 19), (10, 19), (19, 10), (19, 10), (19, 10) e (19, 10). Alm disso, sendo 461 um nmero primo que dividido por 4 deixa resto 1, o resultado de Fermat, todo nmero primo que dividido por 4 deixa resto 1 pode ser escrito como soma dos quadrados de dois nmeros inteiros, de modo nico, a menos da ordem, prova que esses oito so os nicos pontos de coordenadas inteiras pertencentes circunferncia. P9. Se a + b + c = 180, com a, b e c primos, no possvel ter a, b e c mpares; logo, pelo menos um deles, digamos o a, deve ser igual a 2, o que implica b + c = 178. Podemos ter b = c = 89, que primo e, por verificao direta, mostra-se que no h outra possibilidade, j que o tringulo, sendo acutngulo, implica b < 90 e c < 90.

186

RPM OBMEP

Sem a hiptese de o tringulo ser acutngulo, obtemos, por tentativa, as possibilidades: 5 e 173, 11 e 167, 29 e 149, 47 e 131 e 71 e 107. P10. Decompondo 2420 em fatores primos encontramos: 2420 = 22 5 112. Os divisores positivos de 2420 so todos do tipo 2 5 11 com valendo 0, 1 ou 2, valendo 0 ou 1 e valendo 0, 1 ou 2. Logo, pelo Princpio Fundamental da Contagem, a quantidade de divisores positivos de 2420 : 3 2 3 = 18. P11. Observemos que: n! + 2 divisvel por 2, n! + 3 divisvel por 3, ..., n! + n divisvel por n, e assim sendo, nenhum deles primo. Para valores grandes de n essas sequncias de nmeros naturais consecutivos so chamadas desertos de nmeros primos. P12. De 1 a 100 temos 100 nmeros. Para obtermos a resposta nossa pergunta, subtramos de 100 o nmero de primos entre 1 e 100, que 25; o nmero de quadrados de nmeros primos, que 4, e o nmero 1. A resposta 70. P13. Para x = 40, x2 + x + 41 = 402 + 40 + 41 = 40(40 + 1) + 41 = 40.41 + 41 = 41(40 + 1) = 412 que no primo. Tambm para x = 41, x2 + x + 41 = 412 + 41 + 41 = 41(41 + 1 + 1) = 41.43, que no primo. Prova-se que para qualquer valor inteiro de x, 40 < x < 39, tem-se x2 + x + 41 igual a um nmero primo (ver RPM 09, p. 33). P14. 1. Como podemos construir um quadrado com qualquer tamanho, podemos constru-lo com suas diagonais medindo: 2 ou 3 ou 5, ...,
RPM OBMEP

ou qualquer outro valor primo. O lado do quadrado medindo p faz a diagonal medir p, para qualquer p primo.

2 2

187

2. J no caso do hexgono regular, se traarmos duas diagonais, uma passando pelo centro e a outra no, vejamos o que acontece. F Sendo ABCDEF um hexgono regular, no tringulo ABD temos

D C
60
o

BD 3 A B , implicando que as me= sen60 = AD 2 didas das diagonais BD e AD no podem ser simultaneamente expressas por nmeros inteiros, logo no podem ser ambas nmeros primos. 3. Para qualquer outro polgono regular com nmero P par de lados (octgono, decgono, ... etc), se considerarmos dois vrtices P e Q diametralmente opostos e um vrtice M no consecutivo de P nem M de Q, eles determinaro o tringulo PQM, retngulo O em M, cujos lados so trs das diagonais desse polgono e, como j foi provado no P6, as medidas Q desses trs lados no podem ser simultaneamente expressas por nmeros primos. Concluso, apenas o quadrado pode ter todas as suas diagonais com medidas expressas por nmeros primos.

P15. No ano passado meu filho caula certamente tinha 1 ano de idade. Meu outro filho tinha 4 ou 16 anos e eu, o pai, 36 anos. Portanto, hoje, minha idade 37 anos. Quando a minha idade mpar, a do meu caula par e vice-versa; portanto, nunca mais nossas idades voltaro a ser todas simultaneamente representadas por nmeros primos.

188

RPM OBMEP

Solues dos Problemas II


Estantes 5 Bombons coloridos B) 20% Carpinteiro modelo A sim modelo B no modelo C sim modelo D sim Dados Questo 1 17 = 21 4 Questo 2 No, Sim, Sim, No (nesta ordem). Sequncia em escada 10 Terremoto C) A probabilidade de haver um terremoto em Zedpolis nos prximos 20 anos maior do que a probabilidade de no haver um terremoto. Feira de Vero B) Pouco provvel. Bate-papo pela Internet Questo 1 10 da manh ou 10h. Questo 2 Qualquer horrio ou intervalo de tempo que satisfaa a diferena de 9 horas e que esteja compreendido entre um dos seguintes intervalos: Sydney: 16:30 h 18:00 h; Berlim: 7:30 h 9:00 h. ou Sydney: 7:00 h 8:00 h; Berlim: 22:00 h 23:00 h.

189

RPM OBMEP

Assaltos No, um crescimento de aproximadamente 10 no muito grande comparado a um total de aproximadamente 500. Colnia de frias: uma soluo
Nome Vermelho Azul Verde Roxo Laranja Amarelo Branco Total meninos 10 meninas 7 7 6 7 4 20 5 26 adultos HH M M H M H M

H = homem M = mulher

190

RPM OBMEP

Solues dos Problemas III


1. Vamos observar inicialmente que (a + b + c)3 = a3 + b3 + c3. Segue-se, ento, que: a2(b + c) + b2(a + c) + c2(a + b) + 2abc = 0 ou equivalentemente, (a + b)(a + c)(b + c) = 0. Para que isso ocorra, pelo menos um dos fatores deve ser nulo. Supondo a + b = 0, a primeira equao nos d c = 3. Substituindo na segunda, obtemos a = 2 e b = 2 . Segue-se que a4 + b4 + c4 = 89. 2. Suponhamos que seja a aranha A1 que percorre todo o ABC e consideremos o instante em que ela est em A. Seja M o ponto mdio do lado A2A3 do A1A2A3 formado pelas trs aranhas nesse instante.

Traamos r e s retas paralelas a BC e que dividem a altura AH em trs partes iguais. Prolongamos AM at encontrar BC em M. Como r e s dividem AM em trs partes iguais (Tales), ento o baricentro do A1A2A3 que o ponto que dista AM/3 de M, est na regio pintada.

191

RPM OBMEP

Repetindo o argumento quando a aranha A1 est em B e quando est em C, conclumos que o baricentro P comum dos tringulos formados pelas trs aranhas est em trs regies que se cortam exatamente no baricentro do ABC, o que demonstra o resultado pedido. Para justificar esta ltima afirmao, observamos que a reta r e as outras duas construdas analogamente, quando a aranha A1 est em B ou C, encontram-se no baricentro do ABC. 3. a) A probabilidade de que o rapaz acerte na primeira tentativa igual a 1/10, uma vez que ele escolhe ao acaso um dos dez dgitos possveis. b) Para que ocorra a segunda tentativa necessrio que ele tenha errado na primeira, e a probabilidade de isso acontecer igual a 9/10. Dado que errou na primeira tentativa, a probabilidade (condicional) de que ele acerte na segunda igual a 1/9, uma vez que, agora, o nmero de dgitos possveis igual a 9. Logo, a probabilidade de que ele acerte na segunda tentativa (9/10)(1/9) = 1/10. Segue que a probabilidade de que ele consiga conversar com a namorada igual a (1/10) + (1/10) = 1/5. 4. Se 10 divide x2 + y2 + xy, ento 2 divide x2 + y2 + xy; logo, x2 + y2 + xy par, implicando x e y pares, o que, por sua vez, implica x2 + y2 + xy mltiplo de 4. Se 10 divide x2 + y2 + xy, ento 5 divide x2 + y2 + xy. Se mostrarmos que isso implica x e y mltiplos de 5, teremos que 25 dividir x2 + y2 + xy, que mltiplo de 4, logo 100 dividir x2 + y2 + xy. Prova de que x e y so mltiplos de 5 Escrevendo x = 5a + b, y = 5c + d, com a, b, c e d inteiros no negativos e 0 < b < 4, 0 < c < 4, obtemos
RPM OBMEP

x2 + y2 + xy = (5a + b)2 + (5c + d)2 + (5a + b)(5c + d) = 5(5a2 + 2ab + 5c2 + 2cd + 5ac + ad + bc) + b2 + d2 + db. Como 5 divide x2 + y2 + xy, temos que 5 divide b2 + d2 + bd.

192

Se b = d 0 temos b2 + d2 + bd = 3b2, que no mltiplo de 5, j que b = 1, 2, 3 ou 4. Se b d, fazendo todas as possveis substituies para b e d em b2 + d2 + bd, obtemos: 12 + 22 + 1 2 = 7, 12 + 32 + 1 3 = 13, 12 + 42 + 1 4 = 21, 22 + 32 + 2 3 = 19, 22 + 42 + 2 4 = 28, 32 + 42 + 3 4 = 37. Como nenhum dos resultados divisvel por 5, conclumos que b = d = 0 e, portanto, x e y so mltiplos de 5. 5. Consideremos o quadrado MNPQ, com lados paralelos s retas paralelas dadas e contendo os vrtices ABCD, como na figura. Se o ngulo indicado, ento DQ = asen e MD = acos. Logo, a medida dos lados do quadrado MNPQ igual a asen + acos. Indicando por z e z as hipotenusas dos tringulos sombreados e por h e h as alturas desses tringulos, temos:
Q D
q
x h y z

a A a
z y h

B
q
x

h + h + a = asen + acos ou h + h = a(sen + cos 1). Por outro h + h' . Sendo lado, zsencos = h e zsencos = h ou z + z' = sen cos x, y, x e y os catetos indicados na figura, temos

x=

h h' h + h' e x' = , o que implica x + x' = ; sen sen sen

193

RPM OBMEP

y=

h cos

e y' =

h' h + h' , o que implica y + y' = . cos cos

A soma, s, dos permetros dos tringulos sombreados ser:

s = x + x + y + y + z + z
s= s= (h + h' )(sen + cos + 1) h + h' h + h' h + h' + + = sen cos sen cos sen cos

a (sen + cos 1)(sen + cos + 1) a[(sen + cos ) 2 1] = sen cos sen cos a 2sen cos s= s = 2a. sen cos

6. Dois naturais m e n resolvem n2n1 + 1 = m2 se e s se m = 2k + 1 e n2n3 = k(k + 1) (*) para algum natural k. fcil verificar diretamente que, para 0 < n < 6, as nicas solues do problema so (n, m) = (0, 1) e (n, m) = (5, 9). Mostremos que, para n > 7, no existe soluo. De fato, observe em (*) que 2n3 divide k ou k + 1. Se 2n3 divide k + 1, ento k divide n e n k +1 = n > k e k + 1 > 2n3 n + 1 > 2n3. k 2 n 3 Se 2n3 divide k, ento k + 1 divide n e n k = n 3 n > k +1 e k > 2n3 n > 2n3 + 1. k +1 2 Porm, fcil ver, por induo, que n + 1 < 2n3 para n > 7. 7.
RPM OBMEP

Seja x = JC, de modo que AC = 3x. No tringulo retngulo ACC, temos que a medida do ngulo CAC igual a 60o e, portanto, a medida do ngulo ACC igual a 30o. Como JH = JC, temos o ngulo JHC igual a 30o; logo, o ngulo HJB mede 60o.

194

x Portanto, B'J = HJ cos 60 = , 2


implicando CB' = x +

x 3 x AC = = , isto 2 2 2

, B ponto mdio de AC. Logo, BB' simultaneamente altura e mediana do ABC, que , portanto, issceles com AB = BC. Portanto, C = A = 60 , o que mostra que ABC de fato equiltero. 8. Como x2 + 1 tem grau 2, temos que o resto da diviso de p(x) por x2 + 1 tem a forma ax + b com a, b . Temos, portanto, p(x) = (x2 + 1)q(x) + ax + b. Fazendo x = i e x = i, obtemos p (i ) p (i ) a = p (i ) = ai + b 2i . p (i ) = ai + b p (i ) + p (i ) b= 2 Pela frmula de Moivre, temos

p (i ) = (cos p (i ) = (cos

2 2 2007 + i sen ) = cos 2 + i sen 2 = 1 e 2007 2007

2 2 2007 2 2 2007 i sen ) = [cos( ) + i sen( )] = 1. 2007 2007 2007 2007

Substituindo nas expresses de a e b, temos a = 0 e b = 1, isto , o resto da diviso 1.


RPM OBMEP

9. Na figura a seguir, seja M o ponto de interseco de XY e CA' . Como MC perpendicular a AB , temos que MC' perpendicular a A'B' . Vamos mostrar primeiramente que A, M e C so colineares,

195

e que, portanto, AC' perpendicular a A'B' . Para isso, basta mostrar que m( AMC ) = m( A' MC' ) . Como X AC' reto, temos '

m( A'MC' ) = 90 m( M AC' ) = m( M A' X ) . '


Agora, como MA = MA e XA = XA, temos que AMA e AXA so issceles e assim m( M A' X ) = m( M AX ) . Mas m( M AX ) = m( AMC ) , pois A'C paralelo a AZ , o que mostra que m( AMC ) = m( A' MC' ) . Assim, os tringulos ACB e ACA so congruentes pelo caso de congruncia LAL, j que BC = BC = AC = AC e o ngulo C reto. Logo, = , pois so ngulos correspondentes. Os tringulos ACA e AZA so congruentes pelo caso especial cateto-hipotenusa, pois AC = AZ. Logo, = e, portanto, = = , ou seja, o ngulo foi dividido em 3 partes iguais. 10. Sendo f: N N tal que f(f(x)) = x, x N, temos: a) f injetora, pois, se f(x1) = f(x2), ento f(f(x1)) = f(f(x2)), isto , x1 = x2. Temos tambm f sobrejetora, pois, dado y N, seja x = f(y) N e ento f(x) = f(f(y)) = y.

n + 1, se n par b) A funo f: N N dada por f (n) = n 1, se n mpar obedece condio f(f(n)) = n, nN, no caso em que N = {0, 1, 2, 3, ..., n, ...}. E no caso que N = {1, 2, 3, ..., n, ...}, basta tomar f(n) = n 1, se n par e f(n) = n + 1, se n mpar.
RPM OBMEP

Observao Existem diversas outras solues. Por exemplo, sejam A = { a1, a2, a3, ...} e B = {b1, b2, b3, ...} conjuntos de uma partio qual-

196

quer de N (isto , A B = N e A B = ). Basta ento definir f(ai) = bi e f(bi) = ai, i natural. A soluo apresentada corresponde partio de N nos conjuntos A e B dos nmeros pares e mpares, respectivamente. 11. Sejam Mi, i = 1, 2, ..., n, cada uma das moedas,
1 , i = 1, 2, ..., n , a probabilidade de se obter cara jogando a 2i + 1 moeda Mi e PTi a probabilidade de haver um nmero mpar de caras jogando-se as moedas M1, M2, ..., Mi. Pi =

Queremos achar PT2007.

1 Para i = 1, temos que PT1 = P = , pois a probabilidade de haver 1 1 3 cara no lanamento de M1.
Para i = 2, temos que PT2 a probabilidade de M1 ser cara e M2 ser coroa ou M1 ser coroa e M2 ser cara: PT2 = P1(1 P2) + (1 P1)P2.
1 1 1 1 4 2 6 2 PT2 = .(1 ) + (1 ). = + = = . 3 5 3 5 15 15 15 5 n 1 2 e PT2 = , parece que PTn = . Vamos tentar 2n + 1 3 5 provar essa igualdade pelo princpio da induo finita.

Sendo PT1 =

Tese: PTn = Temos


PTn1 =

n . 2n + 1

Hiptese: PTn1 =

(n 1) . 2(n 1) + 1

(n 1) n 1 n 1 . Porm, PT n a = = 2(n 1) + 1 2n 2 + 1 2n 1

PTn = (

n n 1 1 n 1 1 . )(1 ) + [1 ( )]( )= 2n + 1 2n 1 2n + 1 2n 1 2n + 1

197

RPM OBMEP

probabilidade de haver um nmero mpar de caras em M1, M2, ..., Mn1 e Mn ser coroa ou haver um nmero par de caras em M1, M2, ..., Mn1 e Mn ser cara. Portanto, PTn = PTn1(1 Pn) + (1 PTn1)Pn ou

Portanto, PT2007 = 12.

2007 2007 = . 2(2007) + 1 4015

Traamos a perpendicular reta CD, por A, que corta o segmento CD no ponto E, pois ADC agudo (mede 60o). O BDE issceles, pois DE = 2acos 60o = a = BD. Logo, o BEC issceles, pois E BC mede 15o e, portanto, BE = EC. Alm disso, BE = AE, pois BEA issceles, j que ABE e B AE medem 30o. Logo, AE = EC e o AEC issceles com E AC medindo 45o. Assim, = 75o.
Observaes 1. No site www.gogeometry.com possvel encontrar outros problemas desse tipo. 2. Este problema tambm pode ser resolvido utilizando-se a lei dos senos ou dos cossenos. Que tal tentar?

13. Para que a equao

x + 2 x 1 + x 2 x 1 = 2 tenha soluo
1 . Nesse caso, o primeiro 2

real necessrio que 2x 1 > 0, isto , x

radicando sempre positivo e para ver que x 2 x 1 basta observar que: ( x 1) 2 0 x 2 2 x 1 x 2 2 x 1 x = x 2 x 1. Para x > 1/2,
RPM OBMEP

tem-se: x soluo da equao se e somente se

( x + 2 x 1 + x 2 x 1 ) 2 = 2 ( x 1) 2 = 1 x x 1 = 1 x

1 x = 1 x 1 x 0 x 1. Logo, o conjunto soluo da equao o intervalo S = [1/2, 1].

198

14. a) No ABC, retngulo em B, M o ponto mdio de AC e MH perpendicular a BC. Como BH = HC, o BMH e o CMH so congruentes e, portanto, BM = MC = AM. Assim, todo tringulo retngulo pode ser decomposto em dois tringulos issceles ABM e BMC.

A M B H C

b) Dado um PQR, qualquer, traamos uma altura interna que o decompe em dois tringulos retngulos. Usando a), vemos que o PQR pode ser decomposto em n = 4 tringulos issceles. Repetindo o procedimento em um, dois, trs ou quatro desses tringulos, decompe-se o PQR em sete, dez, treze ou dezesseis tringulos issceles (que passaremos a chamar de partes). E assim por diante, PQR pode ser decomposto em n = 4 + 3k partes, k . Para decompor em n = 5 partes, consideremos dois casos: i) PQR no equiltero. Decompomos o tringulo em dois tringulos, um deles issceles, e aplicamos a) no outro (dividido em dois tringulos retngulos), obtendo 5 partes.
P

Repetindo o procedimento de a) em uma, duas, trs, quatro ou cinco partes, decompe-se o PQR em oito, onze, catorze, dezessete ou vinte partes. E assim, em n = 5 + 3k partes, k . ii) PQR equiltero. Uma altura separa o PQR em dois tringulos retngulos. Num deles aplicamos a), obtendo duas partes, e decompomos o outro num tringulo issceles e outro retngulo com vrtice comum no circuncentro C do PQR. Aplicando a) nesse tringulo retngulo, decompe-se o PQR em n = 5 partes. Como em i),
P

199

RPM OBMEP

mostra-se que possvel decompor o PQR em n = 5 + 3k partes, k . Um PQR no equiltero pode ser decomposto em trs tringulos, sendo dois issceles, e aplicando a) no terceiro tringulo (dividido em dois tringulos retngulos), decompomos o PQR em n = 6 partes.
P

Q
P

Se PQR equiltero, utilizando o circuncentro C, decompomos em trs partes, e aplicando a) numa delas obtemos a decomposio em 6 partes. Como nos casos anteriores, o PQR pode ser decomposto em n = 6 + 3k partes, k .
Q

C R

15. A funo f deve satisfazer a igualdade 2f(x) + f(1 x) = x2008 para todo x R. Escrevendo a igualdade para (1 x), obtemos 2f(1 x) + f( 1 (1 x)) = (1 x)2008. Ento, 2 f (1 x) + f ( x) = (1 x) 2008 2008 2 f ( x) + f (1 x) = x ou, isolando f(1 x) na segunda equao e substituindo na primeira,
2[ x 2008 2 f ( x)] + f ( x) = (1 x) 2008 , o que leva a

2 x 2008 (1 x) 2008 . 3 fcil verificar que a ltima funo satisfaz a equao funcional dada, logo a nica soluo. f ( x) =
RPM OBMEP

16. Podemos fatorar 32008 1 da forma seguinte: 32008 1 = (38)251 1 = (38 1)(38250 + 38249 + 38248 + ... + 381 + 1). Observe que a soma 38250 + 38249 + 38248 + ... + 381 + 1 tem 251

200

parcelas, todas mpares; logo, um nmero mpar, no sendo, portanto, divisvel por 2. Por outro lado, 38 1 = (34 1)(34 + 1) = 80 82 = 25 5 41. Ento, a maior potncia de 2 que divide 32008 1 25. 17. a) Como cada pea na posio (i, j) substituda por duas, ocupando as posies (i + 1, j) e (i, j + 1), temos que a soma dos pesos no varia a cada jogada, pois 2ij = 2(i +1)j + 2i(j +1). b) Para determinar a soma de todos os pesos do tabuleiro, observamos que a soma da primeira coluna

1 = 2 (PG infinita de razo 11 2

1/2 e termo inicial 1). Por outro lado, a soma da segunda coluna metade da primeira, a da terceira, 1/4 da primeira, e assim por diante, logo a soma de todos os pesos do tabuleiro

2 +1+

1 1 1 2 + + + ... = = 4. 2 4 8 11 2

c) A soma dos pesos das seis casas iniciais sombreadas

1+

1 1 1 1 1 3 + + + + = 2 + , enquanto a soma dos pesos das casas 2 2 4 4 4 4

3 1 no hachuradas 4 (2 + ) = 1 + . 4 4 Assim, impossvel mover as peas das casas hachuradas inicialmente para fora dessa regio, uma vez que a soma dos pesos das
casas iniciais 2 +

3 ser mantida, em cada jogada, e maior do que 4 1 . 4


RPM OBMEP

a soma dos pesos de todas as casas no hachuradas, 1 +

Observao: o problema ainda no teria soluo caso as peas iniciais ocupassem apenas as casas de pesos 1, 1/2, 1/2, j que para atingir o peso 2 da regio no hachurada necessrio utilizar infinitas casas. Logo, o jogo no terminaria aps um nmero finito de movimentos.

201

18. Temos p =
2 n 2008 n

n3 1
3

+1

2 n 2008 ( n + 1)( n

(n 1)(n 2 + n + 1)
2

n + 1)

n 1 n2 + n + 1 . . 2 2 n 2008 n + 1 2 n 2008 n n + 1
n 1 1 2 3 4 2006 2007 1.2 . = ... = n +1 3 4 5 6 2008 2009 2008.2009 2 2 n 2008

Por um lado, temos (1)

Por outro lado, se g(n) = n2 n + 1, temos que g(n + 1) = (n + 1)2 (n + 1) + 1 = n2 + n + 1 e assim (2)

g (n + 1) g (3) g (4) g (2009) = ... g ( n) g (2) g (3) g (2008) 2 n 2008 n n + 1 2 n 2008

n2 + n + 1
2

g (2009) 20092 2009 + 1 . = g (2) 22 2 + 1

De (1) e (2), conclumos que

p=

1.2 20092 2009 + 1 2 2009.2008 + 1 = 2008.2009 3 2008.2009 22 2 + 1

2 1 (1 + ). 3 2008.2009

19. Observe inicialmente que, sendo 7 um nmero primo que divide N, ento necessariamente ele deve dividir algum dos fatores inteiros (x + 6y), (2x + 5y) ou (3x + 4y). Mas ento observe as relaes abaixo:
RPM OBMEP

3(2 x + 5 y ) + ( x + 6 y ) = 7( x + 3 y ) 2(3 x + 4 y ) + ( x + 6 y ) = 7( x + 2 y ) . 2(2 x + 5 y ) + (3 x + 4 y ) = 7( x + 2 y ) Se algum dos fatores for mltiplo de 7, elas implicam que os outros tambm so, portanto N divisvel por 73 = 343.

202

20. Suponhamos que exista um ngulo AOB tal que AO = BO, AC = CD = BD com os ngulos AOC , COD e BOD congruentes. No AOD, a ceviana OC mediana e bissetriz; logo, tambm a altura e o ngulo ACO reto. No COB, a ceviana OD mediana e bissetriz; logo, tambm a altura e o ngulo ODC reto. Assim, o DOC tem dois ngulos retos, o que um absurdo.

203

RPM OBMEP

RPM
Sociedade Brasileira de Matemtica Presidente: Joo Lucas Marques Barbosa Vice-Presidente: Hilrio Alencar Secretrio-Geral: Marco Antnio Teixeira Tesoureiro: Walcy Santos Comit Editorial da RPM Alcila Augusto editora responsvel Ana Catarina P. Hellmeister editora executiva Alberto Carvalho P. de Azevedo Antonio Luiz Pereira Eduardo Wagner Elon Lages Lima Geraldo vila Jos Paulo Q. Carneiro Paulo Cezar Pinto Carvalho Renate G. Watanabe Severino Toscano do Rego Melo

Seleo e Organizao: Comit Editorial da RPM Diagramao e Ilustrao: Mrcio Alexandre de Castro Silvana Cunha de Vasconcelos

RPM Revista do Professor de Matemtica


Rua do Mato, 1010 bloco B sala 105 CEP 05508-090 So Paulo, SP www.rpm.org.br rpm@ime.usp.br telefone/fax: (11) 3091-6124

Vous aimerez peut-être aussi